You are on page 1of 185

gain entry into the house.

Thereafter, they confiscated different personal properties therein which


were allegedly part of those stolen from the employer. They were in possession of a mission
order but later on claimed that the owner of the house gave his consent to the warrantless
search.

ISSUE/s:
Are the things admissible in evidence? Can they be sued for damages as a result of the said
warrantless search and seizure?

Held:

In this case, the petitioners failed to prove, with clear and convincing evidence, that respondent
Paulina Matillano waived her right against unreasonable search and seizure by consenting
thereto, either expressly or impliedly. Admittedly, respondent Paulina Matillano did not object to
the opening of her wooden closet and the taking of their personal properties.

However, such failure to object or resist did not amount to an implied waiver of her right against
unreasonable search and seizure. The petitioners were armed with handguns; petitioner Lui
threatened and intimidated her.

Respondent Eulogio Matillano, her husband, was out of the house when the petitioner and his
cohorts conducted the search and seizure. He could, thus, not have waived his constitutional
right.

The right against unreasonable searches and seizures is a personal right which may be waived
expressly or impliedly. BUT A WAIVER BY IMPLICATION CANNOT BE PRESUMED.
There must be clear and convincing evidence of an actual intention to relinquish the right. There
must be proof of the following:

a. that the right exists;

b. that the person involved had knowledge, either constructive or actual, of the existence of said
right;

c. that the said person had an actual intention to relinquish the right.

Finally, the waiver must be made voluntarily, knowingly and intelligently in order that the said is
to be valid.

The search was therefore held illegal and the members of the searching party held liable for
damages in accordance with the doctrine laid down in Lim vs. Ponce de Leon and MHP
Garments vs. CA.

9. MICROSOFT V. FARAJALLAH

FACTS:
Petitioners:
-Microsoft Corporation and Adobe Systems Incorporated ( corporations under US laws) are
copyright owners respectively of Microsoft software, and all corresponding users manuals and
Microsoft DOS trademarks in the Philipines;
-Adobe Systems copyright relating to all versions and editions of Adobe software.
Respondents: Samir Farajallah, Virgilio D.C. Herce, Rachel P. Follosco, Jesusito G. Morallos
and Ma. Geraldine S. Garcia are the directors and officers of New Fields (Asia Pacific), Inc., a
domestic corporation with principal office at Ortigas Center, Pasig City.

Petitioners claim: September 2009, they were informed that New Fields was unlawfully
reproducing and using unlicensed versions of their software.

Orion Support, Inc.(OSI): Was engaged by petitioners to assist in the verification of this
information. Norma Serrano and Michael Moradoz : trained researchers from OSI were assigned
to confirm and detect unauthorised copies of adobe and Microsoft software.

Filing of complaint and product identification

-On 17 March 2010, counsel for petitioners filed a letter-complaint with the Chief of the
Philippine National Police Criminal Investigation and Detection Group. The case was assigned
to Police Senior Inspector Ernesto V. Padilla.

Padilla and the two witnesses were able to ascertain that the company was indeed using
counterfeit software, by using two computers of New Fields and getting information about the
installed software on the computer, which revealed that they were using common product
identification or serial numbers, an indication that the software is counterfeit; also they noted that
the company used only one installer of the software which were installed on several computers,
numbering about 90 computers.

After being informed of the results of the investigation and warrants

-Petitioners then issued certifications that they have not authorized New Fields to "copy, print,
reproduce and/or publish unauthorized copies of Microsoft and Adobe software products.

-An application for search warrants was filed by Padilla on 20 May 2010, before Judge Amor
Reyes in her capacity as Executive Judge of the RTC.

-The warrants were served on respondents on 24 May 2010. New Fields employees witnessed
the search conducted by the authorities.

-Several items were seized, including 17 CD installers and 83 computers containing


unauthorized copies of Microsoft and/or Adobe software. (resulted in the seizure of more than
5.4 million pesos worth of pirated and unlicensed software)

New Fields then filed on June 6, 2010 a Motion To Quash Search Warrant Search No. 10-
15912. Petitioners received a copy of the motion, which was set for hearing on June 11, 2010,
on June 10, 2010. The trial court also required them file their comment not later than June 21,
2010. Petitioners objected to the motion, averring that it did not comply with the 3-day notice
rule on motions, as they received a copy of the motion only on June 10, 2010, despite the fact
that the motion was set for hearing on June 11, 2010.
RTC: The RTC also dismissed the petitioners objection to the motion for failure to comply with
the 3-day notice rule, citing they were notified personally of the motion. Petitioners moved to
reconsider; this time, respondents objected to the motion for reconsideration for failure to
comply with the 3-day notice rule in motions. Still, the RTC denied petitioners motion for
reconsideration.

The RTC denied petitioners motion for reconsideration in its Order dated 27 August 2010.
Petitioners filed a petition for certiorari under Rule 65 on 8 November 2010 before the Court of
Appeals.

Petitioners alleged that: the RTC committed grave abuse of discretion in granting the Motion to
Quash despite: (1) respondents failure to comply with the three-day notice requirement; and (2)
the existence of probable cause, and personal knowledge of the warrant applicant.

The Ruling of the CA

-The CA denied the petition for certiorari. The appellate court held that: In the instant case, when
the court a quo ordered petitioners to submit their comment on the motion to quash, it was, in
effect, giving petitioners their day in court. Thus, while the [three]-day notice rule was not
strictly observed, its purpose was still satisfied when respondent judge did not immediately rule
on the motion giving petitioners the opportunity to study and oppose the arguments stated in the
motion.

Issue: Whether Judge Amor Reyes committed grave abuse of discretion in granting the motion
to quash search warrants.

Held: We rule that strict compliance with the three-day notice rule may be relaxed in this case.
However, we sustain petitioners contention that there was probable cause for issuance of a
warrant, and the RTC and CA should have upheld the validity of both warrants.

Compliance with the three-day notice rule: In Anama v. Court of Appeals, 29 we ruled that the
three-day notice rule is not absolute. The purpose of the ruleis to safeguard the adverse partys
right to due process. Thus, if the adverse party was given a reasonable opportunity to study the
motion and oppose it, then strict compliance with the three-day notice rule may be dispensed
with.

As correctly pointed out by the CA: In the instant case, when the court a quo ordered petitioners
to submit their comment on the motion to quash, it was, in effect, giving petitioners their day in
court. Thus, while the [three]-day notice rule was not strictly observed, its purpose was still
satisfied when respondent judge did not immediately rule on the motion giving petitioners x x x
the opportunity to study and oppose the arguments stated in the motion
Existence of probable cause: Under Section 1 of Rule 45 of the Rules of Court, petitions for
review by certiorari "shall raise only questions of law." A question of fact exists when there is a
doubt as to the truth of certain facts, and it can only be resolved through a reexamination of the
body of evidence. In Microsoft Corporation v. Maxicorp, Inc., we ruled that the existence of
probable cause is a question of fact.

Probable cause is dependent on the opinion and findings of the judge who conducted the
examination and who had the opportunity to question the applicant and his witnesses. For
this reason, the findings of the judge deserve great weight. The reviewing court should overturn
such findings only upon proof that the judge disregarded the facts before him or ignored the clear
dictates of reason. This Court is not a trier of facts. As a general rule, we defer to the lower
courts appreciation and evaluation of evidence but is not absolute.

Reason to overturn the rulings of the RTC and CA

since there was grave abuse of discretion in the appreciation of facts. The CA sustained the
quashal of the warrant because the witnesses had "no personal knowledge of the facts upon
which the issuance of the warrants may be justified," and the applicants and the witnesses merely
relied on the screen shots acquired from the confidential informant. We disagree with the
conclusions of the CA. The assailed CA Decision itself stated:

Initial hearsay information or tips from confidential informants could very well serve as
basis for the issuance of a search warrant, if followed up personally by the recipient and
validated. It is clear that Padilla and his companions were able to personally verify the tip of
their informant. In his Affidavit submitted to Judge Amor Reyes prior to the issuance of the
warrant, Padilla stated that: At the time that I was inside the office premises of the NEW FIELDS,
I saw the Product Keys or Product Identification Numbers of the ADOBE and MICROSOFT
computer software programs installed in some of the computer units. Ms. Serrano and Mr.
Moradoz were able to pull up these data since they were allowed to use some of the computers of
the target companies in line with the pretext that we used to gain entry into NEW FIELDS. I
actively read and attentively observed the information reflected from the monitor display unit of
the computers that Ms. Serrano and Mr. Moradoz were able to use.

As mentioned earlier, Padilla has been trained to distinguish illegally reproduced Adobe and
Microsoft software. The evidence likewise shows that there was probable cause for the
issuance of a search warrant. Thus, the requirement of personal knowledge of the applicant
and witnesses was clearly satisfied in this case.

WHEREFORE, the petition is GRANTED. The Decisions are hereby REVERSED and SET
ASIDE. Search Warrants are declared valid.

10. JOSE ANTONIO LEVISTE vs. HON. ELMO M. ALAMEDA, HON. RAUL M.
GONZALEZ, HON. EMMANUEL Y. VELASCO and HEIRS OF THE LATE RAFAEL
DE LAS ALAS
FACTS: Leviste was charged with homicide for the death of Rafael de las Alas on January 12,
2007. After paying a P40,000 cash bond approved by the trial court, he was released from
detention, and his arraignment was set on January 24, 2007.

The heirs of De Las Alas then filed a motion praying for the postponement of the Levistes
Arraignment to allow the public prosecutor to re-examine the evidence on record or to conduct
an investigation to determine the proper offense for Leviste.

The RTC favored the motion of De Las Alas heirs and issued an order on January 27, 2004
postponing the arraignment of Leviste and allowing the prosecution to conduct a reinvestigation
to determine Levistes proper offense. Leviste asked the court to reconsider but the request was
denied through a Court Order dated January 31, 2007. Leviste then assailed these orders via
certiorari and prohibition before the Court of Appeals.

While the petition for certiorari was pending on the Court of Appeals, he then filed a motion
before the Trial Court asking the court to not act on the prosecutors recommendation to
reinvestigate until the Court of Appeals renders its decision.

Nonetheless, the trial court still executed the orders suspending Lesvistes arraignment and
allowing the prosecution to reinvestigate. The trial court then issued an order on February 7,
2007 admitting the results of the reinvestigation and directing the issuance for warrant of arrest
for Leviste for the crime of murder. Please note here that prior to the reinvestigation, Leviste was
charged of homicide. Leviste again questioned these orders via supplemental petition before the
appellate court. So at this point, Leviste supplemented the previous petition he file to include the
questioning of the February 7, 2007 order.

Unfortunately, the Court of Appeals denied Levistes petition so the case was brought to the
Supreme Court.

ISSUE: Whether the amendment of the information (as a result of the investigation) from
homicide to murder is considered a substantial amendment, which would make it not just a right
but a duty of the prosecution to ask for a preliminary investigation.
HELD: Yes. A substantial amendment consists of the recital of facts constituting the offense
charged and determinative of the jurisdiction of the court. All other matters are merely of form.
The following have been held to be mere formal amendments: (1) new allegations which relate
only to the range of the penalty that the court might impose in the event of conviction; (2) an
amendment which does not charge another offense different or distinct from that charged in the
original one; (3) additional allegations which do not alter the prosecutions theory of the case so
as to cause surprise to the accused and affect the form of defense he has or will assume; (4) an
amendment which does not adversely affect any substantial right of the accused; and (5) an
amendment that merely adds specifications to eliminate vagueness in the information and not to
introduce new and material facts, and merely states with additional precision something which is
already contained in the original information and which adds nothing essential for conviction for
the crime charged.
The test as to whether a defendant is prejudiced by the amendment is whether a defense
under the information as it originally stood would be available after the amendment is made, and
whether any evidence defendant might have would be equally applicable to the information in
the one form as in the other. An amendment to an information which does not change the nature
of the crime alleged therein does not affect the essence of the offense or cause surprise or deprive
the accused of an opportunity to meet the new averment had each been held to be one of form
and not of substance.
What is essential is that petitioner was placed on guard to defend himself from the charge
of murder after the claimed circumstances were made known to him as early as the first motion.
Petitioner did not, however, make much of the opportunity to present countervailing evidence on
the proposed amended charge. Despite notice of hearing, petitioner opted to merely observe the
proceedings and declined to actively participate, even with extreme caution, in the
reinvestigation.

11. BORLONGAN VS. JUDGE LIMSIACO


TEODORO C. BORLONGAN, JR., et. al. vs. MAGDALENO M. PEA and HON.
MANUEL Q. LIMSIACO, JR., as Judge Designate of the Municipal Trial Court in Cities,
Bago City
FACTS: Magdaleno Pea instituted a civil case for recovery of agents compensation and
expenses, damages, and attorneys fees against Urban Bank and the petitioners, before the RTC
of Negros Occidental, Bago City.

He anchored his claim for compensation on the contract of agency, allegedly entered into
with the petitioners wherein the former undertook to perform such acts necessary to prevent any
intruder and squatter from unlawfully occupying Urban Banks property located along Roxas
Boulevard, Pasay City.

Petitioners filed a Memorandum (MD) arguing that they never appointed the respondent
as agent or counsel.

The following documents were attached to the MD:


a. A letter dated December 19, 1994 signed by Herman Ponce and Julie Abad on behalf of
Isabela Sugar Company, Inc. (ISCI), the original owner of the subject property;
b. An unsigned letter dated December 7, 1994 addressed to Corazon Bejasa from Marilyn
G. Ong;
c. A letter dated December 9, 1994 addressed to Teodoro Borlongan and signed by Marilyn
G. Ong; and
d. A Memorandum dated November 20, 1994 from Enrique Montilla III.
The abovementioned documents were presented in an attempt to show that the respondent
was appointed as agent by ISCI and not by Urban Bank or by the petitioners.
Pea filed his Complaint-Affidavit with the Office of the City Prosecutor, Bago City. He claimed
that said documents were falsified because the alleged signatories did not actually affix their
signatures, and the signatories were neither stockholders nor officers and employees of ISCI.
Worse, petitioners introduced said documents as evidence before the RTC knowing that they
were falsified.
***City Prosecutors Report (Sept 23, 1998): In the report, the Prosecutor concluded that the
petitioners were probably guilty of four (4) counts of the crime of Introducing Falsified
Documents penalized by the second paragraph of Article 172 of the Revised Penal Code
(RPC). The City Prosecutor concluded that the documents were falsified because the alleged
signatories untruthfully stated that ISCI was the principal of the respondent; that petitioners
knew that the documents were falsified considering that the signatories were mere dummies; and
that the documents formed part of the record of Civil Case No. 754 where they were used by
petitioners as evidence in support of their motion to dismiss, adopted in their answer and later, in
their Pre-Trial Brief. Subsequently, the corresponding Informations were filed with the
Municipal Trial Court in Cities (MTCC), Bago City. The cases were docketed as Criminal Cases
Nos. 6683, 6684, 6685, and 6686. Thereafter, Judge Primitivo Blanca issued the warrants for the
arrest of the petitioners.
***Petitioners (Oct. 1, 1998) filed an Omnibus Motion to Quash (MQ): They insist that they
were denied due process because of the non-observance of a proper procedure on preliminary
investigation prescribed in the Rules of Court; since no such counter-affidavit and supporting
documents were submitted by the petitioners, the trial judge merely relied on the complaint-
affidavit and attachments of the respondent in issuing the warrants of arrest, also in
contravention of the Rules. Moreover they claim that the respondents affidavit was not based on
the latters personal knowledge and therefore should not have been used by the court in
determining probable cause.
On the same day that the Omnibous MQ was filed, the petitioners posted bail. Their bail bonds
expressly provided that they do not intend to waive their right to question the validity of their
arrest. On the date of arraignment, the petitioners refused to enter their plea, for the obvious
reason that the legality of their information and their arrest was yet to be settled by the court.
***MTCCs answer (in response to Omnibus MQ filed by petitioners): They upheld the
validity of the warrant of arrest, saying that it was issued in accordance with the Rules. Besides,
(according to the MTCC) petitioners could no longer question the validity of the warrant since
they already posted bail.
ISSUES:
1. W/N petitioners were deprived of their right to due process of law because of the denial
of their right to preliminary investigation and to submit their counter-affidavit; and
2. W/N this Court can, itself, determine probable cause.

HELD: ***For issues 1 and 2:


The following sections of Rule 112 of the 1985 Rules of Criminal Procedure are relevant
to the aforementioned issues:
SECTION 1. Definition. Preliminary investigation is an inquiry or proceeding for the purpose
of determining whether there is sufficient ground to engender a well-founded belief that a crime
cognizable by the Regional Trial Court has been committed and that the respondent is probably
guilty thereof, and should be held for trial.
SEC. 3. Procedure. Except as provided for in Section 7 hereof, no complaint or information for
an offense cognizable by the Regional Trial Court shall be filed without a preliminary
investigation having been first conducted in the following manner:
(a) The complaint shall state the known address of the respondent and be accompanied by
affidavits of the complainant and his witnesses as well as other supporting documents, in such
number of copies as there are respondents, plus two (2) copies of the official file. The said
affidavits shall be sworn to before any fiscal, state prosecutor or government official authorized
to administer oath, or, in their absence or unavailability, a notary public, who must certify that he
personally examined the affiants and that he is satisfied that they voluntarily executed and
understood their affidavits.
SEC. 9. Cases not falling under the original jurisdiction of the Regional Trial Courts not covered
by the Rule on Summary Procedure.
a. Where filed with the fiscal. If the complaint is filed directly with the fiscal or state
prosecutor, the procedure outlined in Section 3 (a) of this Rule shall be observed. The Fiscal
shall take appropriate action based on the affidavits and other supporting documents submitted
by the complainant.
Records show that the prosecutor relied merely on the affidavits submitted by the
complainant and did not require the petitioners to submit their answer. He should not be faulted
for doing such as this is sanctioned by the rules. Moreover, he is not mandated to require the
submission of counter-affidavits. Probable cause may then be determined on the basis alone of
the affidavits and supporting documents of the complainant, without infringing on the
constitutional rights of the petitioners.
Regarding the issuance of the warrant of arrest, petitioners contend that the warrants were
illegally issued as they were solely based on the affidavits of the complainant. Section 2 of
Article III of the Constitution underscores the exclusive and personal responsibility of the
issuing judge to satisfy himself of the existence of probable cause. But the judge is not required
to personally examine the complainant and his witnesses. Following established doctrine and
procedure, he shall (1) personally evaluate the report and the supporting documents submitted by
the prosecutor regarding the existence of probable cause, and on the basis thereof, he may
already make a personal determination of the existence of probable cause; and (2) if he is not
satisfied that probable cause exists, he may disregard the prosecutors report and require the
submission of supporting affidavits of witnesses to aid him in arriving at a conclusion as to the
existence of probable cause. There is no provision or procedural rule which makes the
submission of counter-affidavits mandatory before the judge could determine probable cause.

12. Bureau of Customs vs. Agnes Devanadera, Acting Secretary, Department of Justice

FACTS:

This case deals with the controversy of Unioil Petroleum Philippines, Inc. and its officers
smuggling oil products from Oilink International Inc.s depot in 2008.

Private respondent UNIOIL Petroleum Philippines, Inc. is engaged in marketing,


distribution, and sale of petroleum, oil and other products, while its co-respondent OILINK
International, Inc. is engaged in manufacturing, importing, exporting, buying, selling, or
otherwise dealing in at wholesale and retails of petroleum, oil, gas and of any and all refinements
and byproducts thereof.

Commissioner Napoleon L. Morales of petitioner Bureau of Customs (BOC) issued Audit


Notification Letter (ANL) informing the President of OILINK that the Post Entry Audit Group
(PEAG) of the BOC will be conducting a compliance audit, including the examination,
inspection, verification and/or investigation of all pertinent records of OILINK's import
transactions for the past three (3)-year period.

The events happened in 2007 as follows (you can omit reading this part, however, I
included it just in case Atty. Gacayan would ask about it) :):

1. March 2: A pre-audit conference was held between the BOC Audit Team and the
representatives of OILINK. During the conference, the Audit Team explained to OILINK
representatives the purpose of the post-entry audit and the manner by which it would be
conducted, and advised it as to the import documents required for such audit.

2. March 14: OILINK submitted to the Audit Team the following documents: Post-Entry
Audit Group General Customs Questionnaire, General Information Sheet for the year 2006, SEC
Registration, Articles of Incorporation, Company By-laws, and Audited Financial Report for the
year 2005.

3. April 20: The Audit Team requested OILINK to submit the other documents stated in the
List of Initial Requirements for Submission, namely: 2004 Audited Financial Report, 2004-2006
Quarterly VAT Returns with the accompanying schedule of importations, Organizational
chart/structure, and List of foreign suppliers with details on the products imported and the total
amount, on a yearly basis.

4. May 7: OILINK expressed its willingness to comply with the request for the production
of the said documents, but claimed that it was hampered by the resignation of its employees from
the Accounting and Supply Department. OILINK also averred that it would refer the matter to
the Commissioner of Customs in view of the independent investigation being conducted by the
latter.

5. June 4: OILINK sent a letter stating that the documents which the Audit Team previously
requested were available with the Special Committee of the BOC, and that it could not open in
the meantime its Bureau of Internal Revenue (BIR) - registered books of accounts for validation
and review purposes.

6. July 11: The Audit Team informed OILINK of the adverse effects of its request for the
postponement of the exit conference and its continuous refusal to furnish it the required
documents. It advised OILINK that such acts constitute as waiver on its part to be informed of
the audit findings and an administrative case would be filed against it, without prejudice to the
filing of a criminal action.

7. July 24: Commissioner Morales approved the filing of an administrative case against
OILINK for failure to comply with the requirements of Customs Administrative Order.

8. September 20: An Order was issued by the Legal Service of the BOC, submitting the case
for resolution in view of OILINK's failure to file its Answer within the prescribed period.
9. December 14: The Legal Service of the BOC rendered a Decision finding that OILINK
violated Section IV.A.2(c) and (e) of CAO 4-20047 when it refused to furnish the Audit Team
copies of the required documents, despite repeated demands.

In review of the foregoing events, it was held that OILINK International Inc. is liable for
violating the Tariff and Customs Code of the Philippines. They were ordered to pay the
administrative fine amounting to Php 2,764,859,304.80. Commissioner Morales sent OILINK
demand letters regarding the payment of such fine twice, in 2007 and in 2008. The final demand
letter in 2008 stated that the BOC will be compelled to file the necessary legal action and put in
force a hold order against OILINKs succeeding shipments to protect the governments interest.

A month after, OILINK failed to settle its outstanding account with the BOC so the hold
order was enforced. OILINKs terminal was closed, however, UNIOILs Corporate
Administrative Supervisor, Rochelle Vicencio, requested the BOC (through District Collector
Horacio Suansing, Jr.) to withdraw the process oils and industrial lubricating oils (collectively
base oils) from OILINKs terminal. This was the subject of a prior Terminalling Agreement
between UNIOIL and OILINK. Her request was granted by Commisioner Morales subject to the
following conditions.

1. Only Unioil products shall be withdrawn subject to proper inventory by the BIR and
BOC.

2. Appropriate duties and taxes due on the products to be withdrawn are fully paid or
settled.

3. The company should allow the operation/withdrawal to be closely monitored and


continuously underguarded by assigned Customs personnel.

On May 9, 2008, a Warrant of Seizure and Detention (WSD) was issued by District
Collector Suansing Jr., directing the BOC officials to seal and padlock the oil tanks/depots of
OILINK located in Bataan.

Kenneth Pundanera, Operations Manager of UNIOIL, sent a letter to Zaldy Almoradie,


District Collector of Mariveles, Bataan for permission to relase UNIOIL products from
OILINKs storage terminal. In his letter, he stated that UNIOIL should not be affected with the
unresolved issue between OILINK and the BOC. Even though UNIOIL has already paid all the
duties, fees, and taxes as required, UNIOIL was still unable to withdraw their products from
OILINKs tanks. As a result, UNIOIL could not supply their customers who are using the
products to supply major government infrastructure projects in the country.

Almoradie approved the release, and UNIOIL withdrew their products from OILINKs
tanks. The agreement is that UNIOIL will only withdraw the products which are mentioned in
the request; however, UNIOIL also withdrew products not mentioned in the request. UNIOIL
said that they withdrew the products believing in good faith that they could do so. A complaint-
affidavit was filed against UNIOIL and OILINK by the BOC, accusing them of unlawful
importation and fraudulent practice against customs revenue. Atty. Balmyrson M. Valdez, a
member of the BOC's Anti-Oil Smuggling Coordinating Committee investigated the illegal
withdrawal by UNIOIL of oil products consigned to OILINK. This was valued at P181,988,627.

The State Prosecutor of the DOJ recommended the dismissal of the complaint-affidavit
for lack of probable cause. It was approved by his co-State Prosecutors and subsequently
approved by then Secretary of the DOJ, Raul Gonzales. The BOC filed a Motion for
Reconsideration which was denied by the DOJ Acting Secretary Agnes Devanadera. The BOC
appealed to the CA but they lost because they did not duly submit to the CA another set of
petition with a certification against forum shopping (in essence, the BOC did not follow the
procedural rules).

So, the BOC filed a petition to the Supreme Court raising the issue that the decision of
the CA was erroneous because they overlooked the merits of the case: it was clearly established
that there is probable cause to indict UNIOIL and OILINK for trial for violation of the TCCP.

ISSUE: Whether there was probable cause to indict UNIOIL and OILINK for trial for violation
of the TCCP.

HELD:

SC opined that since the allegations in the BOCs complaint-affidavit fell short of acts or
omissions constituting violations of the Customs code, then DOJ chief Devanadera correctly
ruled that there was no probable cause to believe that Unioil committed such crimes.
A careful reading of the BOCs complaint-affidavit would show that there is no
allegation to the effect that private respondents committed undervaluation, misdeclaration in
weight, measurement or quantity of more than thirty percent [30 percent] between the value,
weight, measurement or quantity declared in the entry, and the actual value, weight, quantity or
measurement which constitute prima facie evidence of fraud. Nor is there an allegation that
[Unioil] intentionally committed undervaluation, misdescription, misclassification or
misdeclaration in the import entry.
13. ROBERT VS CA

FACTS:
Petitioners, who are corporate officers and members of the Board of Pepsi Cola Products Phils.,
Inc. were prosecuted in connection with the Pepsi Number Fever promotion by handlers of the
supposedly winning 349 Pepsi crowns. Of the four cases filed against the petitioners, probable
cause was found by the investigating prosecutor only for the crime of estafa, but not for the other
alleged offenses.

On 12 April 1993, the information was filed with the trial court without anything accompanying
it. A copy of the investigating prosecutors Joint Resolution was forwarded to and received by
the trial court only on 22 April 1993. However, no affidavits of the witnesses, transcripts of
stenographic notes of the proceedings during the preliminary investigation, or other documents
submitted in the course thereof were found in the records of the case as of 19 May 1993.

On 15 April 1993, petitioners Roberts, et al. filed a petition for review to the Department of
Justice seeking the reversal of the finding of probable cause by the investigating prosecutor.
They also moved for the suspension of the proceedings and the holding in abeyance of the
issuance of warrants of arrest against them. Meanwhile, the public prosecutor also moved to
defer the arraignment of the accused-appellants pending the final disposition of the appeal to the
Department of Justice.

On 17 May 1993, respondent Judge Asuncion issued the challenged order (1) denying, on the
basis of Crespo Vs. Mogul, the foregoing motions respectively filed by the petitioners and the
public prosecutor, and directing the issuance of the warrants of arrest after June 1993 and
setting the arraignment on 28 June 1993. In part, respondent judge stated in his order that since
the case is already pending in this Court for trial, following whatever opinion the Secretary of
Justice may have on the matter would undermine the independence and integrity his court. To
justify his order, he quoted the ruling of the Supreme Court in Crespo, which stated:

In order therefor to avoid such a situation whereby the opinion of the Secretary of Justice who
reviewed the action of the fiscal may be disregarded by the trial court, the Secretary of Justice
should, as far as practicable, refrain from entertaining a petition for review or appeal from the
action of the fiscal, when the complaint or information has already been filed in Court. The
matter should be left entirely for the determination of the Court.

Petitioners went to the Court of Appeals (CA), arguing that the respondent judge had not the
slightest basis at all for determining probable cause when he ordered the issuance of warrants of
arrest. After finding that a copy of the public prosecutors Joint Resolution had in fact been
forwarded to, and received by, the trial court on 22 April 1993, the CA denied petitioner's
application for writ of preliminary injunction. The CA ruled that the Joint Resolution was
sufficient in itself to have been relied upon by respondent Judge in convincing himself that
probable cause indeed exists for the purpose of issuing the corresponding warrants of arrest and
that the mere silence of the records or the absence of any express declaration in the questioned
order as to the basis of such finding does not give rise to an adverse inference, for the respondent
Judge enjoys in his favor the presumption of regularity in the performance of his official duty.
Roberts, et al. sought reconsideration, but meanwhile, the DOJ affirmed the finding of probable
cause by the investigating prosecutor. The CA therefore dismissed the petition for mootness.

ISSUE:

Did Judge Asuncion commit grave abuse of discretion in ordering the issuance of warrants of
arrest without examining the records of the preliminary investigation?

HELD:

**The Court, in a 7-5-2 vote, GRANTED the petition. It SET ASIDE the decision and resolution
of the CA, the resolutions of the DOJ 349 Committee, and the order of respondent judge.

YES, Judge Asuncion committed grave abuse of discretion in ordering the issuance of warrants
of arrest without examining the records of the preliminary investigation.
The teachings then of Soliven, Inting, Lim, Allado, and Webb reject the proposition that the
investigating prosecutors certification in an information or his resolution which is made the
basis for the filing of the information, or both, would suffice in the judicial determination of
probable cause for the issuance of a warrant of arrest. In Webb, this Court assumed that since the
respondent Judges had before them not only the 26-page resolution of the investigating panel but
also the affidavits of the prosecution witnesses and even the counter-affidavits of the
respondents, they (judges) made personal evaluation of the evidence attached to the records of
the case.

In this case, nothing accompanied the information upon its filing on 12 April 1993 with the trial
court. A copy of the Joint Resolution was forwarded to, and received by, the trial court only on
22 April 1993. And as revealed by the certification of respondent judges clerk of court, no
affidavits of the witnesses, transcripts of stenographic notes of the proceedings during the
preliminary investigation, or other documents submitted in the course thereof were found in the
records of this case as of 19 May 1993. Clearly, when respondent Judge Asuncion issued the
assailed order of 17 May 1993 directing, among other things, the issuance of warrants of arrest,
he had only the information, amended information, and Joint Resolution as bases thereof. He did
not have the records or evidence supporting the prosecutor's finding of probable cause. And
strangely enough, he made no specific finding of probable cause; he merely directed the issuance
of warrants of arrest after June 21, 1993. It may, however, be argued that the directive
presupposes a finding of probable cause. But then compliance with a constitutional requirement
for the protection of individual liberty cannot be left to presupposition, conjecture, or even
convincing logic.

13. ROBERT VS CA

FACTS:
Petitioners, who are corporate officers and members of the Board of Pepsi Cola Products Phils.,
Inc. were prosecuted in connection with the Pepsi Number Fever promotion by handlers of the
supposedly winning 349 Pepsi crowns. Of the four cases filed against the petitioners, probable
cause was found by the investigating prosecutor only for the crime of estafa, but not for the other
alleged offenses.

On 12 April 1993, the information was filed with the trial court without anything accompanying
it. A copy of the investigating prosecutors Joint Resolution was forwarded to and received by
the trial court only on 22 April 1993. However, no affidavits of the witnesses, transcripts of
stenographic notes of the proceedings during the preliminary investigation, or other documents
submitted in the course thereof were found in the records of the case as of 19 May 1993.

On 15 April 1993, petitioners Roberts, et al. filed a petition for review to the Department of
Justice seeking the reversal of the finding of probable cause by the investigating prosecutor.
They also moved for the suspension of the proceedings and the holding in abeyance of the
issuance of warrants of arrest against them. Meanwhile, the public prosecutor also moved to
defer the arraignment of the accused-appellants pending the final disposition of the appeal to the
Department of Justice.
On 17 May 1993, respondent Judge Asuncion issued the challenged order (1) denying, on the
basis of Crespo Vs. Mogul, the foregoing motions respectively filed by the petitioners and the
public prosecutor, and directing the issuance of the warrants of arrest after June 1993 and
setting the arraignment on 28 June 1993. In part, respondent judge stated in his order that since
the case is already pending in this Court for trial, following whatever opinion the Secretary of
Justice may have on the matter would undermine the independence and integrity his court. To
justify his order, he quoted the ruling of the Supreme Court in Crespo, which stated:

In order therefor to avoid such a situation whereby the opinion of the Secretary of Justice who
reviewed the action of the fiscal may be disregarded by the trial court, the Secretary of Justice
should, as far as practicable, refrain from entertaining a petition for review or appeal from the
action of the fiscal, when the complaint or information has already been filed in Court. The
matter should be left entirely for the determination of the Court.

Petitioners went to the Court of Appeals (CA), arguing that the respondent judge had not the
slightest basis at all for determining probable cause when he ordered the issuance of warrants of
arrest. After finding that a copy of the public prosecutors Joint Resolution had in fact been
forwarded to, and received by, the trial court on 22 April 1993, the CA denied petitioner's
application for writ of preliminary injunction. The CA ruled that the Joint Resolution was
sufficient in itself to have been relied upon by respondent Judge in convincing himself that
probable cause indeed exists for the purpose of issuing the corresponding warrants of arrest and
that the mere silence of the records or the absence of any express declaration in the questioned
order as to the basis of such finding does not give rise to an adverse inference, for the respondent
Judge enjoys in his favor the presumption of regularity in the performance of his official duty.
Roberts, et al. sought reconsideration, but meanwhile, the DOJ affirmed the finding of probable
cause by the investigating prosecutor. The CA therefore dismissed the petition for mootness.

ISSUE:

Did Judge Asuncion commit grave abuse of discretion in ordering the issuance of warrants of
arrest without examining the records of the preliminary investigation?

HELD:

**The Court, in a 7-5-2 vote, GRANTED the petition. It SET ASIDE the decision and resolution
of the CA, the resolutions of the DOJ 349 Committee, and the order of respondent judge.

YES, Judge Asuncion committed grave abuse of discretion in ordering the issuance of warrants
of arrest without examining the records of the preliminary investigation.

The teachings then of Soliven, Inting, Lim, Allado, and Webb reject the proposition that the
investigating prosecutors certification in an information or his resolution which is made the
basis for the filing of the information, or both, would suffice in the judicial determination of
probable cause for the issuance of a warrant of arrest. In Webb, this Court assumed that since the
respondent Judges had before them not only the 26-page resolution of the investigating panel but
also the affidavits of the prosecution witnesses and even the counter-affidavits of the
respondents, they (judges) made personal evaluation of the evidence attached to the records of
the case.

In this case, nothing accompanied the information upon its filing on 12 April 1993 with the trial
court. A copy of the Joint Resolution was forwarded to, and received by, the trial court only on
22 April 1993. And as revealed by the certification of respondent judges clerk of court, no
affidavits of the witnesses, transcripts of stenographic notes of the proceedings during the
preliminary investigation, or other documents submitted in the course thereof were found in the
records of this case as of 19 May 1993. Clearly, when respondent Judge Asuncion issued the
assailed order of 17 May 1993 directing, among other things, the issuance of warrants of arrest,
he had only the information, amended information, and Joint Resolution as bases thereof. He did
not have the records or evidence supporting the prosecutor's finding of probable cause. And
strangely enough, he made no specific finding of probable cause; he merely directed the issuance
of warrants of arrest after June 21, 1993. It may, however, be argued that the directive
presupposes a finding of probable cause. But then compliance with a constitutional requirement
for the protection of individual liberty cannot be left to presupposition, conjecture, or even
convincing logic.

14. SKECHERS, U.S.A., INC. vs. INTER PACIFIC INDUSTRIAL TRADING CORP.

Facts:
Skechers U.S.A. Inc. filed an application for the issuance of search warrants against an
outlet and warehouse operated by Inter Pacific Industrial Trading Corp. for infringement of
trademark under Section 155, in relation to Section 170 of Republic Act No. 8293, Intellectual
Property Code of the Philippines.

Skechers U.S.A. Inc. has registered the trademark "SKECHERS" and the trademark "S"
(within an oval design) with the Intellectual Property Office (IPO). Two search warrants were
issued against Inter Pacific and more than 6,000 pairs of shoes bearing the S logo were seized.
Inter Pacific moved to quash the warrants arguing that there was no confusing similarity between
petitioners "Skechers" rubber shoes and its "Strong" rubber shoes. RTC granted the motion and
quashed the search warrants. Skeckers filed a petition for certiorari with the CA which affirmed
the decision of the RTC. Thus, Skeckers filed the present petition with the SC assailing that the
CA committed grave abuse of discretion when it considered matters of defense in a criminal trial
for trademark infringement in passing upon the validity of the search warrant when it should
have limited itself to a determination of whether the trial court committed grave abuse of
discretion in quashing the warrants. And that it committed grave abuse of discretion in finding
that Inter Pacific are not guilty of trademark infringement in the case where the sole triable issue
is the existence of probable cause to issue a search warrant. Subsequently, Trendworks
International Corporation (petitioner-intervenor) filed a Petition-in-Intervention with the Court
claiming to be the sole licensed distributor of Skechers products here in the Philippines, but the
same was dismissed. Both Skechers and Trendworks filed separate motions for reconsideration.

Issue: Whether Inter Pacific is guilty of trademark infringement.


Ruling: Yes. The essential element of infringement under R.A. No. 8293 (Intellectual Property
Code) is that the infringing mark is likely to cause confusion. In determining similarity and
likelihood of confusion, two tests have been developed: (1) the Dominancy Test which focuses
on the similarity of the prevalent or dominant features of the competing trademarks that might
cause confusion, mistake, and deception in the mind of the purchasing public. Duplication or
imitation is not necessary; neither is it required that the mark sought to be registered suggests an
effort to imitate. (2) The Holistic or Totality Test which necessitates a consideration of the
entirety of the marks as applied to the products, including the labels and packaging, in
determining confusing similarity.

In the case at bar, the Court applied the Dominancy Test and found that the use of the
stylized "S" by respondent in its Strong rubber shoes infringes on the mark already registered by
Skechers with the IPO. While it is undisputed that petitioners stylized "S" is within an oval
design, to this Courts mind, the dominant feature of the trademark is the stylized "S," as it is
precisely the stylized "S" which catches the eye of the purchaser. Thus, even if respondent did
not use an oval design, the mere fact that it used the same stylized "S", the same being the
dominant feature of petitioners trademark, already constitutes infringement under the
Dominancy Test.

The protection of trademarks as intellectual property is intended not only to preserve the
goodwill and reputation of the business established on the goods bearing the mark through actual
use over a period of time, but also to safeguard the public as consumers against confusion on
these goods. While respondents shoes contain some dissimilarities with petitioners shoes, this
Court cannot close its eye to the fact that for all intents and purpose, Inter Pacific had
deliberately attempted to copy Skechers mark and overall design and features of the shoes.

15. MILLER v SEC OF JUSTICE (2011)

FACTS:
The petitioner is George Miller, a British national and an inmate at the Maximum Security
Compound of the New Bilibid Prison (NBP). While he was Acting General of the Inmates
Crusade Against Drugs (ICAD) in Nov-Dec 1998, he wrote to NBP Superintendent Col. Agalo-
os, alleging the drug trading activities of Giovan Bernardino (ICAD Treasurer), Rodolfo
Bernardo (ICAD Chairman) and Ace Aprid, who were inmates at the Medium Security
Compound. Miller recommended that the two be transferred at the Max Security Compound.

Miller was threatened by Bernardo and Bernardino for authoring the letters, and few days later in
Jan 1999, while he was heading towards the volleyball court at around 2:30PM, he was struck
with a piece of wood at the back of his head. He ran to the infirmary for first aid treatment and
was later brought to the NBP Hospital, where the medical specialist reported that the healing
period for his head wound (8-9cm long, 1cm deep) was 7 to 10 days.

The assailants turned out to be Constantino Quirante and Roberto Ceballos, who were ordered
through their "Bosyo" (Commander of their group, the BC45 gang) Rodrigo Toledo to hit Miller,
and in return Bernardino and Bernardo would pay them P1,500.
The day after the assault, they were paid P100 as initial payment. Three days later, they were
called in for investigation, and because the 1,400 balance never came through, they readily
admitted hitting Miller via sworn statements, written in Tagalog. Toledo likewise corroborated
the story with his own sworn statement.

The investigator recommended that Quirante and Ceballos be charged with Frustrated Murder
and the case be placed under further investigation pending the establishment of sufficient
evidence to indict Bernardo, Bernardino, and Aprid.

In March 1999, Prosecutor Antonio Padilla issued a resolution, stating that only Quirante could
be charged for attempted murder in the RTC of Muntinlupa, and that there is insufficient
evidence against the others. However, it was also stated there that in the event that evidence
against the others "may be unearthed by concerned authorities", refiling of the information may
be made.

In Dec 1999, Quirante, Ceballos and Toledo executed new affidavits in English, providing a
more detailed narration, and categorically implicating Bernardo, Bernardino and Aprid as the
masterminds.

In March 2000, Prosecutor Leopoldo Macinas found "probable cause" against all inmates,
amending the information to include Ceballos, Toledo, Bernardino, Bernardo and Aprid as co-
conspirators.

Bernardino filed a petition for review with the DOJ, arguing that there was no sufficient evidence
to support a claim of conspiracy, since it was based merely on "conflicting testimonies or
affidavits in a language foreign to the affiants".

In March 2002, DOJ Secretary Hernando Perez granted the petition and ordered that Bernardino
be excluded from the information, finding that the English affidavits were not part of the
preliminary investigation, and that they "can only be considered as afterthought or made upon
the prodding or influence of other persons".

Miller filed a motion for reconsideration, which was denied, prompting him to elevate the case to
the CA, who also sustained the Sec. Perez's ruling. Hence, this petition for certiorari to the SC.

ISSUES:
1. WON the Sec of Justice erred in disregarding the English affidavits and ruling to exclude
Bernardino in the information.
2. Who determines probable cause?

HELD:
1. Yes. The SC ruled in favor of Miller. Bernardino should be included in the charge. The
English affidavits were mere reiterations and a more detailed account of the incident. They were
consistent with the original Tagalog statements.
There was probable cause to charge all inmates. In a preliminary investigation for the purpose of
filing an information, only prima facie evidence is required, not a full and exhaustive
presentation of evidence (that is reserved for the actual trial). Here, prima facie evidence has
been established against Bernardino through the affidavits.

2. General rule: The Court does not interfere with the preliminary investigation. The
determination of probable cause for the purpose of filing an information in court is an executive
function which pertains at the first instance to the public prosecutor and then to the Secretary of
Justice. The SOJ may reverse or modify the resolution of the prosecutor.

However: The Supreme Court may ultimately resolve the existence or non-existence of probable
cause by examining the records of the preliminary investigation when necessary for the orderly
administration of justice. The SC may substitute the SOJ's decision, such as in this case.

Probable cause is defined as the existence of such facts and circumstances as would excite the
belief in a reasonable mind, acting on the facts within the knowledge of the prosecutor, that the
person charged was guilty of the crime for which he was prosecuted. To determine the existence
of probable cause, there is need to conduct preliminary investigation. A preliminary investigation
constitutes a realistic judicial appraisal of the merits of a case. Its purpose is to determine
whether (a) a crime has been committed; and (b) whether there is a probable cause to believe that
the accused is guilty thereof. It is a means of discovering which person or persons may be
reasonably charged with a crime.

People vs Del Rosario


July 20, 1994
Facts: Normando del Rosario was charged before RTC of Cavite with illegal possession of
Firearms and ammunition, and Illegal sale of Regulated drugs.
On Sept. 4, upon the application of SPO3 Raymundo Unteveros in the RTC of Cavite, a search
warrant was issued authorizing the seizure of an "undeterminate quality of Methamphetamine
hydrochloride commonly known as shabu and its paraphernalias" in the premises of appelants
house in San Roque, Cavite. On the evening of the same day, PO1 Vernando Luna posed as a
posuer buyer and gave the marked 100 peso bill to the accused. After that PO1 went back to the
police station and the raiding team proceeded in the house of the accused. They found a black
canister on the top of tv set of the accused containing shabu, an aluminum foil and a paltik (.22
caliber) with 3 ammunition.
Contention of accused: the police found the accused on his living room and immediately
searched his house without any witnesses. The police then planted the evidence.

Issue: WON the search warrant was validly implemented.


No. The search warrant was not validly implemented. At the case at bar, the accused-appellant
cannot be convicted of possession of the shabu contained in a canister and allegedly seized at his
house, for the charge against him was for selling shabu with the information alleging that the
"accused, without legal authority did . . . sell to a poseur buyer an aluminum foil containing
Methamphetamine Hydrochloride . . ." Sale is totally different from possession. Article 1458 of
the Civil Code defines sale as a contract whereby "one of the contracting parties obligates
himself to transfer the ownership of and to deliver a determine thing, and the other to pay
therefor a price certain in money or its equivalent", while "possession is the holding of a thing or
the enjoyment of a right" as defined by Article 523 of the Civil Code. Accused-appellant cannot
be convicted of a crime which is not charged in the information for to do so would deny him the
due process of law (People vs. Despavellador, 1 SCRA 205 [1961]; People vs. Mori, 55 SCRA
382 [1974]).

Neither can accused-appellant be convicted of illegal possession of firearm and ammunition. The
search warrant implemented by the raiding party authorized only the search and seizure of ". . .
the described quantity of Methamphetamine Hydrochloride commonly known as shabu and its
paraphernalia" (Exh. O, p. 50, original record). Thus, the raiding party was authorized to seize
only shabu and paraphernalia for the use thereof and no other. A search warrant is not a
sweeping authority empowering a raiding party to undertake a finishing expedition to seize and
confiscate any and all kinds of evidence or articles relating to a crime. The Constitution itself
(Section 2, Article III) and the Rules of Court (Section 3, Rule 126) specifically mandate that the
search warrant must particularly describe the things to be seized. Thus, the search warrant was
no authority for the police officers to seize the firearm which was not mentioned, much less
described with particularity, in the search warrant. Neither may it be maintained that the gun was
seized in the course of an arrest, for as earlier observed, accused-appellant's arrest was far from
regular and legal. Said firearm, having been illegally seized, the same is not admissible in
evidence (Stonehill vs. Diokno, 20 SCRA 383 [1967]). The Constitution expressly ordains the
exclusion in evidence of illegally seized articles.

Any evidence obtained in violation of this or the preceding section shall be inadmissible for any
purpose in any proceeding.

(Section 3[2], Article III, Constitution of the Republic of the Philippines).

17. DE LOS SANTOS-REYES v JUDGE MONTESA

Facts: In her complaint filed on 23 March 1993, Guillerma de los Santos-Reyes charges the
respondent judge with gross ignorance of law and evident dishonesty in the performance of his
work in that he granted bail to the accused in 3 Criminal Cases without the required petition for
bail and without conducting any hearing to accord the prosecution an opportunity to establish
that the evidence of guilt of the accused was strong.

The antecedent facts which gave rise to the instant complaint are as follows:
On November 4, 1990, Patrolman Celso Reyes, Bgy. Captain Pedro Panganiban and Armando
Vitug were ambushed along Ipo-road, Kay-Pian, San Juan del Monte, Bulacan, resulting in the
untimely death of Reyes and Panganiban. The NBI conducted an inquisition of the incident and
after which charged petitioners Estelita Hipolito and Alfredo Bolsico, together with Romeo
Adviento, Romeo Permejo, Rolando Gozum and four (4) John Does with the crimes of murder
and frustrated murder before the Municipal Trial Court of San Jose del Monte, then presided
over by Judge Virginia Pagarogon.
Judge Pagarogon conducted a preliminary investigation of the witnesses and on November 14,
1990 issued an order admitting the complaint and ordering the detention of all the accused after
finding that the crimes charged have been committed and there is reasonable ground to believe
that the accused are probably guilty thereof. No bail was recommend.
On April 4, 1991, the trial court, apparently with a change of heart, issued an order consolidating
the petitions for bail, set them for hearing on April 6, 1991, and directed the DOJ and/or the
Office of the Provincial Prosecutor to forward to it the records of the preliminary investigation of
the cases within ten (10) days from notice. library
On April 22, 1991, the accused withdrew their motion for reinstatement of their petition for bail
bond and opted to pursue their motion to quash the warrants of arrest alleging want of probable
cause.
On May 17, 1991, after examining the records of the cases as forwarded to him by the
prosecution, the trial court found the existence of probable cause but instead of issuing the
corresponding warrants of arrest, for the purpose of acquiring jurisdiction over the persons of the
accused upon their apprehension or voluntary surrender, it ex mero motu granted bail to them
despite the absence of (because it was previously withdrawn) a petition for bail and, worse, the
lack of a hearing wherein the prosecution could have been accorded the right to present evidence
showing that the evidence of guilt is strong.

Respondent judge asserts that he is not administratively liable for what he did because he was
merely guided by the doctrine in Lim vs. Felix, to the effect that the determination of probable
cause for the issuance of a warrant of arrest should be personally determined by the judge. Since
in these cases the issuance of the warrants of arrest was based solely on the certification of the
state prosecutor, he granted the motion to quash the warrants of arrest and, considering that on
the date of the hearing to determine probable cause the witnesses for the prosecution did not
appear and the private prosecutor submitted the issue on the basis of the proceedings had at the
preliminary investigation and the affidavits of witnesses, he formally resolved it on such basis.
He further alleges that since he found the evidence purely circumstantial, except as against
Romeo Permejo who was positively identified as the gunman, he believed that the evidence of
guilt as against the others was not strong and, accordingly, admitted them to bail in the amount
of P80,000 each.

Issue: 1.) WON a hearing is necessary for the determination of a probable cause
2.) may a judge grant application for bail without conducting any hearing to accord the
prosecution an opportunity to establish that the evidence of guilt of the accused was strong.

Ruling : A hearing is not necessary. The determination of probable cause in the issuance of a
warrant of arrest is mandated by Section 2, Article III of the Constitution. In satisfying himself
of the existence of probable cause for the issuance of a warrant of arrest, the judge, following the
established doctrine and procedure, shall either:
(a) personally evaluate the report and the supporting documents submitted by the prosecutor
regarding the existence of probable cause and, on the basis thereof, issue a warrant of arrest, or
(b) if on the face of the information he finds no probable cause, he may disregard the prosecutor's
certification and require the submission of the supporting affidavits of witnesses to aid him in
arriving at a conclusion as to the existence of probable cause.
This procedure is dictated by sound public policy; otherwise judges would be unduly laden with
the preliminary examination and investigation of criminal complaints instead of concentrating on
hearing and deciding cases filed before their courts. At this stage of a criminal proceeding, the
judge is not tasked to review in detail the evidence submitted during the preliminary
investigation; : it is sufficient that he (judge) personally evaluates the report and supporting
documents submitted by the prosecution in determining probable cause.
2.) To grant an application for bail and fix the amount thereof without such hearing duly called
for the purpose of determining whether the evidence of guilt is strong constitutes ignorance or
incompetence whose grossness cannot be excused by a claim of good faith or excusable
negligence or constitutes inexcusable conduct which reflects either gross ignorance of the law or
cavalier disregard of its requirements.

18. LIM VS. FELIX


Facts:
An assassination took place in Masbate Domestic Airport. In this incident, Congressman Moises
M
Espinosa and his security escorts were attacked and killed by a lone assassin (bad ass).
Upon preliminary investigation, it was found that probable cause had been established for the
issuances of arrest of accused Vicente Lim Sr. and 11 other dudes for the crime of Multiple
Murder
With Frustrated Murder. This investigation was based on affidavits and answers given by
witnesses .This case was then brought to the RTC of Masbate by respondent Fiscal Alfane.
Petitioners Vicente Lim Sr. and Susana Lim filed a petition to change the venue from the RTC of
Masbate to the RTC of Makati, which was subsequently granted. Their case was raffled to herein
respondent Judge Felix.
In the Makati RTC, Vicente and Susana filed a motion for all the initial records of the
preliminary
investigation which was conducted in Masbate to be transmitted to Makati. This was done so that
the Makati RTC would be best enlightened when they try to determine probable cause/prima
facie evidence against them.
In another manifestation, the Lims reiterated that the court should conduct a hearing to
determine if there really was prima facie evidence against them based on the documents from
the preliminary investigation. This was denied for lack of merit by respondent Judge Felix. In
this
denial, he stated that the investigation in Masbate had already established prima facie evidence
against them, and that this was confirmed by the Provincial Prosecutor in Masbate. Given that
they were both competent officers, he found no reason to re examine the documents. Petitioners
then filed this case questioning respondent judges order.
Issue :
1) WON a judge may issue a warrant of arrest without bail by simply relying on the
prosecutions certification and recommendation that probable cause exists.
Held:
NO! Respondent judge committed grave abuse of discretion by relying solely on
prosecutors certification.
The 1987 Constitution expressly requires probable cause to be personally determined by the
judge (Art 3., Sec. 2). If a judge relies solely on the certification of the Prosecutor like in this
case, he or she has not personally*determined probable cause. This means that a constitutional
requirement has not been met and therefore the judge has committed grave abuse of discretion.
In this case, the records of the preliminary investigation conducted in Masbate were still in
Masbate when the warrants of arrest were issued against the petitioners. There was no basis for
the respondent Judge to make his own personal determination regarding the existence of a
probable cause for the issuance of a warrant of arrest as mandated by the Constitution.
He could
not possibly have known what transpired in Masbate as he had nothing but a certification.
Significantly, the respondent Judge denied the petitioners' motion for the transmittal of the
records on the ground that the mere certification and recommendation of the respondent Fiscal
that a probable cause exists is sufficient for him to issue a warrant of arrest.
The doctrine in Soliven v. Makasiar still stands. The judge does not have to personally examine
the complainant and his witnesses. The Prosecutor can perform the same functions as a
commissioner for the taking of evidence. However, there should be a report and necessary
documents supporting the Fiscals bare certification, which were not present in this case.
What constitutes personal examination depends on the circumstances of each case. The judge
has discretion to be as brief or as detailed in his or her examination.
However, to be sure, the
judge must at least go beyond the Prosecutors bare certification.
In this case, the evidence the petitioners presented to the judge were documents of recantation
(retraction) of witnesses whose testimonies were used to establish prima facie evidence against
them. Although, the general rule is that recantations are not given much weight in the
determination of a case and in the granting of a new trial, the respondent Judge before issuing his
own warrants of arrest should, at the very least, have gone over the records of the preliminary
examination conducted earlier in the light of the evidence now presented by the concerned
witnesses in view of the political undertones prevailing in the cases.

19. [G.R. No. L-8666. March 28, 1956.]


NATALIO P. AMARGA, vs. HONORABLE MACAPANTON ABBAS
Facts:
Natalio Amarga, the Provincial Fiscal of Sulu, filed in the Court of First Instance of Sulu an
information for murder (criminal case No. 1131, People of the Philippines vs. Madpirol, Awadi,
Rajah, Sali, Insa and Maharajah Bapayani). At the foot of the information the Petitioner certified
under oath that he has conducted the necessary preliminary investigation. As the only
supporting affidavit was that of Iman Hadji Rohmund Jubair, to the effect that the latter was
told that the deceased was shot and killed by three persons named: Hajirul Appang, Rajah
Appang and Awadi Bagal. Petitioner had failed or refused to present other evidence sufficient to
make out a prima facie case, leading to the decision of the Respondent dismissing the case
without prejudice to reinstatement.
Issue:
1. Whether or not a judge can hold the issuance of a corresponding warrant of arrest in a
preliminary investigation.
2. Whether or not a judge can dismiss a case without prejudice to reinstatement given the
following circumstances.
Held:
1. Yes. Respondent judge argues that the issuance of a warrant of arrest involves a judicial
power which necessarily imposes upon him the legal duty of first satisfying himself that there is
probable cause, independently of and notwithstanding the preliminary investigation made by the
provincial fiscal.
2. No. While the Respondent Judge was within his right in requiring the Petitioner to submit
further evidence so as to show probable cause for the issuance of a warrant of arrest, he exceeded
his jurisdiction in dismissing the case which was filed with the Court of First Instance of Sulu
not merely for purposes of preliminary investigation. In other words, the failure or refusal of the
Petitioner to present further evidence, although good as a ground for the Respondent Judge not to
issue a warrant of arrest, is not a legal cause for dismissal.
PETITION GRANTED.

10. 20TH CENTURY FOX FILM CORPORATION vs. COURT OF APPEALS,


EDUARDO M. BARRETO, RAUL SAGULLO and FORTUNE LEDESMA

Facts:

In a letter-complaint, 20th Century Fox Film Corporation through counsel sought the NBIs
assistance in the conduct of searches and seizures in certain videotape outlets all over Metro
Manila engaged in the unauthorized sale and renting out of copyrighted films in videotape form
which constitute a flagrant violation of PD No. 49 (otherwise known as the Decree on the
Protection of Intellectual Property).

The NBI conducted surveillance and investigation of the outlets pinpointed by the petitioner and
subsequently filed three (3) applications for search warrants against the video outlets owned by
the private respondents. The applications were consolidated, heard, and issued by the Regional
Trial Court of Makati, Branch 132.

Armed with the search warrants, they raided the video outlets and seized the items described
therein. An inventory of the items seized was made and left with the private respondents. The
latter filed a motion to lift search warrants and release seized properties. The Court lifted the
search warrants.

The petitioner filed a motion for reconsideration in the lower court but it was denied. Then the
petitioner filed a petition for certiorari before the Court of Appeals to annul the decisions made
by the lower court but again it was dismissed.

The petitioner then appealed before the SC maintaining that the lower court issued the
questioned search warrants after finding the existence of a probable cause justifying their
issuance.

Issue:

Whether there is probable cause to issue said search warrants


Held:

No. The respondent Court had an erroneous finding as to the existence of probable cause and
therefore declared the search and seizure to be unreasonable.

Probable cause for a valid search is defined "as such facts and circumstances which would lead a
reasonably discreet and prudent man to believe that an offense has been committed and that the
objects sought in connection with the offense are in the place sought to be searched." Further,
"no less than personal knowledge by the complainant or his witnesses of the facts upon which
the issuance of a search warrant may be justified" in order to convince the judge, not the
individual making the affidavit and seeking the issuance of the warrant, of the existence of a
probable cause.

In the instant case, the lower court lifted the three questioned search warrants in the absence of
probable cause that the private respondents violated P.D. 49. As found out by the court, the NBI
agents who acted as witnesses did not have personal knowledge of the subject matter of their
testimony which was the alleged commission of the offense by the private respondents. Only the
petitioner's counsel who was also a witness during the application for the issuance of the search
warrants stated that he had personal knowledge that the confiscated tapes owned by the private
respondents were pirated tapes taken from master tapes belonging to the petitioner. However, the
lower court did not give much credence to his testimony in view of the fact that the master tapes
of the allegedly pirated tapes were not shown to the court during the application.

Certainly, the action of the lower court correcting such action is within the power and authority
of the respondent Court to perform, provided that it is not exercised in an oppressive or arbitrary
manner. Indeed, the order of the respondent Court declaring the existence of probable cause is
not final and does not constitute res judicata.

21. EDUARDO QUINTERO, petitioner,


vs.
THE NATIONAL BUREAU OF INVESTIGATION, HON. ELIAS ASUNCION, Judge of
the Court of First Instance of Manila, and HON. JOSE FLAMINIANO, City Fiscal of
Pasay City, respondents.
(Sorry its so haba -.- ang daming ganap sa case eh (you can delete the part on the detail about
the amounts received if you like).. p.s. Petitioner is deceased (moot and academic), but the court
still decided on the case)

Facts:
On 19 May 1972, Eduardo Quintero, delegate of the first district of Leyte to the 1971
Constitutional Convention (Con-Con), delivered a privilege speech. He disclosed that certain
persons had distributed money to some delegates of the Con-Con, apparently in an effort to
influence the delegates in the discharge of their functions (He himself had received P11,150.00).
However, he did not reveal the names of the persons who gave him the money; and he begged at
that time not to be made to name names.
However, pressure mounted on Quintero to reveal the Identities of the people behind the
"payola" scheme. Hence, on 30 May 1972 (the day after he returned from Tacloban City, where
he had attended the funeral of his brother), Quintero released from his hospital bed in San Juan
de Dios Hospital, where he was hospitalized due to an indisposed condition, a sworn statement
addressed to the Committee on Privileges of the Con-Con, mentioning the names of the persons
who gave him the "payola." The sworn statement revealed the following:

1. Amount No. 1. P500.00 The envelope containing the amount was handed to
me at the Manila Hotel on March 19, 1971, by Delegate Gabriel Yniquez. He later
made me understand it from the First Lady.
2. Amount No. 2. P500.00 The envelope containing the amount was received
from the office of Representative Nicanor Yniquez of Southern Leyte on April 22,
1971.
3. Amount No. 3. P500.00 The envelope containing the amount was received
from Mrs. Paz Mate (wife of Congressman Mate of Leyte) in May 1971. She told
me that other delegates from Leyte were being given the same amount of money by
the First Lady.
4. Amount No. 4. P500.00 The envelope containing the amount was received in
the house of Congressman Marcelino Veloso on June 2, 1971 from Delegate
Domingo Veloso at Bayview Hotel, Manila. Other envelopes were also given to
other Samar-Leyte delegates.
5. Amount No. 5. P500.00 The envelope containing the amount was handed to
me by Delegate Jaime Opinion on June 10, 1971 in the suite of Delegate Domingo
Veloso at the Bayview Hotel, Manila. Other envelopes were also given to other
Samar-Leyte delegates.
6. Amount No 6 P500.00 The envelope containing this amount was handed to me
by Delegate Domingo Veloso in the Convention Hall on June 23, 1971. He made
me understand it came "from the same source."
7. Amount No. 7. P2,000.00 The envelope containing the amount was handed to
me by Delegate Ramon Salazar on June 27, 1971, in the residence of Delegate
Augusta Syjuco. Delegate Salazar told me that the First Lady met Samar-Leyte
delegates that noon and since I was not in that meeting, the money was being sent to
me.
8. Amount No. 8. P200.00 The envelope containing the amount was handed to
me by Delegate Domingo Veloso on June 28, 1971 during a party given by
President and Mrs. Diosdado Macapagal for the delegates and their ladies. Delegate
Veloso told me the money came from Delegate Augusta Syjuco.
9. Amount No. 9. P500.00 The envelope containing the amount was handed to
me by Delegate Federico dela Plana at the Convention Hall on July 13, 1971.
10. Amount No. 10. P500.00 The envelope containing the amount was left inside
my drawer in the Convention Hall on August 5, 1971 by Delegate Constantino
Navarro, Jr. He said it came from Delegate Venancio Yaneza.
11. Amount No. 11. P500.00 The envelope containing the amount was placed on
my desk under a piece of paper in the session hall on August 11, 1971 by Delegate
Constantino Navarro, Jr. He said it came from Delegate Venancio Yaneza.
12. Amount No. 12. P450.00 The envelope containing the amount was handed to
me by Delegate Domingo Veloso on September 6,1971. He said it came "from
Imelda." According to Delegate Veloso, Yniquez took from the envelope P50.00 for
an unnamed delegate.
13. Amount No. 13. P500.00 The envelope containing the amount was handed to
me on September 23, 1971 by Delegate Domingo Veloso near the men's room. He
said it came "from the First Lady."
14. Amount No. 14. P500.00 The envelope containing the amount was handed to
me on October 6, 1971 by Delegate Domingo Veloso near the office of the
Sergeant-at-Arms. Two other delegates, Delegate Damian Aldaba and Delegate
Antero Bongbong, received an envelope each that same afternoon.
15. Amount No. 15. P500.00 The envelope containing the amount was handed to
me by Delegate Gabriel Yniquez on December 2, 1971 at the entrance of the
Oakroom.
16. Amount No. 16. P1,000.00 The envelope containing the amount was handed
to me by Delegate Gabriel Yniquez on January 13,1972. He said. "This is for the
months of December and January.
17. Amount No. 17. P500.00 The envelope containing the amount was handed to
me on March 7, 1972 by Delegate Flor Sagadal, in the session hall. The envelope
was covered by a piece of paper which Delegate Sagadal placed on my desk.
18. Amount No. 18. P1,000.00 The envelope containing the amount was handed
to me by Delegate Damian Aldaba on May 8, 1972. He said it came from Delegate
Gabriel Yniquez.
Thus, the then First Lady, Mrs. Imelda R. Marcos, among others, was implicated in the Quintero
in expose. Hours after Quintero's statement was made public, President Ferdinand E. Marcos
went on the air and on TV to denounce Mr. Quintero, and averred that he "shall not rest until I
have unmasked this pretender, his master-minds and accomplices."

In the evening of the same day, the agents of the National Bureau of Investigation (NBI) raided
the house of Quintero, at 2281 Mayon St., Sta., Aria, Manila, on the basis of Search Warrant No.
7 issued by Judge Elias Asuncion. After the raid, said NBI agents claimed to have found in the
Quintero residence, and therefore seized, bundles of money amounting to P379,000.00. The NBI
filed with the City Fiscal of Pasay a criminal complaint for direct bribery against Quintero.

Hence, Quintero seeks to annul and declare as void and without legal effect Search Warrant No.
7, issued Judge Elias Asuncion (CFI Manila) as well as all acts and proceedings taken
thereunder.

Issue: WON the search warrant was properly issued and WON the search conducted violates his
rights
Held: 1. No. The search warrant is null and void (no force and effect) since it violates the
constitution and the Rules of Court.

The 1935 Constitution (in force at the time of the issuance of the questioned search warrant)
provides:
Article IIIBill of Rights
Section 1 (3) The rights of the people to be secure in their persons, houses,
papers, and effects against unreasonable searches and seizures shall not be
violated, and no warrants shall issue but upon probable cause, to be determined
by the judge after examination under oath or affirmation of the complainant and
the witnesses he may produce, and particularly describing the place to be
searched, and the persons or things to be seized.
Section 3, Rule 126 of the Rules of Court provided:
Sec. 3. Requisites for issuing search warrant. A search warrant shall not issue
but upon probable cause in connection with one specific offense to be determined
by the judge or justice of the peace after examination under oath or affirmation of
the complainant and the witnesses he may produce, and particularly describing
the place to be searched and the persons or things to be seized.
No search warrant shall issue for more than one specific offense.

A search warrant may issue only upon the finding of the judge of "probable cause," defined as
"such facts and circumstances which would lead a reasonable, discreet and prudent man to
believe that an offense has been committed, and that the objects sought in connection with the
offense are in the place sought to be searched.

In the case at bar, the questioned search warrant was issued by Judge Asuncion, upon application
of NBI agent Samuel Castro it was also accompanied by an affidavit of the complainant,
Congressman Artemio Mate. The interrogations conducted by the respondent judge, upon the
applicant NBI agent Samuel Castro, showed that the latter knew nothing, of his own personal
knowledge, to show that Mr. Quintero had committed any offense. On the other hand, it is clear
from a careful examination of Congressman Mate's statement that, from it, no judicious,
reasonable and prudent man could conclude that probable cause existed that Mr. Quintero had
committed the crime of direct bribery. His statement is replete with conclusions and inferences
drawn from what he allegedly witnessed (seeing 2 unidentified men handing Quintero a folder
which he claims contained the money for the expose) when he visited Mr. Quintero in the
hospital. It lacked the directness and definiteness. As held in one case, persons swearing to, or
supporting the application for, search warrants, must set forth the facts that they know
personally. Search warrants are not issued on loose, vague or doubtful basis of fact, nor on mere
suspicion or belief.

However, Judge Asuncion still issued the warrant. The judge should have been cautious in
issuing the search warrant and should have been alert to some ulterior motives on the part of
Congressman Mate since his wife was one of those implicated in the "expose." "The questions
propounded are not sufficiently searching to establish probable cause. Asking of leading
questions to the deponent in an application for search warrant and conducting of examination in
a general manner would not satisfy the requirements for the issuance of a valid search warrant."
(Probable cause was not established by the Judges investigation/interrogation)

Another circumstance which points to the nullity of the questioned search warrant, for having
been issued without probable cause, is the fact that the search warrant delivered to the occupant
of the searched premises, Generoso Quintero (nephew of Quintero), was issued in connection
with the offense of "grave threats" and not "direct bribery," which was the criminal complaint
filed against Quintero.

The offense charged or labelled in the questioned search warrant had no relation at all to the
evidence, i.e., "half a million pesos, Philippine currency," ordered to be seized in said search
warrant. There was thus no ground whatsoever for the respondent judge to claim that facts and
circumstances had been established, sufficient for him to believe that the crime of "grave threats"
had been committed, because, on the basis of the evidence alone, and what was ordered to be
seized in the search warrant he issued, no relation at all can be established between the crime
supposedly committed (grave threats) and the evidence ordered to be seized.
According to the judge, when the search warrant was presented to him by applicant NBI agent
Samuel Castro, he saw that the crime charged was for "grave threats." But after he allegedly
conducted his interrogations, he found that the proper charge should be "Direct Bribery." Hence
he caused the proper changes in the search warrant, but inadvertently, he failed to make the
proper changes in the sole copy that was presented by the NBI agents to Generoso Quintero,
although the copy retained by the NBI agents had been corrected.

To this, petitioner claimed that the changes in the questioned search warrant were made after the
search was made. According to petitioner, his counsel, Atty. Ordonez who was present during
the latter part of the raid-questioned in fact the materiality of the property being seized to the
offense stated on the warrant, i.e., "grave threats." Consequently, if the copy in the possession of
the raiding party had indeed been corrected before the search, the raiding party, would have been
able to clear up the matter at once, when petitioner's counsel raised the question with them.
However, the raiding party kept silent on the matter at that time, thereby negating their later
pretenses.

The Court notes the admission of the respondents that it was an NBI form which was used for the
search warrant, and that it was pre-filled by the applicant, before it was presented to the
respondent judge, but that, he (the judge) allegedly made the changes after he had conducted his
examination. The Court considers the act of the respondent judge in entertaining a pre-filled
search warrant as irregular; it casts doubt upon his impartiality.

2. Yes. Disregarding for a moment the absence of "probable cause," the search itself that was
conducted by the NBI agents who raided the house of petitioner, pursuant to the questioned
search warrant, was highly irregular. The two (2) occupants of the house who witnessed the
search conducted, Generoso Quintero and Pfc. Alvaro Valentin, were closeted in a room where a
search was being made by a member of the raiding party, while the other NBI agents were left to
themselves in the other parts of the house, where no members of the household were in a position
to watch them, and thus they conducted a search on their own.

Such a procedure, wherein members of a raiding party can roam around the raided premises
unaccompanied by any witness is held to be violative of both the spirit and the letter of the law,
which provides that "no search of a house, room, or any other premises shall be made except in
the presence of at least one competent witness, resident of the neighborhood."
Another irregularity committed by the agents was their failure to comply with the requirement of
Sec. 10, Rule 126 of the Rules of Court which provides that "The officer seizing property under
the warrant must give a detailed receipt for the same to the person on whom or in whose
possession it was found, or in the absence of any person, must, in the presence of at least one-
witness, leave a receipt in the place in which he found the seized property." The receipt issued by
the seizing party in the case at bar, 19 showed that it was signed by a witness, Sgt. Ignacio
Veracruz. This person was a policeman from the Manila Metropolitan Police (MMP), who
accompanied the agents of respondent NBI during the conduct of the search. This requirement of
the Rules was rendered nugatory, when the one who attested to the receipt from the raiding party
was himself a member of the raiding party.

The circumstances prevailing before the issuance of the questioned search warrant, and the actual
manner in which the search was conducted in the house of the petitioner, all but imperfectly, and
yet, strongly suggest that the entire procedure, from beginning to end, was an orchestrated
movement designed for just one purpose to destroy petitioner Quintero's public image with
"incriminating evidence," and, as a corollary to this, that the evidence allegedly seized from his
residence was "planted" by the very raiding party that was commanded to "seize" such
incriminating evidence.

22. THE PRESIDENTIAL ANTI-DOLLAR SALTING TASK FORCE, petitioner,


vs.
HONORABLE COURT OF APPEALS, HONORABLE TEOFILO L, GUADIZ,
JR.,Presiding Judge, REGIONAL TRIAL COURT, Branch 147: NCR (MAKATI), and
KARAMFIL IMPORT-EXPORT CO., INC., respondents.
FACTS: On March 12, 1985, State Prosecutor Jose B. Rosales, who is assigned with the
Presidential Anti-Dollar Salting Task Force, issued search warrants Nos. 156, 157, 158, 159, 160
and 161 against the petitioners Karamfil Import-Export Co., Inc., P & B Enterprises Co., Inc.,
Philippine Veterans Corporation, Philippine Veterans Development Corporation, Philippine
Construction Development Corporation, Philippine Lauan Industries Corporation, Inter-trade
Development (Alvin Aquino), Amelili U. Malaquiok Enterprises and Jaime P. Lucman
Enterprises.
The application for the issuance of said search warrants was filed by Atty. Napoleon Gatmaytan
of the Bureau of Customs who is a deputized member of the PADS Task Force. Attached to the
said application is the affidavit of Josefin M. Castro who is an operative and investigator of the
PADS Task Force. Castro is likewise the sole deponent in the purported deposition to support the
application for the issuance of the six (6) search warrants involved in this case. The application
filed by Atty. Gatmaytan, the affidavit and deposition of Josefin M. Castro are all dated March
12, 1985.
The private respondent (the petitioner) went to the RTC on a petition to enjoin the
implementation of the search warrants in question. On April 16, 1985, the lower court issued the
first of its challenged Orders, and held:
...the Court hereby declares Search Warrant Nos. 156, 157, 158, 159, 160, and 161 to be null
and void. Accordingly, the respondents are hereby ordered to return and surrender immediately
all the personal properties and documents seized by them from the petitioners by virtue of the
aforementioned search warrants.
On April 4, 1986, the Presidential Anti-Dollar Salting Task Force went to the respondent Court
of Appeals to contest, on certiorari, the twin Orders of the lower court. In ruling initially for the
Task Force, the Appellate Court held:
Herein petitioner is a special quasi-judicial body with express powers enumerated under PD
1936 to prosecute foreign exchange violations defined and punished under P.D. No. 1883. The
petitioner, in exercising its quasi-judicial powers, ranks with the Regional Trial Courts, and the
latter in the case at bar had no jurisdiction to declare the search warrants in question null and
void. Besides as correctly pointed out by the Assistant Solicitor General the decision of the
Presidential Anti-Dollar Salting Task Force is appealable to the Office of the President.
On November 12, 1986, Karamfil Import-Export Co., Inc. sought a reconsideration, on the
question primarily of whether or not the Presidential Anti-Dollar Salting Task Force is "such
other responsible officer' countenanced by the 1973 Constitution to issue warrants of search and
seizure. The Court of Appeals, on Karamfil's motion, reversed itself and issued its Resolution,
dated September 1987, and subsequently, its Resolution, dated May 20, 1988, denying the
petitioner's motion for reconsideration.
ISSUE: Whether the Presidential Anti-Dollar Salting Task Force is a quasi-judicial body, and
one co-equal in rank and standing with the Regional Trial Court, and accordingly, beyond the
latter's jurisdiction
RULING: No. This Court finds the Appellate Court to be in error, since what the petitioner puts
to question is the Regional Trial Court's act of assuming jurisdiction over the private
respondent's petition below and its subsequent countermand of the Presidential Anti-Dollar
Salting Task Force's orders of search and seizure, for the reason that the presidential body, as an
entity (allegedly) coordinate and co-equal with the Regional Trial Court, is not vested with such
a jurisdiction. An examination of the Presidential Anti-Dollar Salting Task Force's petition
shows indeed its recognition of judicial review (of the acts of Government) as a basic privilege
of the courts. Its objection, precisely, is whether it is the Regional Trial Court, or the superior
courts, that may undertake such a review.
A quasi-judicial body has been defined as "an organ of government other than a court of law and
other than a legislature, which affects the rights of private parties through either adjudication or
rule making."
A perusal of the Presidential Anti-Dollar Salting Task Force's organic act, Presidential Decree
No. 1936, as amended by Presidential Decree No. 2002, convinces the Court that the Task Force
was not meant to exercise quasi-judicial functions, that is, to try and decide claims and execute
its judgments. As the President's arm called upon to combat the vice of "dollar salting" or the
black marketing and salting of foreign exchange, it is tasked alone by the Decree to handle the
prosecution of such activities, but nothing more.
The Court sees nothing in the provisions of Presidential Decree No. 1936 (except with respect to
the Task Force's powers to issue search warrants) that will reveal a legislative intendment to
confer it with quasi-judicial responsibilities relative to offenses punished by Presidential Decree
No. 1883. Its undertaking is simply to determine whether or not probable cause exists to warrant
the filing of charges with the proper court, meaning to say, to conduct an inquiry preliminary to a
judicial recourse, and to recommend action "of appropriate authorities". It is not unlike a fiscal's
office that conducts a preliminary investigation to determine whether or not prima facie evidence
exists to justify haling the respondent to court, and yet, while it makes that determination, it
cannot be said to be acting as a quasi-court. For it is the courts, ultimately, that pass judgment on
the accused, not the fiscal.
If the Presidential Anti-Dollar Salting Task Force is not, hence, a quasi-judicial body, it cannot
be said to be co-equal or coordinate with the Regional Trial Court. There is nothing in its
enabling statutes that would demonstrate its standing at par with the said court.
In that respect, we do not find error in the respondent Court of Appeal's resolution sustaining the
assumption of jurisdiction by the court a quo.

23. SOLIVEN VS MAKASIAR

FACTS:
Soliven broadcasted the statement that President Aquino hid under her bed during a coup
d' etat. Pres. Cory Aquino filed a criminal complaint for libel against Beltran.
Beltran argues that "the reasons which necessitate presidential immunity from suit impose a
correlative disability to file suit". He contends that if criminal proceedings ensue by virtue of the
President's filing of her complaint-affidavit, she may subsequently have to be a witness for the
prosecution, bringing her under the trial court's jurisdiction. This would in an indirect way defeat
her privilege of immunity from suit, as by testifying on the witness stand, she would be exposing
herself to possible contempt of court or perjury.

ISSUES: In these consolidated cases, three principal issues were raised:


(1) whether or not petitioners were denied due process when informations for libel were filed
against them although the finding of the existence of a prima facie case was still under review by
the Secretary of Justice and, subsequently, by the President;
(2) whether or not the constitutional rights of Beltran were violated when respondent RTC judge
issued a warrant for his arrest without personally examining the complainant and the witnesses,
if any, to determine probable cause; and
(3) whether or not the President of the Philippines, under the Constitution, may initiate criminal
proceedings against the petitioners through the filing of a complaint-affidavit.
HELD:
Subsequent events have rendered the first issue moot and academic.
MARCH 30, 1988: Secretary of Justice denied petitioners motion for reconsideration
APRIL 7, 1988: A second motion for reconsideration filed by petitioner Beltran was
denied by the Secretary of Justice
MAY 2, 1988: On appeal, the President, through Executive Secretary, affirmed the
resolution of the Secretary of Justice
MAY 16, 1988: Motion for reconsideration was denied by the Executive Secretary
With these developments, petitioners' contention that they have been denied the
administrative remedies available under the law has lost factual support.
It may also be added that with respect to petitioner Beltran, the allegation of denial of due
process of law in the preliminary investigation is negated by the fact that instead of submitting
his counter- affidavits, he filed a "Motion to Declare Proceedings Closed," in effect waiving his
right to refute the complaint by filing counter-affidavits. Due process of law does not require that
the respondent in a criminal case actually file his counter-affidavits before the preliminary
investigation is deemed completed. All that is required is that the respondent be given the
opportunity to submit counter-affidavits if he is so minded.
The second issue, raised by petitioner Beltran, calls for an interpretation of the
constitutional provision on the issuance of warrants of arrest. The pertinent provision reads:
Art. III, Sec. 2. The right of the people to be secure in their persons, houses,
papers and effects against unreasonable searches and seizures of whatever nature and
for any purpose shall be inviolable, and no search warrant or warrant of arrest shall
issue except upon probable cause to be determined personally by the judge after
examination under oath or affirmation of the complainant and the witnesses he may
produce, and particularly describing the place to be searched and the persons or things
to be seized.
What the Constitution underscores is the exclusive and personal responsibility of the
issuing judge to satisfy himself of the existence of probable cause. In satisfying himself of the
existence of probable cause for the issuance of a warrant of arrest, the judge is not required to
personally examine the complainant and his witnesses which Beltran contends. Following
established doctrine and procedure, he shall: (1) personally evaluate the report and the supporting
documents submitted by the fiscal regarding the existence of probable cause and, on the basis
thereof, issue a warrant of arrest; or (2) if on the basis thereof he finds no probable cause, he may
disregard the fiscal's report and require the submission of supporting affidavits of witnesses to
aid him in arriving at a conclusion as to the existence of probable cause.
Sound policy dictates this procedure, otherwise judges would be unduly laden with the
preliminary examination and investigation of criminal complaints instead of concentrating on
hearing and deciding cases filed before their courts.
On June 30, 1987, the Supreme Court unanimously adopted Circular No. 12, setting
down guidelines for the issuance of warrants of arrest. The procedure therein provided is
reiterated and clarified in this resolution. It has not been shown that respondent judge has
deviated from the prescribed procedure. Thus, with regard to the issuance of the warrants of
arrest, a finding of grave abuse of discretion amounting to lack or excess of jurisdiction cannot
be sustained.
On the third issue, the rationale for the grant to the President of the privilege of
immunity from suit is to assure the exercise of Presidential duties and functions free from any
hindrance or distraction, considering that being the Chief Executive of the Government is a job
that, aside from requiring all of the office holders time, also demands undivided attention.
But this privilege of immunity from suit, pertains to the President by virtue of the office
and may be invoked only by the holder of the office; not by any other person in the Presidents
behalf. Thus, an accused in a criminal case in which the President is complainant cannot raise the
presidential privilege as a defense to prevent the case from proceeding against such accused.
Moreover, there is nothing in our laws that would prevent the President from waiving the
privilege. Thus, if so minded the President may shed the protection afforded by the privilege and
submit to the courts jurisdiction. The choice of whether to exercise the privilege or to waive it is
solely the Presidents prerogative. It is a decision that cannot be assumed and imposed by any
other person.
Petition is dismissed
26. IN THE MATTER OF THE PETITION FOR HABEAS CORPUS OF ROBERTO
UMIL, ROLANDO DURAL and RENATO VILLANUEVA, MANOLITA O. UMIL and
NICANOR P. DURAL, FELICITAS V. SESE, petitioners,
vs.
FIDEL V. RAMOS, MAJ. GEN. RENATO DE VILLA, BRIG. GEN. RAMON
MONTANO, BRIG. GEN. ALEXANDER AGUIRRE, respondent

Facts: On 1 February 1988, military agents were dispatched to the St. Agnes Hospital,
Roosevelt Avenue, Quezon City, to verify a confidential information which was received by
their office, about a "sparrow man" (NPA member) who had been admitted to the said hospital
with a gunshot wound. That the wounded man in the said hospital was among the five (5) male
"sparrows" who murdered two (2) Capcom mobile patrols the day before, or on 31 January 1988
at about 12:00 o'clock noon, before a road hump along Macanining St., Bagong Barrio, Caloocan
City. The wounded man's name was listed by the hospital management as "Ronnie Javellon,"
twenty-two (22) years old of Block 10, Lot 4, South City Homes, Bian, Laguna however it was
disclosed later that the true name of the wounded man was Rolando Dural. In view of this
verification, Rolando Dural was transferred to the Regional Medical Servicesof the CAPCOM,
for security reasons. While confined thereat, he was positively identified by the eyewitnesses as
the one who murdered the 2 CAPCOM mobile patrols.

On February 6, 1988 a petition for habeas corpus was filed with the court on behalf of
Umil, Dural, and Villanueva.

Issue: Whether or not the detention of the petitioners is lawful

(That the assailed decision, in upholding the validity of the questioned arrests made
without warrant, and in relying on the provisions of the Rules of Court, particularly
Section 5 of Rule 113 (Arrest), disregards the fact that such arrests violated the
constitutional rights of the persons arrested)

Sec. 5. Arrest without warrant; when lawful. A peace officer or a private person may,
without a warrant, arrest a person:
(a) When, in his presence, the person to he arrested has committed, is actually
committing, or is attempting to commit an offense;
(b) When an offense has in fact just been committed, and he has personal knowledge
of facts indicating that the person to be arrest has committed it; and

Held: The court find no merit on the motions of the accused.


The Court's decision of 9 July 1990 rules that the arrest Rolando Dural (G.R. No. 81567) without
warrant is justified it can be said that, within the contemplation of Section 5 Rule 113, he (Dural)
was committing an offense, when arrested because Dural was arrested for being a member of the
New People's Army, an outlawed organization, where membership penalized, 7 and for
subversion which, like rebellion is, under the doctrine of Garcia vs. Enrile, 8 a continuing
offense, thus:
The crimes of insurrection or rebellion, subversion, conspiracy or proposal to commit such
crimes, and other crimes and offenses committed in the furtherance (sic) on the occasion thereof,
or incident thereto, or in connection therewith under Presidential Proclamation No. 2045, are all
in the nature of continuing offenses which set them apart from the common offenses, aside from
their essentially involving a massive conspiracy of nationwide magnitude. . . .

Given the ideological content of membership in the CPP/NPA which includes armed struggle for
the overthrow of organized government, Dural did not cease to be, or became less of a
subversive, FOR PURPOSES OF ARREST, simply because he was, at the time of arrest,
confined in the St. Agnes Hospital. Dural was identified as one of several persons who the day
before his arrest, without warrant, at the St. Agnes Hospital, had shot two (2) CAPCOM
policemen in their patrol car. That Dural had shot the two (2) policemen in Caloocan City as part
of his mission as a "sparrow" (NPA member) did not end there and then. Dural, given another
opportunity, would have shot or would shoot other policemen anywhere as agents or
representatives of organized government. It is in this sense that subversion like rebellion (or
insurrection) is perceived here as a continuing offense. Unlike other so-called "common"
offenses, i.e. adultery, murder, arson, etc., which generally end upon their commission,
subversion and rebellion are anchored on an ideological base which compels the repetition of the
same acts of lawlessness and violence until the overriding objective of overthrowing organized
government is attained.

It may also be said, under the facts of the Umil case, that the arrest of Dural falls under Section
5, paragraph (b), Rule 113 of the Rules of Court, which requires

Two (2) conditions for a valid arrest without warrant


1. that the person to be arrested has just committed an offense
2. the arresting peace officer or private person has personal knowledge of facts indicating
that the person to be arrested is the one who committed the offense

It has been ruled that "personal knowledge of facts," in arrests without warrant must be based
upon probable cause, which means an actual belief or reasonable grounds of suspicion

It is clear in the facts of the case that these requirements had been complied with.

The actual facts supported by circumstances are: first the day before, or on 31 January 1988,
two (2) CAPCOM soldiers were actually killed in Bagong Bario, Caloocan City by five (5)
"sparrows" including Dural; second a wounded person listed in the hospital records as
"Ronnie Javellon" was actually then being treated in St. Agnes Hospital for a gunshot wound;
third as the records of this case disclosed later, "Ronnie Javellon" and his address entered in
the hospital records were fictitious and the wounded man was in reality Rolando Dural.

As to the condition that "probable cause" must also be coupled with acts done in good faith by
the officers who make the arrest, the Court notes that the peace officers wno arrested Dural are
deemed to have conducted the same in good faith, considering that law enforcers are presumed to
regularly perform their official duties. The records show that the arresting officers did not appear
to have been ill-motivated in arresting Dural.

24.Pendon vs. CA

Facts: First lieutenant Filipe Rojas, OIC of the Philippine Constabulary - Criminal Investigation
Services filed an application for search warrant since he was informed that Kenneth Siao of
Kener trading has in his possession the NAPOCOR supplies like bolts, aluminum wires and
other tower parts and line accessory which were illegally acquired. The application of search
warrant was witnessed by an employee of NAPOCOR and an employee of CIS. Judge
Magallanes of Bacolod City then issued the search warrant. The PC officers conducted the raid
and seized bolts and angular bars.The receipt was made by PC Sergeant Digno Mamaril and
marked with "from Kenneth Siao".A complaint was filed against Siao. It was then that he also
made a counter affidavit alleging that he relinquished all his rights and ownership of the keneth
trading to Erle Pendon Jr. (petitioner ). The fiscal recommended the dismissal of case against
Siao and the filing of case against Pendon. The case was assigned to Judge Jocson of Negros
Occidental. Pendon file an application for the return of the seized items on the ground that the
search warrant was illegally issued. Judge Jocson opposed the application since one of the items
were marked with the identity of NPC. The CA also denied the motion for reconsideration.

ISSUE : whether the search warrant was legal.

HELD: No. A search warrant will only be justified upon finding a probable cause. The
application failed to provide facts and circumstances which would lead to believe that an offense
was committed. The judge failed to examine the witnesses carefully in a form of searching
questions and answers to determine whether there was a probable cause. Further the possession
of those seized items were not prohibited by law. Thus the return must be made to Pendon.

25. PEOPLE VS HON. ENRIQUE INTING, G.R. No. 88919


FACTS: In 1988, Mrs. Editha Barba filed a letter-complaint against OIC-Mayor Dominador
Regalado of Tanjay, Negros Oriental with the COMELEC for allegedly transferring her, a
permanent Nursing Attendant, Grade I, in the office of the Municipal Mayor to a very remote
barangay and without obtaining prior permission or clearance from COMELEC as required by
law. The COMELEC then directed Atty. Gerardo Lituanas, then the Provincial Election
Supervisor of Dumaguete City to conduct a preliminary investigation on the case, to prepare and
file the necessary information in court, and to handle the prosecution if it was found that there
was a prima facie case.
After preliminary investigation, Lituanas found a prima facie case, hence filed a case for
violation of sec. 261 par. (h), of the Omnibus Election Code. The court then, through Judge
Inting, on Sept 30, found probable cause and issued a warrant of arrest as recommended by
Lituanas.
But Oct 3, the court set aside the Sept 30 decision because it said that a Provincial Election
Supervisor is not allowed to determine probable cause according to Art 3 Sec. 2 of the
Constitution. The court says it will give the information filed due course only if signed by the
Provincial Fiscal but the same was not given by Lituanas.
ISSUE: Does a preliminary investigation conducted by a Provincial Election Supervisor
involving election offenses have to be coursed through the Provincial Fiscal now Provincial
Prosecutor, before the Regional Trial Court may take cognizance of the investigation and
determine whether or not probable cause exists?
HELD: NO, it does not need the approval of the Provincial Fiscal. While the judge alone
determines probable cause (Art. 3 Sec. 2), the 1987 Constitution empowers the COMELEC to
conduct preliminary investigations in cases involving election offenses for the purpose of helping
the Judge determine probable cause and for filing an information in court. This power is
exclusive with COMELEC (Art. IX-C Sec. 2 par 6). The evident constitutional intendment in
bestowing this power to the COMELEC is to insure the free, orderly and honest conduct of
elections, failure of which would result in the frustration of the true will of the people and make
a mere idle ceremony of the sacred right and duty of every qualified citizen to vote. To divest the
COMELEC of the authority to investigate and prosecute offenses committed by public officials
in relation to their office would thus seriously impair its effectiveness in achieving this clear
constitutional mandate. Bearing these principles in mind, it is apparent that the respondent trial
court misconstrued the constitutional provision when it quashed the information filed by the
Provincial Election Supervisor.

26.B Umil vs. Ramos

Facts:
Rolando Dural was identified as one of the five (5) male sparrows of the New Peoples
Army who shot two (2) CAPCOM policemen. The shooting took place at Bagong, Bario,
Caloocan City during the noon of January 31, 1988. After the incident, the military agents
received information about a wounded man, listed by the hospital management at Ronnie
Javellon, was admitted to St. Agnes Hospital, Roosevelt Avenue, Quezon City. The said man
was then arrested after being identified as Dural. The petition at bar is to ascertain whether the
detention prisoner was illegal or not by looking as to the questioned arrest without warrant was
made in accordance with law.

Issue:
Whether or not the arrest made without warrant is lawful

Held:
Yes. The general rule that no peace officer or person has the power or authority to arrest
anyone without a warrant of arrest has exceptions. The law expressly allowing arrests without
warrant is found in Section 5, Rule 113 of the Rules of Courts which states that an arrest without
a warrant is lawful (1) when the person arrested is actually committing, or is attempting to
commit an offense; and (2) when the person conducting the arrest has personal knowledge of the
facts indicating that the person to be arrested has committed it. In the case at bar, the arrest
warrant was justified because in the contemplation of the said rule, Dural was committing an
offense as he was identified as a member of the NPA, an outlawed organization, where
membership is penalized. Further, his mission as a sparrow did not end at the time of the
commission of murder since subversion like rebellion or insurrection is perceived here as a
continuing offense. Also, the police officers who conducted the arrest has personal knowledge
of facts which are based on probable cause (actual belief or reasonable grounds of suspicion).
They also conducted the arrest in good faith, considering that they are presumed to be
performing their official duties. Thus, the arrest was lawful since it complies with those listed in
Rule 113 or the Rules of Court.

*Note: The mere suspicion of being a Communist Party member of a subversive is absolutely not
a ground for the arrest without warrant of the suspect. The court predicated the validity of the
arrests in compliance with the conditions set forth in Section 5, Rule 113, Rules of Court, a long
existing law, and which, for stress, are probable cause and good faith of the arresting peace
officers, and further, on the basis of, as the records show, the actual facts and circumstance
supporting the arrests

27. ATTY. MIGUEL P. PADERANGA petitioner, vs. HON. FRANKLIN M. DRILON

FACTS:
An information for multiple murder was filed in the Regional Trial Court, Gingoog City, against
Felipe Galarion, Manuel Sabit, Cesar Sabit, Julito Ampo, Eddie Torion, John Doe, Peter Doe and
Richard Doe, for the deaths on May 1, 1984 of Renato Bucag, his wife Melchora Bucag, and
their son Renato Bucag II. Venue was, however, transferred to Cagayan de Oro City. Only Felipe
Garion was tried and found guilty. The others remained at large.
Felizardo Roxas, alias "Ely Roxas," "Fely Roxas" and "Lolong Roxas," was later on included as
a co-accused. Roxas retained petitioner Paderanga as his counsel. Paderanga, as counsel, filed an
Omnibus Motion to dismiss, to Quash the Warrant of Arrest and to Nullify the Arraignment. The
motion was denied and instead, the City Prosecutor was directed "to conduct another preliminary
investigation or reinvestigation in order to grant the accused all the opportunity to adduce
whatever evidence he has in support of his defense.
In the course of the preliminary investigation, Felizardo Roxas was implicated herein petitioner
in the commission of the crime charged. that the preliminary investigation was not yet completed
when said resolution was promulgated, and that he was deprived of his right to present a
corresponding counter-affidavit and additional evidence crucial to the determination of his
alleged "linkage" to the crime charged.
ISSUES:
a. The preliminary investigation is not yet complete and as such, cannot be used as basis for
implicating him for the crime charged.
b. There exists no prima facie evidence or probable cause to justify his inclusion in the second
amended information.
HELD:

a. Preliminary investigation is generally inquisitorial, and it is often the only means of


discovering the persons who may be reasonably charged with a crime, to enable the fiscal to
prepare his complaint or information. It is not a trial of the case on the merits and has no purpose
except that of determining whether a crime has been committed and whether there is probable
cause to believe that the accused is guilty thereof, and it does not place the person against whom
it is taken in jeopardy. As such, he can be implicated in the crime charged even if the preliminary
investigation is not yet complete.
b. A preliminary investigation is defined as an inquiry or proceeding for the purpose of
determining whether there is sufficient ground to engender a well-founded belief that a crime
cognizable by the Regional Trial Court has been committed and that the respondent is probably
guilty thereof, and should be held for trial. The quantum of evidence now required in preliminary
investigation is such evidence sufficient to "engender a well-founded belief as to the fact of the
commission of a crime and the respondent's probable guilt thereof. A preliminary investigation is
not the occasion for the full and exhaustive display of the parties' evidence; it is for the
presentation of such evidence only as may engender a well-grounded belief that an offense has
been committed and that the accused is probably guilty thereof. We are in accord with the state
prosecutor's findings in the case at bar that there exists prima facie evidence of petitioner's
involvement in the commission of the crime, it being sufficiently supported by the evidence
presented and the facts obtaining therein.

29. PEOPLE VS VILLANUEVA

FACTS: On September 4, 1959, the Chief of Police of Alaminos, Laguna, charged Simplicio
Villanueva with the Crime of Malicious Mischief before the Justice of the Peace Court of said
municipality. Said accused was represented by counsel de officio but later on replaced by counsel
de parte. The complainant in the same case was represented by City Attorney Ariston Fule of
San Pablo City, having entered his appearance as private prosecutor, after securing the
permission of the Secretary of Justice. The condition of his appearance as such, was that every
time he would appear at the trial of the case, he would be considered on official leave of absence,
and that he would not receive any payment for his services. The appearance of City Attorney
Fule as private prosecutor was questioned by the counsel for the accused, invoking the case of
Aquino, et al. vs. Blanco, et al.,

L-1532, Nov. 28, 1947, wherein it was ruled that "when an attorney had been appointed to the
position of Assistant Provincial Fiscal or City Fiscal and therein qualified, by operation of law,
he ceased to engage in private law practice." Counsel then argued that the JP Court in
entertaining the appearance of City Attorney Fule in the case is a violation of the above ruling.
On December 17, 1960 the JP issued an order sustaining the legality of the appearance of City
Attorney Fule.

Under date of January 4, 1961, counsel for the accused presented a "Motion to Inhibit Fiscal Fule
from Acting as Private Prosecutor in this Case," this time invoking Section 32, Rule 27, now
Sec. 35, Rule 138, Revised Rules of Court, which bars certain attorneys from practicing. Counsel
claims that City Attorney Fule falls under this limitation. The JP Court ruled on the motion by
upholding the right of Fule to appear and further stating that he (Fule) was not actually enagaged
in private law practice. This Order was appealed to the CFI of Laguna, presided by the Hon.
Hilarion U. Jarencio, which rendered judgment on December 20, 1961, the pertinent portions of
which read:

The present case is one for malicious mischief. There being no reservation by the offended party
of the civil liability, the civil action was deemed impliedly instituted with the criminal action.
The offended party had, therefore, the right to intervene in the case and be represented by a legal
counsel because of her interest in the civil liability of the accused.
ISSUE: Whether or not Ariston Fule is engaged in private law practice.
HELD: In view of the foregoing, this Court holds that Asst. City Attorney Ariston D. Fule may
appear before the Justice of the Peace Court of Alaminos, Laguna as private prosecutor in this
criminal case as an agent or a friend of the offended party.

WHEREFORE, the appeal from the order of the Justice of the Peace Court of Alaminos, Laguna,
allowing the apprearance of Ariston D. Fule as private prosecutor is dismissed, without costs.
side from the considerations advanced by the learned trial judge, heretofore reproduced, and
which we consider plausible, the fallacy of the theory of defense counsel lies in his confused
interpretation of Section 32 of Rule 127 (now Sec. 35, Rule 138, Revised Rules), which provides
that "no judge or other official or employee of the superior courts or of the office of the Solicitor
General, shall engage in private practice as a member of the bar or give professional advice to
clients." He claims that City Attorney Fule, in appearing as private prosecutor in the case was
engaging in private practice. We believe that the isolated appearance of City Attorney Fule did
not constitute private practice within the meaning and contemplation of the Rules. Practice is
more than an isolated appearance, for it consists in frequent or customary actions, a succession of
acts of the same kind. In other words, it is frequent habitual exercise (State vs. Cotner, 127, p. 1,
87 Kan. 864, 42 LRA, M.S. 768). Practice of law to fall within the prohibition of statute has been
interpreted as customarily or habitually holding one's self out to the public, as customarily and
demanding payment for such services (State vs. Bryan, 4 S.E. 522, 98 N.C. 644, 647). The
appearance as counsel on one occasion is not conclusive as determinative of engagement in the
private practice of law. The following observation of the Solicitor General is noteworthy:

Essentially, the word private practice of law implies that one must have presented himself to be
in the active and continued practice of the legal profession and that his professional services are
available to the public for a compensation, as a source of his livelihood or in consideration of his
said services.
For one thing, it has never been refuted that City Attorney Fule had been given permission by his
immediate superior, the Secretary of Justice, to represent the complainant in the case at bar, who
is a relative.

CONFORMABLY WITH ALL THE FOREGOING, the decision appealed from should be, as it
is hereby affirmed, in all respects, with costs against appellant.

30. FISCAL PLACER, et Al. v. JUDGE VILLANUEVA

FACTS

Petitioners are fiscals of Butuan City while the respondent is the judge of Branch I City Court of
Butuan. The case stems from several criminal information which were certified by the
petitioners. According to them, they have conducted preliminary investigations to establish
probable cause, or in the words of Assistant Fiscals Felixberto Guiritan and Ernesto Brocoy
prima facie case, to allow the issuance of the warrants-of-arrest against Renato and Edgardo
Dayan (for Estafa) and Benito Ybaez (for violation of BP 22).
After receipt of the information, Judge Villanueva set a hearing for the purpose of determining
whether he would issue the corresponding warrants of arrest. After the hearing, the judge
required the petitioners to submit affidavits of the prosecution witnesses and other documentary
evidences in support of the information to aid him to find probable cause.
Petitioners filed two motions for reconsideration, contending that under P.D. Nos. 77 and 911,
the petitioners are authorized to determine the existence of a probable cause in a preliminary
examination/investigation, and that their findings as to the existence thereof constitute sufficient
basis for the issuance of warrants of arrest by the court. Nevertheless, respondent judge denied
said motions and reiterated his order to petitioners to submit the supporting affidavits and other
documents within five days from notice. The reason being that it has been the practice ever since
and that he wants to satisfy a finding of probable cause.

ISSUE/S
Whether or not the respondent city judge may, for the purpose of issuing a warrant of arrest,
compel the fiscal to submit to the court the supporting affidavits and other documentary evidence
presented during the preliminary investigation.|

In the same vein, we can ask whether or not the judge, in issuing warrants of arrest, is bound by
the prima facie case/probable cause found by fiscals/prosecutors.

HELD

Yes, he may compel the fiscals to submit to the court the supporting affidavits and other
documentary evidence presented during the preliminary investigation. Because he can, it would
also mean that he is not bound solely by the prima facie case/ probable cause found by
fiscals/prosecutors.

The reason for this can be traced in the letters of Section 6, Rule 112 of the ROC which states:
If the judge be satisfied from the preliminary examination conducted by him or by the
investigating officer that the offense complained of has been committed and that there is
reasonable ground to believe that the accused has committed it, he must issue a warrant
or order for his arrest
What is controlling is not the fact that there was probable cause found in the preliminary
examination of the fiscals but if the judge is satisfied to issue the warrant of arrest. If he is not,
then he may compel the investigating officer, or in this case the fiscal, to present more evidences
or simply to dismiss the case for lack of probable cause. To wit, the issuance of a warrant is not a
mere ministerial function; it calls for the exercise of judicial discretion on the part of the issuing
magistrate.

This is even true given that the cases were summary proceedings and under Section 9, Paragraph
2 of the Rules on Summary Proceedings, it says , the complaint or information must be
accompanied by the affidavits of the complainant and of his witnesses in such number of copies
as there are defendants plus two (2) copies for the court's files.
The obvious purpose of requiring the submission of affidavits of the complainant and of his
witnesses is to enable the court to determine whether to dismiss the case outright or to require
further proceedings. Without the affidavits of the prosecution witnesses and other evidence
which, as a matter of long-standing practice had been attached to the informations filed in his
sala, respondent found the informations inadequate bases for the determination of probable
cause. In previous and ensuing instances, after petitioners had submitted the required affidavits,
respondent wasted no time in issuing the warrants of arrest in the cases where he was satisfied
that probable cause existed.

31. TOLENTINO VS VILLALUZ, JULY 27, 1987

FACTS:
The case stems from eleven petitions filed by Tagaytay City officers. The first two petitions were
filed by the City Mayor and Vice Mayor, while the 3rd up to the 11th were filed by councilors of
Tagaytay City including Bayot, former treasurer of the city and Parra andCastillo who were the
Ating City Auditor and Incumbent Officer in Charge of the City Treasurer Office.

Prior the filing of the 11 petitions, Fiscal Mojica filed a complaint against BAyot, Parra,and
Castillo before the Circuit Criminal Court of Pasig, Rizal which was presided by Judge Villaluz.
The complaint involved violation of the Anti-Graft and Corrupt Practices Act for their refusal to
pay Mojicas salary (4,200/annum) since June, 1969 despite the opinion of the Sec of Justice
concurred in by the Auditor General that the salary of a City Fiscal can be paid out of the
appropriations for City Attorney.

The judge conducted preliminary examination and investigation after which he issued a
resolution ruling that under existing laws, the City fiscal was entitled to a salary from the city;
holding that there exists a prima facie case against Bayot, Parra and Castillo; ordering the
issuance of a warrant of arrest against them; directing the City Fiscal to file the necessary
information against them and to conduct a preliminary examination and investigation in the case
to determine the criminal liability of the members of the said city council and thereafter file the
corresponding information in the court of competent jurisdiction, if the evidence warrants.

After the filing of the necessary information, Espridion MAnalastas filed another complaint
against the three, but this time, the complaint included incumbent City Mayor Atty Isaac
Tolentino, and City Councilors Alfred Ner and Afren Mendiola. Manalastas asserts that they
refused to pay his salary as the Asst City Fiscal from 1971 up to the present despite the opinion
of the Sec of Justice concurred in by the Auditor General that the salary of an Asst. City Fiscal
can be paid out of the appropriations for City Attorney.

Judge Villaluz set the case for prelim investigation, after which the parties of Tolentino filed an
Urgent Motion to Quash/ Dismiss though it was denied by Judge Villaluz. A month later, a
motion for reconsideration was filed for the denial of their previous Urgent Motion to Quash was
filed but same was denied.
Hence this petition which was filed by the parties of Tolentino asserting that Judge Villaluz
giving due course to the complaints filed by Mojica and Manalastas and proceeding with the
preliminary investigation was a violation of law and the constitutional rights of the accused to
due process. They asserted that Criminal Circuit Courts were created with limited jurisdiction,
concurrent with regular courts of first instance, to try and decide only certain specific cases
pursuant to sec 1 of RA 5179.

HELD:
The contention of the petitioners is untenable. As indicated in the previous case of Collector of
Customs vs Villaluz, et. Al., supra, what is limited by RA 5179 is the scope of cases that maybe
tried by the Circuit Criminal Courts. The power of preliminary examination and investigation
exercised by judges of CFI is also possessed by the judges of Circuit Criminal Courts. The main
purpose in creating Criminal Circuit Courts is to alleviate the burden of the regular CFIs and to
accelerate the disposition of cases therein as well as to stem the tide of criminality; hence it is
logical that such authority vested in the judges of CFI is likewise conferred on Circuit Criminal
Courts. Otherwise, the CFI would still be carrying the burden of conducting preliminary
investigations in those cases where the Circuit Criminal Courts have jurisdiction and
consequently delaying the trial and disposition of criminal cases pending before such CFIs.
Another contention of the petitioners is that of improper venue alleging that Circuit Criminal
Courts have no power, authority and jurisdiction to try and decide, much less conduct the
questioned preliminary investigations of cases over crimes which are allegedly committed in the
Province of Cavite, because of improper venue. The rule is that the Circuit Criminal Courts
may hold sessions anywhere within their respective districts but this rule is subject to the
condition that cases shall be heard within the province where the crime subject of the offense
was committed. However, the law also directs that when the interests of justice so demands
with the prior approval of the SC, cases may be heard in neighboring province within the
district, which conditions do not exist in the instant petition such as to justify the hearing in
Pasig of an offense committed in the Province of Cavite.
Accordingly, the petition is dismissed.

Search and Seizure

CRUZ VS. GATAN

FACTS: Serafin Cruz was arrested along Kennon Road at a police checkpoint in Baguio City for
being a member of a subversive organization as ordered by Pres. Ferdinand Marcos in line with
the Martial Law. He was brought to Pampanga where he was held for custodial investigation by
Gen. Romeo Gatan. A Petition for Habeas Corpus was made on his behalf claiming that he was
being held incommunicado, he was being held in custody without court order and he is not a
member of a subversive organizatio n. Cruz, however, opted to stay under the protective custody
of the Philippine Constabulary.
ISSUE: Was the arrest justifiable?

DECISION: The arrest and continued detention was justifiable under General Order No. 2-A, as
amended, the President of the Philippines, pursuant to Proclamation No. 1081, dated September
21, 1972, ordered the Secretary of National Defense "to arrest or cause the arrest and take into
custody and to hold them until otherwise ordered released by me or by my duly designated
representative: 1. Such persons as may have committed crimes and offenses in furtherance or on
the occasion of or incident to or in connection with the crimes of insurrection or rebellion.

OLAES VS. PEOPLE

FACTS: Adolfo Olaes questioned the admission by Judge Alicia Santos of evidence allegedly
acquired through an invalid search warrant and an extrajudicial confession obtained without the
assistance of a counsel. The petitioners claim that the search warrant issued by the respondent
judge is unconstitutional because it does not indicate the specific offense they are supposed to
have committed but was generally stated as a violation of RA 6425 or the Dangerous Drugs Act
of 1972 because the respondents has in their possession and control and custody of marijuana
dried stalks/leaves/seeds/cigarettes and other regulated/prohibited and exempt narcotics
preparations.

ISSUE: Can the evidence against the petitioners be admitted?

DECISION: Yes for those obtained through the search warrant and no for the extrajudicial
confession. The search warrant was regular in all material respects that it is not necessary to
specify which section of the Dangerous Drugs Act was violated because the act charged was
described, which is enough for the judge to personally determine probable cause.

For the extrajudicial confession, it is necessary for the arresting officers to inform the accused of
his right to counsel and the effects of waiving such right. In waiving such right, it must be in
writing, which happened in this case, and the accused must be assisted by counsel, which did not
happen in this case.

Geronimo v Ramos

FACTS: Meliton Geronimo and Bayani Ferrera were candidates in the mayoralty elections in
1980. Geronimo was elected as mayor of Baras, Rizal with 2,695 votes as against Ferreras 2,370
votes (325 margin).

He was subsequently disqualified based on the petition filed by Julian Pendre to disqualify him
in January 19, 1980 on the ground that he was a political turncoat which the COMELEC
affirmed. On January
28, 1980 (2 days before election), Geronimo filed a petition for certiorari to restrain the
COMELEC and was granted. On September 26, 1981, the SC ruled that Geronimo was
disqualified for being a political turncoat. And he COMELEC then proclaimed Ferrera as the
mayor.

On May 3, 1982, the Geronimo together with some of his political followers of more than fifty
persons entered en masse the Municipal Hall of Baras, occupied its premises and continued to do
so until May 13, 1982, causing a paralyzation of official business in the municipality. During this
period, Ferrera held office in his own house. Parenthetically, Geronimo did not enter the office of
the mayor but stayed in another room in the municipal building.

On May 4, 1982, Pendre filed with the COMELEC a motion to cite and declare Geronimo in
contempt. On May 12, 1982, the COMELEC after hearing, issued Resolution No. 82-605,
finding Geronimo guilty of contempt and sentencing him to suffer an imprisonment of five (5)
months and to pay a fine of P1,000.00. In said resolution, the COMELEC simply "noted"
Geronimo's urgent motion because of its previous denial of his motion for reconsideration

At about 2:00 o'clock in the early morning of May 14, 1982, Geronimo and his followers, mostly
women were forcibly taken out of the municipal hall of Baras, Rizal by the military with tile use
of teargas grenades. Gun shots were also fired by the Philippine Constabulary. Some of
Geronimo's followers retaliated with empty bottles when they heard the breaking of the glass
windows of the room where Geronimo was staying. The petitioner was seized, handcuffed, and
brought to the National Penitentiary in Muntinglupa, Rizal.

A series of criminal charges were filed against Geronimo and his 75 followers: : Usurpation of
Authority [Art. 177, Revised Penal Code (RPC)]; Violation of Usurpation of Authority of
Official (sic) [Art. 177, Revised Penal Code (RPC)]; Tumultous Affray [Art. 153, RPC]:
Sedition (Art. 139, RPC); Illegal Possession of Firearms; Disobedience to a Person in Authority
or the Agent of such Person (Art. 151, RPC) and Alarm and Scandal (Art. 155, RPC).

Geronimo filed a petition for habeas corpus with the allegation that his arrest and detention was
done with grave abuse of discretion and without jurisdiction.

ISSUE: WON charges against Geronimo and his followers were issued with grave abuse and
discretion

HELD: There was no grave abuse of discretion on the part of respondent COMELEC when it
held the petitioner guilty of contempt. However, we find the penalty of five (5) months
imprisonment to be harsh. Time and again, this Court has held that the power to punish for
contempt should be exercised on the preservative and not on the vindictive principle, on the
corrective and not on the retaliatory Idea of punishment. The thirteen (13) days during which the
petitioner was confined in the National Penitentiary at Muntinglupa, Rizal more than suffice for
the purpose of serving his sentence for contempt.
The charges were filed almost successively: one on March 20; one on April 12; one on April 14;
two on May 4, two on May 14, and one on May 19, and with the same court and presided over
by the same judge of the Municipal Trial Court of Teresa, Rizal . In one of the criminal
complaints wherein about 75 people were charged, the warrants of arrest were issued on the
same day that the preliminary examination was conducted. Such a hasty and manifestly
haphazard manner of conducting the preliminary examination to determine probable cause for
the issuance of the warrants of arrest and eventually for the filing of the necessary information
cannot be sanctioned by this Court. A judge must first satisfy himself of the existence of probable
cause before issuing a warrant or order of arrest. The requirements are strict. (See Placer v.
Villanueva, 126 SCRA 463). The examination must be legitimate and not a feigned one intended
to justify a course of action already predetermined.

The SC ruled that it was highly improbable for the judge to be able to determine the existence of
reasonable grounds to believe that the offenses have been committed and that each and everyone
of the seventy-six (76) persons are probably guilty thereof in a matter of a few hours and to
proceed with the issuance of the warrants of arrest also on the same day. There was grave abuse
on the part of the judge on the issuance of the warrants of arrests.

ENRILE v SALAZAR

G.R. No. 92163 June 5, 1990

FACTS:

In the afternoon of February 27, 1990, Senate Minority Floor Leader Juan Ponce Enrile was
arrested by law enforcement officers led by Director Alfredo Lim of the National Bureau of
Investigation on the strength of a warrant issued by Hon. Jaime Salazar of the Regional Trial
Court of Quezon City Branch 103, in Criminal Case No. 9010941.

The warrant had issued on an information signed and earlier that day filed by a panel of
prosecutors composed of Senior State Prosecutor Aurelio C. Trampe, State Prosecutor Ferdinand
R. Abesamis and Assistant City Prosecutor Eulogio Mananquil, Jr., charging Senator Enrile, the
spouses Rebecco and Erlinda Panlilio, and Gregorio Honasan with the crime of rebellion with
murder and multiple frustrated murder allegedly committed during the period of the failed coup
attempt against then President Corazon Aquino from November 29 to December 10, 1990.

Senator Enrile was taken to and held overnight at the NBI headquarters on Taft Avenue, Manila,
without bail, none having been recommended in the information and none fixed in the arrest
warrant. The following morning, February 28, 1990, he was brought to Camp Tomas Karingal in
Quezon City where he was given over to the custody of the Superintendent of the Northern
Police District, Brig. Gen. Edgardo Dula Torres.
On the same date of February 28, 1990, Senator Enrile, through counsel, filed the petition for
habeas corpus herein (which was followed by a supplemental petition filed on March 2, 1990),
alleging that he was deprived of his constitutional rights. He questioned the regularity of the
issuance of the warrant of arrest against him. He claimed that it only took Judge Salazar one hour
and twenty minutes from the raffling of the case to him to issue the warrant. Enrile claimed that
such period is so short that it was impossible for the judge to have been able to examine the
voluminous record of the case from the prosecutions office.

Enrile also claimed that the crime charged was non-existent. He insists that there is no such
crime as rebellion with murder and multiple frustrated murder citing the landmark case of People
vs Hernandez where it was ruled that rebellion cannot be complexed with common crimes such
as murder; as such, the proper crime that should have been charged against him is simple
rebellion which is bailable.

For the prosecution, the Solicitor General argued that the Hernandez ruling should be abandoned
and that it should be ruled that rebellion cannot absorb more serious crimes like murder.

ISSUE: WON not Judge Salazar personally determined probable cause in the case at bar

HELD: This Court has already ruled, however, that it is not the unavoidable duty of the judge to
make such a personal examination, it being sufficient that he follows established procedure by
personally evaluating the report and the supporting documents submitted by the prosecutor.
Petitioner claims that the warrant of arrest issued barely one hour and twenty minutes after the
case was raffled off to the respondent Judge, which hardly gave the latter sufficient time to
personally go over the voluminous records of the preliminary investigation. Merely because said
respondent had what some might consider only a relatively brief period within which to comply
with that duty, gives no reason to assume that he had not, or could not have, so complied; nor
does that single circumstance suffice to overcome the legal presumption that official duty has
been regularly performed.

PEOPLE v MENGOTE

FACTS:The Western Police District of Manila received a tip that there were suspicious persons
lurking at North Bay Boulevard in Tondo. As stated by Patrolmen Mercado and Juan in the trial,
they saw two men looking from side to side, one of whom was holding his abdomen. The two
tried to run away, when the officers introduced themselves as policemen. Mengote and his
companion failed to escape because the other police were just around. They were then searched.
Mengote was found with a .38 caliber Smith and Weeson revolver with 6 live bullets. His
companion, Nicanor Morellos, had a fan knife
Aside from the police officers, the prosecution presented a withness, Rigoberto Danganan. The
witness identified the weapon confiscated from Mengote as one of those robbed in his house in
Malabon. The accused contended that the weapon had been planted. Mengote was convicted of
violating PD 1866 for illegally possessing a firearm and was sentenced with the penalty of
reclusion perpetua.

ISSUE: Whether or not the revolver should not have been admitted in evidence because of its
illegal seizure.

HELD: The Supreme Court reiterated the importance of Article III, Section 3(2) of the
Constitution. However, the Solicitor General stated that the said provision is not applicable in the
case at bar. The lawful arrest of Mengote was based on Rule 113, Section 5 of the Rules of
Court.
Sec. 5. Arrest without warrant when lawful. A peace officer or private person may, without a
warrant, arrest a person;
(a) When, in his presence, the person to be arrested has committed, is actually committing, or is
attempting to commit an offense;
(b) When an offense has in fact just been committed, and he has personal knowledge of facts
indicating that the person to be arrested has committed it; and
(c) When the person to be arrested is a prisoner who has escaped from a penal establishment or
place where he is serving final judgment or temporarily confined while his case is pending, or
has escaped while being transferred from one confinement to another.
In cases failing under paragraphs (a) and (b) hereof, the person arrested without a warrant shall
be forthwith delivered to the nearest police station or jail, and he shall be proceeded against in
accordance with Rule 112, Section 7.
Paragraph C is inapplicable to Mengote as he did not escape from any penal institution.
Requirements for Paragraph A is 1) after he has committed or while he is actually committing or
is at least attempting to commit an offense, (2) in the presence of the arresting officer. Looking
from side to side and holding his abdomen at 11:30 in the morning is definitely not an
offense. These acts did not establish the requirements of paragraph A. Par. B is no less applicable
because its no less stringent requirements have also not been satisfied. The prosecution has not
shown that at the time of Mengote's arrest an offense had in fact just been committed and that the
arresting officers had personal knowledge of facts indicating that Mengote had committed it. All
they had was hearsay information from the telephone caller, and about a crime that had yet to be
committed.
WHEREFORE, the appealed decision is REVERSED and SET ASIDE. The accused-appellant is
ACQUITTED and ordered released immediately unless he is validly detained for other offenses.
PEOPLE V GO (2001)

FACTS:On October 22, 1992, at around 10:00 PM, SPO1 Mauro Piamonte and SPO3 Candido
Liquido, members of the Intelligence and Follow-up Unit of the Calamba Police, went to the
police outpost at Crossing, Calamba, to follow up an intelligence report that methamphetamine
hydrochloride, or shabu, was being supplied there. Police civilian agent Ronnie Panuringan
reported that he saw accused-appellant Luisito Go, also known as King Louie, enter the
Flamingo Disco House with two women, with a gun tucked in his waist. The policemen
proceeded to the Flamingo, 100m away from the outpost, and informed the owner that they were
conducting an Operation Bakal, whereby they search for illegally possessed firearms. The owner
allowed them in and told a waiter to accompany them to the second floor of the disco. The waiter
turned on the lights, and the police officers saw Go and his lady companions seated at a table.
They identified themselves and asked Go to stand up. When the he did so, the policemen saw the
gun tucked in his waist. SPO1 Piamonte asked for the license of the gun, but accused-appellant
was unable to produce any. Instead, accused-appellant brought out the drivers license of a
certain Tan Antonio Lerios. SPO1 Piamonte confiscated the gun, which was later identified as a
9mm Walther P88, Serial Number 006784, with a magazine containing ten (10) rounds of live
ammunition. Go was invited to the police precinct for questioning.

On the way out of the disco, Go asked permission to bring his car, which was parked outside.
The police officers accompanied him to his Honda Civic. Through the windshield, SPO3 Liquido
noticed a PNP ID hanging from the rearview mirror. He asked Go if he was a member of the
PNP, and he said no. The police officers asked him for his drivers license and registration
papers, but he was unable to produce them. When Go opened the door, SPO3 Liquido took the
ID and found that it belonged to SPO4 Zenaida Bagadiong. The police officers saw pieces of
glass tooters and tin foils on the backseat and floor of the car. They asked Go why he had these
items, but he did not say anything. Instead, he suggested that they talk the matter over, and
intimated that he had money. SPO3 Liquido replied that they should talk at the police
headquarters. Accused-appellant took out an attach case from the car and opened it. There were
two black clutch bags inside. Accused-appellant opened the first bag, which contained shiny
white substance wrapped in cellophane. The second bag contained P120,000.00 cash.

The police officers brought Go the police station. The investigator found eight cellophane bags
containing granules suspected to be shabu in one of the clutch bags. When the attach case was
opened, the police officers found that it also contained three glass tooters, tin foils, an improvised
burner, magazines and newspapers.

Go was charged for two offenses: (1) violation of RA 6452 or the Dangerous Drugs Act for
possession of 750 grams of methamphetamine hydrochloride, and (2) violation of PD 1866 for
possession of unlicensed loaded firearm and ammunition. He was found guilty on both by the
trial court, affirmed by the CA.
ISSUE: WON the warrantless search and seizure of the gun and thereafter, the shabu, were legal.

HELD: Yes.

The constitutional proscription, that no person shall be arrested without any warrant of arrest is
not a hard-and-fast rule. The Rules of Court and jurisprudence recognize exceptional cases where
an arrest may be effected without a warrant. Among these are when, in the presence of a peace
officer, the person to be arrested has committed, is actually committing, or is attempting to
commit an offense; or when an offense has in fact just been committed, and the arresting officer
has personal knowledge of facts indicating that the person to be arrested has committed it.

In the cases at bar, the police saw the gun tucked in appellants waist when he stood up. The gun
was plainly visible. No search was conducted as none was necessary. Accused-appellant could
not show any license for the firearm, whether at the time of his arrest or thereafter. Thus, he was
in effect committing a crime in the presence of the police officers. No warrant of arrest was
necessary in such a situation, it being one of the recognized exceptions under the Rules.

As a consequence of appellants valid warrantless arrest, he may be lawfully searched for


dangerous weapons or anything which may be used as proof of the commission of an offense,
without a search warrant, as provided in Rule 126, Section 12. This is a valid search incidental to
the lawful arrest. The subsequent discovery in his car of drug paraphernalia and the crystalline
substance, which was later identified as shabu, though in a distant place from where the illegal
possession of firearm was committed, cannot be said to have been made during an illegal search.
As such, the seized items do not fall within the exclusionary clause, which states that any
evidence obtained in violation of the right against warrantless arrest cannot be used for any
purposes in any proceeding. Hence, not being fruits of the poisonous tree, so to speak, the objects
found at the scene of the crime, such as the firearm, the shabu and the drug paraphernalia, can be
used as evidence against appellant. Drugs discovered as a result of a consensual search is
admissible in evidence.

PITA VS. CA

FACTS: On December 1 and 3, 1983, pursuing an Anti-Smut Campaign initiated by the Mayor
of the City of Manila, Ramon D. Bagatsing, elements of the Special Anti-Narcotics Group,
Auxilliary Services Bureau, Western Police District, INP of the Metropolitan Police Force of
Manila, seized and confiscated from dealers, distributors, newsstand owners and peddlers along
Manila sidewalks, magazines, publications and other reading materials believed to be obscene,
pornographic and indecent and later burned the seized materials in public at the University belt
along C.M. Recto Avenue, Manila, in the presence of Mayor Bagatsing and several officers and
members of various student organizations. Among the publications seized, and later burned, was
"Pinoy Playboy" magazines published and co-edited by plaintiff Leo Pita.
He filed an injunction case against the mayor of manila to enjoin him from confiscating more
copies of his magazine and claimed that this was a violation of freedom of speech. The court
ordered him to show cause. He then filed an Urgent Motion for issuance of a temporary
restraining order against indiscriminate seizure.

Defendant Mayor Bagatsing admitted the confiscation and burning of obscence reading materials
but admitted that these were surrendered by the stall owners and the establishments were not
raided.

The other defendant, WPD Superintendent, Narcisco Cabrera, filed no answer.

On January 11, 1984, the trial court issued an Order setting the case for hearing on January 16,
1984 "for the parties to adduce evidence on the question of whether the publication 'Pinoy
Playboy Magazine alleged (sic) seized, confiscated and/or burned by the defendants, are
obscence per se or not".

On February 3, 1984, the trial court promulgated the Order appealed from denying the motion
for a writ of preliminary injunction, and dismissing the case for lack of merit

The CA also dismissed the appeal due to the argument that freedom of the press is not without
restraint.

In the SC, the petitioner claimed that:

1. The CA erred in holding that the police officers could without any court warrant or order seize
and confiscate petitioner's magazines on the basis simply of their determination that they are
obscene.

2. The Court of Appeals erred in affirming the decision of the trial court and, in effect, holding
that the trial court could dismiss the case on its merits without any hearing thereon when what
was submitted to it for resolution was merely the application of petitioner for the writ of
preliminary injunction.

ISSUE: Whether the search and seizure was illegal.

HELD: YES

Test for obscenity: "whether the tendency of the matter charged as obscene, is to deprave or
corrupt those whose minds are open to such immoral influences and into whose hands a
publication or other article charged as being obscene may fall
Also, "whether a picture is obscene or indecent must depend upon the circumstances of the case,
and that ultimately, the question is to be decided by the "judgment of the aggregate sense of the
community reached by it." (Kottinger)

The issue is a complicated one, in which the fine lines have neither been drawn nor divided.

Katigbak- "Whether to the average person, applying contemporary standards, the dominant
theme of the material taken as a whole appeals to prurient interest."

Kalaw-Katigbak represented a marked departure from Kottinger in the sense that it measured
obscenity in terms of the "dominant theme" of the work, rather than isolated passages, which
were central to Kottinger (although both cases are agreed that "contemporary community
standards" are the final arbiters of what is "obscene"). Kalaw-Katigbak undertook moreover to
make the determination of obscenity essentially a judicial question and as a consequence, to
temper the wide discretion Kottinger had given unto law enforcers.

The latest say on American jurisprudence was Miller v. California, which expressly abandoned
Massachusettes, and established "basic guidelines," to wit: "(a) whether 'the average person,
applying contemporary standards' would find the work, taken as a whole, appeals to the prurient
interest . . .; (b) whether the work depicts or describes, in a patently offensive way, sexual
conduct specifically defined by the applicable state law; and (c) whether the work, taken as a
whole, lacks serious literary, artistic, political, or scientific value.

The lack of uniformity in American jurisprudence as to what constitutes "obscenity" has been
attributed to the reluctance of the courts to recognize the constitutional dimension of the
problem.

Apparently, the courts have assumed that "obscenity" is not included in the guaranty of free
speech, an assumption that, as we averred, has allowed a climate of opinions among magistrates
predicated upon arbitrary, if vague theories of what is acceptable to society.

In the case at bar, there is no challenge on the right of the State, in the legitimate exercise of
police power, to suppress smut provided it is smut. For obvious reasons, smut is not smut simply
because one insists it is smut. So is it equally evident that individual tastes develop, adapt to
wide-ranging influences, and keep in step with the rapid advance of civilization. What shocked
our forebears, say, five decades ago, is not necessarily repulsive to the present generation.

But neither should we say that "obscenity" is a bare (no pun intended) matter of opinion. As we
said earlier, it is the divergent perceptions of men and women that have probably compounded
the problem rather than resolved it.

Undoubtedly, "immoral" lore or literature comes within the ambit of free expression, although
not its protection. In free expression cases, this Court has consistently been on the side of the
exercise of the right, barring a "clear and present danger" that would warrant State interference
and action. But the burden to show this lies with the authorities.

"There must be objective and convincing, not subjective or conjectural, proof of the existence of
such clear and present danger."

As we so strongly stressed in Bagatsing, a case involving the delivery of a political speech, the
presumption is that the speech may validly be said. The burden is on the State to demonstrate the
existence of a danger, a danger that must not only be: (1) clear but also, (2) present, to justify
State action to stop the speech.

The Court is not convinced that the private respondents have shown the required proof to justify
a ban and to warrant confiscation of the literature for which mandatory injunction had been
sought below. First of all, they were not possessed of a lawful court order: (1) finding the said
materials to be pornography, and (2) authorizing them to carry out a search and seizure, by way
of a search warrant.

Has petitioner been found guilty for publishing obscene works under Presidential Decrees Nos.
960 and 969? This not answered, one can conclude that the fact that the former respondent
Mayor's act was sanctioned by "police power" is no license to seize property in disregard of due
process. The PDs dont give the authorities the permission to execute high-handed acts.

It is basic that searches and seizures may be done only through a judicial warrant, otherwise, they
become unreasonable and subject to challenge.

There is of course provision for warrantless searches under the Rules of Court but as the
provision itself suggests, the search must have been an incident to a lawful arrest and it must be
on account fo a crime committed.

The Court rejected the argument that "[t]here is no constitutional nor legal provision which
would free the accused of all criminal responsibility because there had been no warrant, and
there is no "accused" here to speak of, who ought to be "punished".

PEOPLE VS. MENDOZA

FACTS: On November 11, 1988, accused-appellant, his wife Cecilia Mendoza, and their then
10-year-old daughter attended the birthday party of a relative of accused-appellant held at
McDonald's in Harrison Plaza. While the party was going on, accused-appellant letf and
proceeded to Kentucky Fried Chicken Restaurant where he had some beer. When it was time for
Cecilia and Charmaine to go home, they could not find accused-appellant, hence, they decided to
just leave.
Cecilia and Charmaine arrived home at around 7 o'clock in the evening but accused-appellant
was not yet there. After a while, mother and daughter left for the house of Cecilia's parents in
Bacoor, Cavite to bring some perfume for Cecilia's brother, Francisco.

At about 9 o'clock in the evening, Cecilia and Charmaine left Bacoor. They rode a jeepney and at
the gate of the subdivision where they live, they saw the car of Rowena Hernandez, Cecilia's
god-daughter, and they hitched a ride home. Finally home, they saw their car already parked in
the garage of their neighbor. All the lights in their house were on but the screen door was locked.
They knocked at the window but accused-appellant did not respond. A moment later, however,
accused-appellant opened the back door and mother and daughter went straight to the master's
bedroom

While inside the master's bedroom, accused-appellant who was drunk instructed Charmaine to
get cold water and to douse him. She willingly obliged, after which she was told to go to her
room. She changed her clothes and readied herself for bed. While in her room, Charmaine heard
her parents quarrelling over the issue of Cecilia and Charmaine having left accused-appellant at
the party. Thereafter, Charmaine suddenly heard three gunshots. Running out of her room,
Charmaine saw her mother Cecilia down on the floor of their living room, bleeding profusely.
Charmaine saw accused-appellant hiding a gun under the bed in her parents' room.

Accused-appellant subsequently called his brother-in-law, Sgt. Antonio Gabac, and told him that
Cecilia had been shot and is already dead. Gabac, on the other line, told accused-appellant not to
touch anything and that he would be arriving shortly. When Gabac finally arrived, he and
accused-appellant carried the lifeless body of Cecilia into accused-appellant's car and brought her
to the Perpetual Help Hospital.

Cecilia's father, Alipio Eusebio, having been informed of his daughter's death, and that
valuables were being taken out of his daughter's house, decided to remove, together with
his sons, the remaining pieces of property therein, including accused-appellant's personal
effects

From the aforestated personal effects of accused-appellant, Alipio found Mission Order No.
86-580-893 dated November 7, 1986 issued to accused-appellant by Col. Eladio Gonzales,
PAF (GSC), Acting Wing Commander, 580th Aircraft Central Warning Wing, Villamor
Airbase, Pasay City, which authorized accused-appellant to carry a Colt Revolver, 38
Caliber with Serial No. 41001 from November 15, 1986 to December 15, 1986. There was
also a Memorandum Receipt for Equipment, dated November 10, 1986, approved by
Captain Luis L. Salanguit of the Philippine Air Force and Lt. Col. Ramon Bandong and
issued to one Octavio L. Mendoza, Captain, PAF, Assistant Director for Personnel which
described the firearm as "One Colt Revolver SN 41001"

ISSUE: Wether the trial court erred in substantially and almost totally relying on illegally
procured and/or inadmissible, unauthenticated, questionable documents, in grave violation of
accused's constitutional right to privacy of communication and papers, and/or his right against
unreasonable search and seizure.

HELD: NO

Charmaine testified thal the fatal gun, when exhibited in court, was the gun she saw on
the night her mother was not shot. And weeks earlier, she said, it was the same gun which she
saw with his father. Defense witness, Antonio Gabac, when asked by the Las Pias police
investigators to surrender the gun, claimed that the same was surrendered to him by accused-
appellant shortly after the shooting incident. The possession of the fatal gun by accused-
appellant is further established by the memorandum receipt signed by accused-appellant
himself and a mission order authorizing him to carry the said weapon. But accused-
appellant claims that these documents were illegally procured in grave violation of his
constitutional right to privacy of communication and papers, and/or his right against
unreasonable search and seizure.

The Solicitor General is correct in explaining that such rights applies as a restraint directed only
against the government and its agencies. The case in piont is People vs. Marti (193 SCRA 57
[1991]) where this Court had the occasion to rule that the constitutional protection against
unreasonable searches and seizures refers to the immunity of one's person from
interference by government and it cannot be extended to acts committed by private
individuals so as to bring it within the ambit of alleged unlawful intrusion.

In the instant case, the memorandum receipt and mission order were discovered by accused-
appellant's father-in-law Alipio Eusebio, a private citizen. Certainly, a search warrant is
dispensable.

Finally, contrary to accused-appellant's claim that he was licensed and authorized to carry a .45
caliber pistol, the certification of Captain Abraham Garcillano, Chief, Records, Legal and
Research Branch of the Firearm and Explosive Unit, dated December 29, 1989, shows that
accused-appellant is not a licensed fiream holder of any kind.

SILAHIS INTERNATIONAL HOTEL and JOSE PANLILIO

vs.

ROGELIO SOLUTA ET AL

FACTS:

Coronel Floro Maniego, General Manager of the Rapier Enforcement Professional Investigation
and Security Agency, Inc. (REPISA) which Silahis International Hotel contracted to provide its
security force to the hotel had been receiving reports that sale and/or use of marijuana, dollar
smuggling, and prostitution were going on in the union office at the hotel and that there existed a
theft syndicate in the hotel. Acting on this tip, Jose Marcel Panlilio, the Vice President for
Finance of his co-petitioner Silahis International Hotel allowed Maniego to place the suspected
members and officers of the union under surveillance.

At dawn of January 11, 1988, Loida Somacera, a laundry woman at the hotel, saw five
unidentified men forcibly trying to open the doors of the union door. Somecera narrated to
Rogelio Soluta (one of the respondents herein) the incident to which Soluta responded
immediately. Soluta thus immediately lodged a complaint before the Security Officer. And he
fetched a locksmith, Efren Guevarra, who tried to assist him, Edna, Arnold Ilustrisimo and Ed
Bautista open the door. At that instant, men in barong tagalog armed with clubs arrived and
started hitting Soluta and his companions, drawing them to run to the female locker room, and to
thereafter proceed to the Engineering Office where they called for police assistance.

While awaiting the arrival of the police, Henry Babay, a union officer, and Panlilio, on the latters
request, met. At the meeting, Panlilio told Babay that they proceed to the union office where they
would settle the mauling incident, to which Babay replied that the door of the office could not be
opened. Panlilio thereupon instructed Villanueva to force open the door, and the latter did. Once
inside, Panlilio and his companions began searching the office, over the objection of Babay who
even asked them if they had a search warrant. A plastic bag was found containing marijuana
flowering tops.

Since marijuana was found in the room, a complaint was filed againt Soluta, Babay and other
union officers was filed before the Fiscals Office of Manila, for violation of The Dangerous
Drugs Act.

The RTC of Manila acquitted the petitioners due to the fact that the specimen and/or the
marijuana flowering tops allegedly found inside the Union Office occupied by the accused are
not admissible in evidence and were confiscated under suspicious circumstance.

Soluta and his fellow union officers, together with the union, thereafter filed before the Manila
RTC a Complaint against Silahis and Palilio including prosecuting Fiscal Jose Bautista and Atty.
Eduardo Tutaan who assisted in the prosecution of the case against them, for malicious
prosecution and violation of their constitutional right against illegal search.
The RTC of Manila ruled in favor of Soluta et al and ordered Silahis et al. to pay damages to
Soluta and company. On appeal, the Court of Appeals affirmed the decision of the RTC but
lowered the amount to be awarded to the Soluta et al.

ISSUES:

1. WON PETITIONERS SEARCH OF THE UNION OFFICE IN THE INSTANT CASE WAS
ENTIRELY REASONABLE UNDER THE CIRCUMSTANCES;

2. WON THE COURT OF APPEALS GRAVELY ERRED IN ITS CONCLUSION THAT


PETITIONERS ARE LIABLE FOR DAMAGES UNDER ARTICLE 32 OF THE CIVIL CODE

HELD:

1. NO. Petitioners argue that being private persons, they are not covered by the standards set
forth in People vs. Aruta as the constitutional protection against illegal searches and seizures is
not meant to be invoked against private individuals. Petitioners further argue that the search of
the union office was reasonable under the circumstances, given that the hotel owns the room
where the union holds office; the search was not without probable cause as it was conducted
precisely due to reports received by petitioners that the union office was being used as a venue
for illegal activities, particularly the sale and/or use of prohibited drugs; and the search was
conducted with the consent and in the presence of union officer Babay. The petition fails.

Petitioners had, by their own claim, already received reports in late 1987 of illegal activities
allegedly undertaken in the union office and Maniego conducted surveillance of the union
officers. Yet, in the morning of January 11, 1988, petitioners and their companions barged into
and searched the union office without a search warrant, despite ample time for them to obtain
one, and notwithstanding the objection of Babay.

The course taken by petitioners and company stinks in illegality, it not falling under any of the
exceptional instances when a warrantless search is allowed by law. Petitioners violation of
individual respondents constitutional right against unreasonable search thus furnishes the basis
for the award of damages under Article 32 of the Civil Code.
2. NO. Article 32 of the New Civil Code provides that any public officer or employee, or
any private individual, who directly or indirectly obstructs, defeats, violates or in any manner
impedes or impairs the right to be secure in ones person, house, papers, and effects against
unreasonable searches and seizures shall be liable to the latter for damages. It necessary to
hold not only public officers but also private individuals civilly liable for violation of rights
enumerated in Article 32 of the Civil Code. That is why it is not even necessary that the
defendant under this Article should have acted with malice or bad faith, otherwise, it would
defeat its main purpose, which is the effective protection of individual rights. It suffices that
there is a violation of the constitutional right of the plaintiff.

PEOPLE OF THE PHILIPPINES vs. ANDRE MARTI

FACTS:

On August 14, 1987, the appellant and his common-law wife, Shirley Reyes, went to the booth
of the "Manila Packing and Export Forwarders" in the Pistang Pilipino Complex, Ermita, Manila,
carrying with them four (4) gift wrapped packages. Anita Reyes (the proprietress and no relation
to Shirley Reyes) attended to them. The appellant informed Anita Reyes that he was sending the
packages to a friend in Zurich, Switzerland. Anita Reyes then asked the appellant if she could
examine and inspect the packages. Appellant, however, refused, assuring her that the packages
simply contained books, cigars, and gloves and were gifts to his friend in Zurich. In view of
appellant's representation, Anita Reyes no longer insisted on inspecting the packages.

Before delivery of appellant's box to the Bureau of Customs and/or Bureau of Posts, Mr. Job
Reyes (proprietor) and husband of Anita (Reyes), following standard operating procedure,
opened the boxes for final inspection. When he opened appellant's box, a peculiar odor emitted
therefrom. His curiousity aroused, he squeezed one of the bundles allegedly containing gloves
and felt dried leaves inside. Opening one of the bundles, he pulled out a cellophane wrapper
protruding from the opening of one of the gloves. He made an opening on one of the cellophane
wrappers and took several grams of the contents thereof

Job Reyes then prepared a letter reporting the shipment to the NBI and requesting a laboratory
examination of the samples he extracted from the cellophane wrapper. He brought the letter and
a sample of the shipment to the Narcotics Section of the NBI. He was interviewed by the Chief
of Narcotics Section. Job Reyes informed the NBI that the rest of the shipment was still in his
office. Therefore, Job Reyes and three (3) NBI agents, and a photographer, went to the Reyes'
office at Ermita, Manila.
Job Reyes brought out the box in which appellant's packages were placed and, in the presence of
the NBI agents, opened the top flaps, removed the styro-foam and took out the cellophane
wrappers from inside the gloves. Dried marijuana leaves were found to have been contained
inside the cellophane wrappers. The package which allegedly contained books was likewise
opened by Job Reyes. He discovered that the package contained bricks or cake-like dried
marijuana leaves. The package which allegedly contained tabacalera cigars was also opened. It
turned out that dried marijuana leaves were neatly stocked underneath the cigars.

The NBI agents made an inventory and took charge of the box and of the contents thereof, after
signing a "Receipt" acknowledging custody of the said effects

A complaint for violation of The Dangerous Drugs Act was filed against Marti. Marti contended
that he was not the owner of the packages but rather a certain Michael, a German national, whom
appellant met in a pub along Ermita, Manila: that in the course of their 30-minute conversation,
Michael requested him to ship the packages and gave him P2,000.00 for the cost of the shipment
since the German national was about to leave the country the next day.

ISSUE: WON THE LOWER COURT ERRED IN ADMITTING IN EVIDENCE THE


ILLEGALLY SEARCHED AND SEIZED OBJECTS CONTAINED IN THE FOUR PARCELS

HELD: NO. The Court sees no cogent reason why the same should not be admitted against him
in the prosecution of the offense charged. The contraband in the case at bar having come into
possession of the Government without the latter transgressing appellant's rights against
unreasonable search and seizure.

First, the factual considerations of the case at bar readily foreclose the proposition that NBI
agents conducted an illegal search and seizure of the prohibited merchandise. Records of the case
clearly indicate that it was Mr. Job Reyes, the proprietor of the forwarding agency, who made
search/inspection of the packages. Said inspection was reasonable and a standard operating
procedure on the part of Mr. Reyes as a precautionary measure before delivery of packages to the
Bureau of Customs or the Bureau of Posts.

It will be recalled that after Reyes opened the box containing the illicit cargo, he took samples of
the same to the NBI and later summoned the agents to his place of business. Thereafter, he
opened the parcel containing the rest of the shipment and entrusted the care and custody thereof
to the NBI agents. Clearly, the NBI agents made no search and seizure, much less an illegal one,
contrary to the postulate of accused/appellant.

Second, the mere presence of the NBI agents did not convert the reasonable search effected by
Reyes into a warrantless search and seizure proscribed by the Constitution. Merely to observe
and look at that which is in plain sight is not a search. Having observed that which is open,
where no trespass has been committed in aid thereof, is not search. Where the contraband articles
are identified without a trespass on the part of the arresting officer, there is not the search that is
prohibited by the constitution.

WHEREFORE, the judgment of conviction finding appellant guilty beyond reasonable doubt of
the crime charged is hereby AFFIRMED.

VALID WARRANTLESS SEARCH AND SEIZURE

3. Search made incidental to a valid arrest

Search incident to a lawful arrest, commonly known as search incident to arrest (SITA) or
the Chimel rule, is a legal principle that allows police to perform a warrantless search of an
arrested person, and the area within the arrestees immediate control, in the interest of officer
safety, the prevention of escape, and the destruction of evidence.

3. PEOPLE OF THE PHILIPPINES vs SPO3 SANGKI AR, MIKE TALIB, and JORDAN
MUSA
Facts:
On December 20, 2002, a confidential informant (CI) came to the Heinous Crime
Investigation Section (HCIS) of the Davao City Police Department and reported that three (3)
suspected drug pushers had contacted him for a deal involving six (6) plastic sachets of shabu.
He was instructed to go that same morning to St. Peters College at Toril, Davao City and look for
an orange Nissan Sentra car.
Police Chief Inspector Fulgencio Pavo, Sr. immediately formed a buy-bust team who
would act as poseur-buyer.
The team proceeded to the school where PO1 Ayao and the CI waited by the gate. At
around 8:45 a.m., an orange Nissan Sentra bearing plate number UGR 510 stopped in front of
them. The two men approached the vehicle and the CI talked briefly with SPO3 Ara, THe
poseur-buyer said he had the money and Ara took out several sachets with crystalline granules
from his pocket and handed them to PO1 Ayao (poseur-buyer), who thereupon gave the pre-
arranged signal to arrest the accused.
Recovered from the group were plastic sachets of white crystalline substance: six (6) big
sachets, from Ara by PO1 Ayao; five (5) big sachets, from Musa by SPO1 Furog; and a small
sachet, from Talib by PO2 Lao.
The three suspects were brought to the HCIS and the seized items indorsed to the
Philippine National Police (PNP) Crime Laboratory for examination. Forensic Chemist Austero,
who conducted the examination, found that the confiscated sachets all tested positive for shabu.

Ara insisted that he was not holding anything and that the shabu taken from him was planted. He
asserted that the only time he saw shabu was on television.
RTC pronounced accused-appellants guilty of the crimes charged.
ISSUES:

1. Whether the buy-bust conducted was valid;

2. Whether the chain of custody over the shabu was unbroken.

HELD:
Warrantless Arrest and Seizure Valid
Owing to the special circumstances surrounding the drug trade, a buy-bust operation has
long been held as a legitimate method of catching offenders. It is a form of entrapment employed
as an effective way of apprehending a criminal in the act of commission of an offense. We have
ruled that a buy-bust operation can be carried out after a long period of planning. The period of
planning for such operation cannot be dictated to the police authorities who are to undertake such
operation.It is unavailing then to argue that the operatives had to first secure a warrant of arrest
given that the objective of the operation was to apprehend the accused-appellants in flagrante
delicto. In fact, one of the situations covered by a lawful warrantless arrest under Section 5(a),
Rule 113 of the Rules of Court is when a person has committed, is actually committing, or is
attempting to commit an offense in the presence of a peace officer or private person.
It is erroneous as well to argue that there was no probable cause to arrest accused-
appellants. Probable cause, in warrantless searches, must only be based on reasonable ground of
suspicion or belief that a crime has been committed or is about to be committed. Probable cause
was provided by information gathered from the CI and from accused-appellants themselves when
they instructed PO1 Ayao to enter their vehicle and begin the transaction. The illegal sale of
shabu inside accused-s vehicle was afterwards clearly established. Thus, as we have previously
held, the arresting officers were justified in making the arrests as accused had just committed a
crime when Ara sold shabu to PO1 Ayao.Talib and Musa were also frisked for contraband as it
may be logically inferred that they were also part of Aras drug activities inside the vehicle. This
inference was further strengthened by Musas attempt to drive the vehicle away and elude arrest.
Since the buy-bust operation was shown to be a legitimate form of entrapment. The
pieces of evidence thus seized therein were admissible. It was within legal bounds and no
anomaly was found in the conduct of the buy-bust operation.

Validity of Buy-Bust Operation


There are requirements that must be complied with in proving the legitimacy of drug buy-
bust operations. Nevertheless, this Court has ruled that presentation of the marked money used is
not such a requirement. In the prosecution for the sale of dangerous drugs, the absence of marked
money does not create a hiatus in the evidence for the prosecution, as long as the sale of
dangerous drugs is adequately proved and the drug subject of the transaction is presented before
the court. In the instant case, the police officers testimonies adequately established the illegal
sale of shabu. The shabu was then presented before the trial court. The non-presentation of the
marked money may, thus, be overlooked as a peripheral matter.

Requirements of RA 9165 on Proper Inventory


Musa contends that since the markings on the seized items were only made at the police
station, there is a great possibility that these were replaced. The result, he argues, would be a lack
of guarantee that what were inventoried and photographed at the crime laboratory were the same
specimens confiscated from the accused.
The Implementing Rules and Regulations (IRR) do not require strict compliance as to the
chain of custody rule. The arrest of an accused will not be invalidated and the items seized from
him rendered inadmissible on the sole ground of non-compliance with Sec. 21, Article II of RA
9165. We have emphasized that what is essential is the preservation of the integrity and the
evidentiary value of the seized items, as the same would be utilized in the determination of the
guilt or innocence of the accused.

ACCUSED ARE GUILTY.

3. PEOPLE VS PENAFLORIDA

April 10, 2008

FACTS:

The chief of investigation and operation of the PNP station in Tiagon, Camarines sur, SPO3
Vicente Competente received a tip from an asset that a bundle of marijuana is being transported
from one barangay in Tiagnon to Huyon-huyon. Then mayor Domingo Agravente organized a
group of police who, upon his orders, proceeded to Huyon-huyon in order to arrest Penaflorida.
On their way to Huyon-huyon, the police over took Penafloridas bycle, flagged him down and
found marijuana wrapped in a cellophane and a newspaper together with some grocery
items. Police officers confiscated the marijuana and broungth Penaflorida to their headquarters
and was charged of violation of section 4 article II of RA 6425 otherwise known as Dangerous
Drugs act of 1972.

In assailing his conviction, Penaflorida said that the package containing marijuana was only
given to him by Boyet Obias who requested him to bring the same to Jimmy Gonzales.
Penaflorida further assails that there is no evidence to prove that he freely and consciously
possessed the marijuana and that the police the police should have first investigated the matter
and tried to obtain the warrant, intead of arbitrarily detaining him.

ISSUE:

WON Penafloridas arrest was illegal.

HELD:

No. The police received the tip around 1:00 pm that Penaflorida was transporting marijuana
to Huyon-huyon. Certainly they had no time to secure a warrant as Penaflorida was already in
transit and was already committing a crime. Penaflorida was seen riding a bicycle carrying with
him the package of marijuana which demonstrates that he was already committing a crime.
Under said circumstances, the police had probable cause to believe that Penaflorida was
committing a crime. Thus the warrantless arrest was justified. As to the defense of Penaflorida
of not knowing the contents of the package, the trial court held that his defense was impossible
because marijuana has a distinct, sweet and unmistakable aroma which could have alarmed him.
Lastly, violation of RA 6425 constitutes a crime mala prohibita , wherein the mere possession of
a prohibited drug without legal authority is punishable. Intent, motive or knowledge thereof is
not necessary.

3. MORENO VS AGO CHI

12 Phil 439

FACTS:

In May 1904, a complaint was presented against the Ago Chi charging him with the
crime of assassination. He was arrested and tried for said crime in the Court of First Instance in
the city of Manila. The Roberto Moreno was appointed by the court to defend the Ago Chi
during the progress of the trial. Ago Chi was found guilty of the crime charged and was
sentenced with the penalty of death. Ago Chi appealed to the Supreme Court. On April 1906.
The court modified the finding of the lower court and sentenced the Ago Chi to be imprisoned
for a period of twenty years of reclusion temporal.

When Ago Chi was arrested, the police found P700 in his possession which was then
surrendered to the clerk of court. Moreno then petitioned the court to turn over to him the amount
of P600 from the money which had been taken from the defendant as reasonable compensation
for his services. In December 1906, Moreno was awarded with P550 compensation. He then filed
a petition to the clerk of court to recover said amount.

Ago Chi was brought into court on the day fixed for the hearing and then and there stated
to the court that he was willing that a portion of the money originally taken from him by the
officer making the arrest might be applied upon the execution, but that he wished some of it
reserved for his own use.

After hearing the respective parties, the judge of the lower court ordered that the clerk pay to
Moreno, out of said money, the sum of P50 and further ordered that the balance should be
retained by the clerk for the benefit of the Ago Chi.

Moreno Appealed said decision, assailing that court erred in arbitrarily fixing the value of the
services rendered by him in the trial of said cause at the sum of P50, and in not ordering the clerk
to pay to him the sum of P550.

ISSUE:

May third persons, creditors for example of the defendant, obtain a lien or claim upon
such property taken from the defendant upon arrest?

HELD:

No. third persons, creditors, etc. cannot acquire any claim or lien upon such property
while thus in the hands of the officer, which they might not acquire had the property remained in
the custody of the defendant himself. To hold otherwise would lead to unlawful and forcible
searches of the person under cover of criminal of debts. Furthermore, to deprive the defendant of
his money or property under other circumstances than those mentioned above is to deprive him,
perhaps, of the lawful means of defense.When it is fully shown that the property so taken was in
no way connected with the crime charged, the court should not permit any advantage to be taken
of the defendant, by reason of the fact that he had been deprived of his property by the officer
and against his will.

3. G.R. No. 109232 December 29, 1995


PEOPLE OF THE PHILIPPINES, plaintiff-appellee, vs.ANG CHUN KIT also known as
"ROMY ANG," accused-appellant.

FACTS:

ANG CHUN KIT:

A Chinese national and a member of a HK-based drug syndicate operating in Metro


Manila

was collared by NARCOM operatives in a buy-bust operation after he sold to an


undercover agent for P400,000.00 a kilo of methamphetamine hydrochloride known as
shabu

his car: contained drugs neatly tucked in a Kleenex box.

8 November 1991(3pm): Confidential Informant (CI) reported to Chief Investigator Avelino I.


Razon that he (CI) had arranged a transaction with a drug dealer interested in selling a kilo of
shabu for P400,000.00 and agreed to do the sale at 7pm at the lobby of the Cardinal Santos
Medical Center.

Chief Investigator Razon

organized a buy-bust team composed of Chief Inspector Rolando Magno as team leader,
SPO3 Lolita Bugarin, SPO2 Cesar Jacobo as poseur-buyer, SPO2 Albert San Jose, and SPO2
Domingo Rubi.

40) bundles of genuine and counterfeit P100-bills were prepared with each bundle
supposed to contain P10,000.00.

To camouflage the counterfeit bills genuine P100-bills were placed on the top and bottom
of ten (10) bundles.

The CI and SPO2 Jacobo ( carrying the money): went to the lobby of CSMC while others moved
around to avoid detection.

7:15pm: Ang Chun Kit arrived in a Toyota Corolla.

Meeting:

He was met at the lobby by the CI who introduced SPO2 Jacobo to him as the person
interested to buy shabu.

After quick look at the money, SPO2 Jacobo and the CI followed him to the parking lot
where the latter took out from the trunk of his car a blue SM Shoemart plastic bag and
handed it to SPO2 Jacobo.
After ascertaining that the bag contained approximately one (1) kilo of shabu, SPO2
Jacobo handed the boodle money to the accused.

Signal( arrest):

SPO2 Jacobo casually lit a cigarette to signal to the other NARCOM operatives to move
in and effect the arrest.

The other members of the team closed in, placed the accused under arrest and seized the
money from him. They also searched his vehicle and found on the dashboard of his car
three (3) packets more of crystalline substance in a Kleenex box.

Substance found in SM Shoemart bag and in the Kleenex box to be shabu and weighing 983.27
grams and 293.70 grams

Contention of the Accused:

He denied the charges. He contended that a friend named Johnny Sy called him asking
him if he could ride with him to Greenhills to visit a sick friend at the CSMC and that the car
was owned by his cousin, Roman Ong. When he went to the parking lot, (20) to (30) minutes
later, plainclothesmen with guns drawn, Johnny and Anthony in tow, suddenly appeared from
nowhere and arrested him and Uy without informing them the reason for their arrest. He together
with Uy, Sy and Co was then brought to Camp Crame where he was mauled, detained and
interrogated without the assistance of counsel. His repeated requests to make a telephone call to
his relatives and counsel were denied.

He alleges that the buy bust operation was a frame up and the evidence was merely
planted.

Search and Seizure:

The defense argues : shabu found inside the car is inadmissible in evidence as it was
obtained through an illegal search and seizure, the same having been found inside the car and
not in the person of the accused who was outside the car.

But the search inside the car was an incident of a lawful arrest. It must be remembered that the
accused was with a driver who was inside the car. Upon the arrest of the accused, the arresting
agents also had to neutralize the driver inside the car who could be presumed at that instance to
be acting together and in conspiracy with the accused. For a weapon could have easily been
concealed in the dashboard of the vehicle, which was very well within the reach of the driver at
that time. Corollarily, in People v. Figueroa we reiterated that "[t]he warrantless search and
seizure, as an incident to a suspect's lawful arrest, may extend beyond the person of the one
arrested to include the premises or surroundings under his immediate control." 21 Thus whether
the accused gave his consent to the search of the car which the arresting agents say he did, but
which he denies, is immaterial.

HELD: we find it difficult to believe the version of the accused. He did not even present Johnny
Sy or Anthony Co to substantiate his story, much less did he reveal the name of the patient they
were to visit in the hospital. Besides it appears that there was no reason for the accused to wait
for Johnny Sy and Anthony Co in the parking lot as they did not have any prior agreement to
meet there. On the contrary the accused still had to attend a dinner somewhere and should not
have waited any longer.

WHEREFORE, the Decision of the trial court finding accused appellant Ang Chun Kit also
known as "Romy Ang" guilty beyond reasonable doubt of selling methamphetamine
hydrochloride in violation of Sec. 15, Art. III, R.A. 6425, as amended, sentencing him to life
imprisonment and ordering him to pay a fine of P30,000.00 is AFFIRMED. Costs against
accused appellant.

3. G.R. No. 34917 September 7, 1931

THE PEOPLE OF THE PHILIPPINE ISLANDS, plaintiff-appellee, vs.

LUA CHU and UY SE TIENG, defendants-appellants.

The defendants Lua Chu and Uy Se Tieng appeal from the judgment of the Court of First
Instance of Cebu convicting them of the illegal importation of opium

FACTS:

November 1929: accused Uy Se Tieng wrote to his correspondent in HK to send him a shipment
of opium.

November 4, 1929: Juan Samson (chief of the customs secret service of Cebu) called Joaquin
Natividad(collector of customs for the Port of Cebu) at his office, took 300 pesos from his
drawer and handed it, saying: "This is for you, and a shipment will arrive shortly, and you will
soon be able to recoup your travelling expenses. It was later found out that the shipment
consisted of opium.

Uy Se Tieng : that same night went to Samson's house and told him he had come by order of
Natividad to talk to him about the opium. He informed Samson that the opium shipment
consisted of 3,000 tins, and that will pay Natividad P6,000 or a P2 a tin, and that the opium had
been in Hongkong since the beginning of October awaiting a ship that would go direct to Cebu.

December 14, 1929: The Kolambugan steamship arrived at Cebu. Samson detailed one of his
men to watch the ship. While Natividad, instructed him to do everything possible to have the
cargo unloaded, and to require Uy Se Tieng to pay over the P6,000.
Uy Se Tieng went to Samson's house that night and was told that he must pay over the P6,000
before taking the opium out of the customhouse.

The following day Samson informed Colonel Francisco of the Constabulary, of all that had taken
place, and the said colonel instructed the provincial commander, Captain Buenconsejo, to discuss
the capture of the opium owners with Samson.

Buenconsejo and Samson agreed to meet at the latter's house that same night.

Samson went to provincial fiscal and asked for a stenographer to take down the conversation he
would have with Uy Se Tieng in the presence of Captain Buenconsejo.

December 17, 1929: Captain Buenconsejo, Lieutenant Fernando; and the stenographer went to
Samson's house and concealed themselves behind a curtain made of strips of wood which hung
from the window overlooking the entrance to the house on the ground floor.

Uy Se Tieng: arrived and asked Samson if he had brought the money. He replied that he had not,
saying that the owner of the opium, who was Lua Chu, was afraid of him. Samson then told him
to tell Lua Chu not to be afraid, and that he might come to Samson's house.

After pointing out to Uy Se Tieng a back door entrance into the garden, he asked him where the
opium was, and Uy Se Tieng answered that it was in the cases numbered 11 to 18, and that there
were 3,252 tins.

Uy Se Tieng returned accompanied by his codefendant Lua Chu, who said he was not the sole
owner of the opium, but that a man from Manila, named Tan, and another in Amoy were also
owners.

Samson then asked Lua Chu when he was going to get the opium, and the latter answered that
Uy Se Tieng would take charge of that. On being asked if he had brought the P6,000, Lua Chu
answered, no, but promised to deliver it when the opium was in Uy Se Tieng's warehouse.

Uy Se Tieng and one Uy Ay arrived at Samson's house, and as Uy Se Tieng was handing certain
papers, Uy Ay, Captain Buenconsejo, who had been hiding, appeared and arrested the two
Chinamen, taking the aforementioned papers, which consisted of bills of lading and in invoice
written in Chinese characters, and relating to the articles.

After having taken Uy Se Tieng and Uy Ay to the Constabulary headquarters, and notified the
fiscal, Captain Buenconsejo and Samson went to Lua Chu's home to search it and arrest him.
Captain Buenconsejo and Samson also took Lua Chu to the Constabulary headquarters, and then
went to the customhouse to examine the cases marked "U.L.H." In the cases marked Nos. 11 to
18, they found 3,252 opium tins hidden away in a quantity to dry fish.

The value of the opium confiscated amounted to P50,000.


Contention of the Accused:

The defense attempted to show that after Juan Samson had obtained a loan of P200 from Uy Se
Tieng, he induced him to order the opium from Hongkong saying that it only cost from P2 to P3
a tin there, while in Cebu it cost from P18 to P20, and that he could make a good deal of money
by bringing in a shipment of that drug; that Samson told Uy Se Tieng, that there would be no
danger, because he and the collector of customs would protect him;

But the defendants' principal defense is that they were induced by Juan Samson to import the
opium in question. Juan Samson denies this, and his conduct in connection with the introduction
of the prohibited drug into the port of Cebu, bears him out.

A public official who induces a person to commit a crime for purposes of gain, does not
take the steps necessary to seize the instruments of the crime and to arrest the offender, before
having obtained the profit he had in mind. It is true that Juan Samson smoothed the way for the
introduction of the prohibited drug, but that was after the accused had already planned its
importation and ordered said drug, leaving only its introduction into the country through the
Cebu customhouse to be managed, and he did not do so to help them carry their plan to a
successful issue, but rather to assure the seizure of the imported drug and the arrest of the
smugglers.

The doctrines referring to the entrapment of offenders and instigation to commit crime, as laid
down by the courts of the United States, are summarized in 16 Corpus Juris, page 88, section 57,
as follows:

ENTRAPMENT AND INSTIGATION. While it has been said that the practice of entrapping
persons into crime for the purpose of instituting criminal prosecutions is to be deplored, and
while instigation, as distinguished from mere entrapment, has often been condemned and has
sometimes been held to prevent the act from being criminal or punishable, the general rule is that
it is no defense to the perpetrator of a crime that facilitates for its commission were purposely
placed in his way, or that the criminal act was done at the "decoy solicitation" of persons seeking
to expose the criminal, or that detectives feigning complicity in the act were present and
apparently assisting in its commission. Especially is this true in that class of cases where the
offense is one of a kind habitually committed, and the solicitation merely furnishes evidence of a
course of conduct. Mere deception by the detective will not shield defendant, if the offense was
committed by him free from the influence or the instigation of the detective. The fact that an
agent of an owner acts as supposed confederate of a thief is no defense to the latter in a
prosecution for larceny, provided the original design was formed independently of such agent;
and where a person approached by the thief as his confederate notifies the owner or the public
authorities, and, being authorized by them to do so, assists the thief in carrying out the plan, the
larceny is nevertheless committed. It is generally held that it is no defense to a prosecution for an
illegal sale of liquor that the purchase was made by a "spotter," detective, or hired informer; but
there are cases holding the contrary.

Held:

Juan Samson neither induced nor instigated the herein defendants-appellants to import the opium
in question, as the latter contend, but pretended to have an understanding with the collector of
customs, Joaquin Natividad who had promised them that he would remove all the difficulties
in the way of their enterprise so far as the customhouse was concerned not to gain the P2,000
intended for him out of the transaction, but in order the better to assure the seizure of the
prohibited drug and the arrest of the surreptitious importers. There is certainly nothing immoral
in this or against the public good which should prevent the Government from prosecuting and
punishing the culprits, for this is not a case where an innocent person is induced to commit a
crime merely to prosecute him, but it simply a trap set to catch a criminal.

Wherefore, we are of opinion and so hold, that the mere fact that the chief of the customs secret
service pretended to agree a plan for smuggling illegally imported opium through the
customhouse, in order the better to assure the seizure of said opium and the arrest of its
importers, is no bar to the prosecution and conviction of the latter.

3. PEOPLE VS FIGUEROA 248 SCRA 679


FACTS:
Robert Figueroa was convicted of the RTC of Paranaque City for violation of Section 14-A [2]
of RA 6425 or the Dangerous Drugs Act of 1972. His co-accused Beatrice Valerio was acquitted.
Accordingly, the two willfully, unlawfully and feloniously manufacture, produce, prepare or
process methamphetamine hydrochloride or shabu, a regulated drug amounting to a 2.4 liters,
directly by means of chemical synthesis.
Before the arrest of Figueroa and Valerio, Palencia, an NBI Special Investigator received a call
from their informant who reported that a certain OBET or Figueroa was allegedly engaged in
large scale drug trafficking in Makati City. Palencia and Soriano, an NBI Intellegence Agent,
then instructed the informant for a buy bust operation. Palencia and Soriano, through the buy
bust operation, succeeded in apprehending Obet. (Although before he was apprehended, he
managed to take his mistress and her two children as hostages after firing two shots at Palencia
and Soriano. He surrendered to one Major Roberto Reyes, thereafter.)
During his interrogation at the NBI headquarters, he volunteered that the source of his shabu was
a certain Betty (Valerio) of 263 El Grande Street, B.F. Homes, Paranaque City. Palencia and
Soriano took him to the said address for a follow-up operation. Betty insistently denied the
existence of the shabu, PALENCIA told OBET to confer with Betty. After a while, OBET
proceeded to the kitchen of the guesthouse located outside the main house, followed by Betty.
OBET then promptly pointed to what he termed as liquid shabu inside a white pail along with
other drug paraphernalia, such as a beaker spray. PALENCIA and SORIANO seized the items.
During the trial and on cross examination, PALENCIA admitted that he and SORIANO
conducted the search without a search warrant, but with the consent of Betty. He also admitted
that he did not actually see OBET or Betty in the act of manufacturing shabu
NBI Intelligence Agent II SORIANO corroborated PALENCIA's testimony. He likewise
admitted that the custodial investigation of OBET, during which he divulged Betty as the source
of shabu, was conducted in the absence of any counsel. SORIANO also confirmed PALENCIA's
testimony that they were not armed with a search warrant, but that they conducted the follow-up
operation at Betty's house under the hot pursuit theory.
The trial court agreed with the prosecution's theory that the warrantless arrests of OBET and
Betty were conducted within the purview of valid warrantless arrests enumerated in Section
5,[24] Rule 113 of the Rules of Court. It then ruled as valid the consented warrantless search
conducted at the house of Betty. Consequently, it found that the very items seized by the NBI
agents at the kitchen of Betty's guesthouse were admissible as the corpus delicti of the violation
of Section 14-A of the Dangerous Drugs Act. Thus, the trial court "believed" that the
paraphernalia seized were indispensable to the processing or manufacturing of shabu into
crystallized form. Although it conceded that the prosecution witnesses did not actually see the
crystallization processes, the trial court observed that the Dangerous Drug Act does not require
that there be actual manufacturing activities at the time of the seizure.
The trial court, however, acquitted Betty for failure of the prosecution to adduce evidence that
she, in conspiracy with OBET, manufactured shabu without the requisite authority.
Unsatisfied with the verdict, OBET appealed the decision to the SC on the ground that the State
failed to show by convincing evidence that shortly prior to or during custodial investigation, he
was apprised of his constitutional rights to remain silent, to have a competent and independent
counsel preferably of his own choice, and to be informed of such rights. He asserts that he did
not waive those rights.
HELD:
Needless to state, OBET cannot be investigated for anything in relation to shabu while under
custody without informing him of his rights to remain silent and to have a competent and
independent counsel preferably of his own choice. Any waiver of such rights should be in
writing and made in the presence of a counsel pursuant to Section 12 (1)[26], Article III of the
Constitution. It has been held that these rights attach from the moment the investigation starts,
i.e. when the investigating officers begin to ask questions to elicit information and confessions or
admissions from the suspect.
It is always incumbent upon the prosecution to prove at the trial that prior to in-custody
questioning, the confessant was informed of his constitutional rights. The presumption of
regularity of official acts does not prevail over the constitutional presumption of innocence.
Hence, in the absence of proof that the arresting officers complied with these constitutional
safeguards, extrajudicial statements, whether inculpatory or exculpatory, made during custodial
investigation are inadmissible and cannot be considered in the adjudication of a case. In other
words, confessions and admissions in violation of Section 12 (1), Article III of the Constitution
are inadmissible in evidence against the declarant and more so against third persons. This is so
even if such statements are gospel truth and voluntarily given. Such statements are useless except
as evidence against the very police authorities who violated the suspect's rights.
SORIANO admitted that the custodial investigation of OBET was conducted without the
presence of a lawyer, and there is no proof that OBET waived said right and the right to remain
silent. No waiver in writing and in the presence of a counsel was presented. Thus, pursuant to
paragraph 3 of Section 12 of Article III of the Constitution any admission obtained from OBET
in the course of his custodial investigation was inadmissible against him and cannot be used as a
justification for the search without a warrant.
The search conducted on Betty's house was allegedly consented to by Betty. Indeed, a consented
search is one of the exceptions to the requirement of a search warrant. In case of consented
searches or waiver of the constitutional guarantee, against obtrusive searches, it is fundamental
that to constitute, a waiver, it must first appear that (1) the right exists; (2) that the person
involved had knowledge, either actual or constructive, of the existence of such right; and (3) the
said person had an actual intention to relinquish the right. The third condition does not exist in
the instant case. The fact is, Betty asked for a search warrant.
Neither can the search be appreciated as a search incidental to a valid warrantless arrest of either
Betty or OBET as intimated by the trial court. First, Betty's arrest did not precede the search.
Second, per the prosecution's evidence OBET was not arrested for possession or sale of regulated
or prohibited drugs as a consequence of the buy-bust operation. He surrendered after taking
hostage Estrella and her two children, although he was thereafter held in custody for further
questioning on illegal drugs.
There is no showing that the house occupied by Betty and the articles confiscated therefrom
belong to OBET. That OBET pointed to PALENCIA and SORIANO the places where the
articles were found provides no sufficient basis for a conclusion that they belonged to him. Even
if the articles thus seized actually belonged to him, they cannot be constitutionally and legally
used against him to establish his criminal liability therefor, since the seizure was the fruit of an
invalid custodial investigation. Wherefore, Obet is acquitted of the crime charged.

3. NOLASCO VS. PANO, 139 SCRA 541

Facts:

Milagros Aguilar-Roque was arrested together with Cynthia Nolasco by the Constabulary
Security Group (CSG). Milagros had been wanted as a high ranking officer of the CPP. The
arrest took place at 11:30 a.m. of August 6, 1984. At noon of the same day, her premises were
searched and 428 documents, a portable typewriter and 2 boxes were seized.
Earlier that day, Judge Cruz Pao issued a search warrant to be served at Aguilar-Roques leased
residence allegedly an underground house of the CPP/NPA. On the basis of the documents
seized, charges of subversion and rebellion by the CSG were filed by but the fiscals office
merely charged her and Nolasco with illegal possession of subversive materials.

Aguilar-Roque asked for suppression of the evidence on the ground that it was illegally obtained
and that the search warrant is void because it is a general warrant since it does not sufficiently
describe with particularity the things subject of the search and seizure, and that probable cause
has not been properly established for lack of searching questions propounded to the applicants
witness.

Issue:

Whether the search warrant was valid?

Held:

NO. Section 3, Article IV of the Constitution, guarantees the right of the people to be secure in
their persons, houses, papers and effects against unreasonable searches and seizures of whatever
nature and for any purpose. It also specifically provides that no Search Warrant shall issue except
upon probable cause to be determined by the Judge or such other responsible officer as may be
authorized by law, after examination under oath or affirmation of the complainant and the
witnesses he may produce, and particularly describing the place to be searched and the things to
be seized.

It is at once evident that the foregoing Search Warrant authorizes the seizure of personal
properties vaguely described and not particularized. It is an all- embracing description which
includes everything conceivable regarding the Communist Party of the Philippines and the
National Democratic Front. It does not specify what the subversive books and instructions are;
what the manuals not otherwise available to the public contain to make them subversive or to
enable them to be used for the crime of rebellion. There is absent a definite guideline to the
searching team as to what items might be lawfully seized thus giving the officers of the law
discretion regarding what articles they should seize as, in fact, taken also were a portable
typewriter and 2 wooden boxes.
It is thus in the nature of a general warrant and infringes on the constitutional mandate requiring
particular description of the things to be seized. In the recent rulings of this Court, search
warrants of similar description were considered null and void for being too general.

Search of moving vehicles

PEOPLE OF THE PHILIPPINES vs BELEN MARIACOS

GR NO. 188611 June 16 2010

FACTS:

October 27, 2005 in Brgy Balbalayang, PO2 Pallayoc met with secret agent of the Barangay
Intelligence Network who informed him that a baggage of marijuana had been loaded in a
passenger jeepney that was about to leave for the poblacion. The agent mentioned 3 bags and 1
plastic bag. Further, the agent described a backpack bag with O.K. marking. PO2 Pallayoc
boarded the said jeepney and positioned himself on top thereof. He found bricks of marijuana
wrapped in newspapers. He them asked the other passengers about the owner of the bag, but no
one know. When the jeepney reached the poblacion, PO2 Pallayoc alighted together with other
passengers. Unfortunately, he did not noticed who took the black backpack from atop the
jeepney. He only realized a few moments later that the said bag and 3 other bags were already
being carried away by two (2) women. He caught up with the women and introduced himself as a
policeman. He told them that they were under arrest, but the women got away.

RULING:

UNDER ARTICLE III, SECTION 2 OF THE PHILIPPINE CONSTITUTION PROVIDES: The


right of the People to be secure in their persons, houses, papers, and effects against unreasonable
searches and seizures of whatever nature and for any purpose shall be inviolable, and no search
warrant or warrant of arrest shall issue except upon probable cause to be determined personally
by the Judge after examination under oath or affirmation of the complainant and the witnesses he
may produce, and particularly describing the place to be searched and the persons or things to be
seized.

But In this case the WARRANTLESS SEARCH is justified.

Reason: MOVING VEHICLE (WARRANTLESS SEARCH) This has been justified on the
ground that the mobility of motor vehicles makes it possible for the vehicle to be searched to
move out of the locality or jurisdiction in which the warrant must be sought.

This is no way, however, gives the police officers unlimited discretion to conduct warrantless
searches of automobiles in the absence of probable cause when a vehicle is stopped and
subjected to an extension search, such a warrantless search has been held to be valid only as long
as officers conducting the search have reasonable or probable cause to believe before the search
that they will find the instrumentality or evidence pertaining to a crime, in the vehicle to be
searched. When an accused is charged with illegal possession or transportation of prohibited
drugs, the ownership thereof is immaterial. Consequently, proof of ownership of the confiscated
marijuana is not necessary. Appellants alleged lack of knowledge does not constitute a valid
defence. Lack of criminal intent and good faith are not exempting circumstances where the crime
charge is malum prohibitum Based on the testimony of PO2 Pallayoc, after appellants arrest, she
was immediately brought to the police station where she stayed while waiting for the Mayor. It
was the Mayor who opened the packages, revealing the illegal drugs, which were thereafter
marked and sent to the police crime laboratory the following day. Contrary to appellants claim,
the prosecutions evidence establishes the chain of custody from the time ofappellants arrest until
the prohibited drugs were tested at the police crime laboratory.

While it is true that the arresting officer failed to state explicitly the justifiable ground for non-
compliance with Section 21, this does not necessarily mean that appellants arrest was illegal or
that the items seized are inadmissible. The justifiable ground will remain unknown because
appellant did not question the custody and disposition of the items taken from her during the
trial.[38] Even assuming that the police officers failed to abide by Section 21, appellant should
have raised this issue before the trial court. She could have moved for the quashal of the
information at the first instance. But she did not. Hence, she is deemed to have waived any
objection on the matter.

Further, the actions of the police officers, in relation to the procedural rules on the chain of
custody, enjoyed the presumption of regularity in the performance of official functions. Courts
accord credence and full faith to the testimonies of police authorities, as they are presumed to be
performing their duties regularly, absent any convincing proof to the contrary. [39]

In sum, the prosecution successfully established appellants guilt. Thus, her conviction must be
affirmed.

WHEREFORE, the foregoing premises considered, the appeal is DISMISSED. The Decision of
the Court of Appeals in CA-G.R. CR-HC No. 02718 is AFFIRMED (GUILTY as charged and
sentences here (sic) to suffer the penalty of life imprisonment and to pay a fine of P500,000.00.
The 7,030.3 grams of marijuana are ordered confiscated and turned over to the Philippine Drug
Enforcement Agency for destruction in the presence of the Court personnel and media.)

SO ORDERED.

ESPANO VS CA

RODOLFO ESPANO vs. COURT OF APPEALS and PEOPLE OF THE PHILIPPINES

G.R. No. 120431 April 1, 1998


Facts:

Pat. Pagilagan together with other police officers went to Zamora and Pandacan Streets, Manila
to confirm reports of drug pushing in the area. They saw petitioner selling something to another
person. After the alleged buyer left, they approached petitioner, identified themselves as
policemen, and frisked him. The search yielded two plastic cellophane tea bags of marijuana.
When asked if he had more marijuana, he replied that there was more in his house. The
policemen went to his residence where they found ten more cellophane tea bags of marijuana.
Petitioner was brought to the police headquarters where he was charged of possession of
prohibited drugs.

Issue:

Whether or not the pieces of evidence were inadmissible

Ruling:

The Supreme Court held that Section 5 Rule 113 of the Rules of Court provides:

Arrest without warrant; when lawful a peace officer or a private person may, without a
warrant, arrest a person:

When, in the presence, the person to be arrested has committed, is actually committing, or is
attempting to commit an offense . . .

Petitioners arrest falls squarely under the aforecited rule. He was caught in flagrante as a result
of a buy bust operation conducted by police officers on the basis of information received
regarding the illegal trade of drugs within the area. The police officer saw petitioner handling
over something to an alleged buyer. After the buyer left, they searched him and discovered two
cellophane of marijuana. His arrest was, therefore, lawful and the two cellophane bag of
marijuana seized were admissible in evidence, being fruits of the crime.

Carroll vs US 267 U.S. 132 (1925)

FACTS: This was during the time of the Prohibition Act, when liquor was banned in the United
States. Federal undercover agents arranged an undercover purchase to buy liquor from Kiro and
Carroll. They said they had to go to Grand Rapids, Michigan to get the liquor. The undercover
purchase never came to fruition because the two never returned. Later, while the agents were on
their rounds, they saw both on a highway. The agents gave chase, stopped them and searched the
car. They found 68 bottles of liquor. Carroll and Kiro say that this violates the Fourth
Amendment (search and seizure) because there was no search warrant.
ISSUE: Was the warrantless search on the automobile valid?

HELD: Yes, it was valid. This is now known as the Carroll doctrine: Search without a warrant
of an automobile, and seizure therein of liquor subject to seizure and destruction under the
Prohibition Act, do not violate the Amendment, if made upon probable cause, i.e., upon a belief,
reasonably arising out of circumstances known to the officer, that the vehicle contains such
contraband liquor.

PEOPLE VS. MALMSTEDT

FACTS: Accused-appellant Mikael Malmstedt, a Swedish national, entered the Philippines for
the third time in December 1988 as a tourist. On May 7, 1989, the accused went to Sagada and
stayed there for 2 days. On May 11, 1989, from Sagada, he took a bus ride going to Baguio.

On the same date (may 11), Captain Alen Vasco, the Commanding Officer of the First Regional
Command (NARCOM) stationed at Camp Dangwa, received reports that vehicles coming from
Sagada were transporting marijuana and other prohibited drugs and that a Caucasian coming
from Sagada had in his possession prohibited drugs. He then ordered his men to set up a
temporary checkpoint at Kilometer 14, Acop, Tublay, Mountain Province, for the purpose of
checking all vehicles coming from the Cordillera Region.

The bus where accused was riding reached the said checkpoint. 2 NARCOM officers inspected
the bus. One of them noticed a bulge on accused's waist. Suspecting the bulge on accused's waist
to be a gun, the officer asked for accused's passport and other identification papers. The accused
failure to comply, he was ordered to show what was bulging on his waist. The bulge contained
hashish, a derivative of marijuana which is a prohibited drug. He was brought to the headquarters
for investigation. On his luggage were teddy bears where other hashish was found.

The accused argues that the search of his personal effects was illegal because it was made
without a search warrant and, therefore, the prohibited drugs which were discovered during the
illegal search are not admissible as evidence against him.

ISSUE: Whether the search and seizure made was illegal.

HELD: No. The search is made pursuant to a lawful arrest, there is no need to obtain a search
warrant. Accused was searched and arrested while transporting prohibited drugs (hashish). A
crime was actually being committed by the accused and he was caught in flagrante delicto. Thus,
the search made upon his personal effects falls squarely under paragraph (1) of the foregoing
provisions of law, which allow a warrantless search incident to a lawful arrest.
Warrantless search of the personal effects of an accused has been declared by this Court as valid,
because of existence of probable cause, where the smell of marijuana emanated from a plastic
bag owned by the accused, or where the accused was acting suspiciously, and attempted to flee.

Mustang Lumber Inc. v CA

FACTS: On 1 April 1990, an information that a huge stockpile of narra flitches, shorts, and
slabs were seen inside the lumberyard of the petitioner. The Special Actions and Investigation
Division (SAID) of DENR organized a team of foresters and policemen and sent it to conduct
surveillance at the said lumberyard. They saw coming out from the lumberyard a truck loaded
with lumber of assorted sizes and dimensions. Since the driver could not produce the required
invoices and transport documents, the team seized the truck together with its cargo and
impounded them at the DENR compound.

On 3 April 1990, the team was able to secure a search warrant from Executive Judge Adriano R.
Osorio of the Regional Trial Court in Valenzuela. The team seized on that date from the
petitioners lumberyard four truckloads of narra shorts, trimmings, and slabs; a negligible number
of narra lumber; and approximately 200,000 board feet of lumbers.

On 4 April 1990, the team returned to the premises of the petitioner 's lumberyard in Valenzuela
and placed under administrative seizure the remaining stockpile of lumbers because the
petitioner failed to produce upon demand the corresponding certificate of lumber origin,
auxiliary invoices, tally sheets, and delivery receipts from the source of the invoices covering the
lumber to prove the legitimacy of their source and origin.

The accused questioned the seizure on 1 April 1990, without any search and seizure order issued
by a judge.

The trial court held that the warrantless search and seizure on 1 April 1990 of the petitioner's
truck, which was moving out from the petitioner's lumberyard in Valenzuela, Metro Manila,
loaded with large volumes of lumber without covering document showing the legitimacy of its
source or origin did not offend the constitutional mandate that search and seizure must be
supported by a valid warrant.

The petitioner appealed from the decision to the Court of Appeals however it was dismissed.

ISSUE: whether the seizure made on April 1 and the continuation of search and seizure on April
4 was illegal.

HELD: The seizure of such truck and its cargo was a valid exercise of the power . The search
was conducted on a moving vehicle. Such a search could be lawfully conducted without a search
warrant
Search of a moving vehicle is one of the five doctrinally accepted exceptions to the constitutional
mandate that no search or seizure shall be made except by virtue of a warrant issued by a judge
after personally determining the existence of probable cause. The other exceptions are (1) search
as an incident to a lawful arrest, (2) seizure of evidence in plain view, (3) customs searches, and
(4) consented warrantless search.

The search on 4 April 1990 was a continuation of the search on 3 April 1990 done under and by
virtue of the search warrant issued on 3 April 1990.

Under Section 9, Rule 126 of the Rules of Court, a search warrant has a lifetime of ten days.
Hence, it could be served at any time within the said period, and if its object or purpose cannot
be accomplished in one day, the same may be continued the following day or days until
completed. Thus, when the search under a warrant on one day was interrupted, it may be
continued under the same warrant the following day, provided it is still within the ten-day period.

Seizure of goods concealed to avoid duties/taxes

PAPA vs MAGO

FACTS: Reliable information was received by the Manila Police Department, to the effect that a
certain shipment of personal effects, allegedly misdeclared and undervalued, would be released
the following day from the customs zone of the port of Manila and loaded on two trucks, and
upon orders of petitioner Ricardo Papa, Chief of Police of Manila and a duly deputized agent of
the Bureau of Customs, conducted surveillance at gate No. 1 of the customs zone. When the
trucks left gate No. 1 at about 4:30 in the afternoon of November 4, 1966, elements of the
counter-intelligence unit went after the trucks and intercepted them at the Agrifina Circle,
Ermita, Manila. The load of the two trucks consisting of nine bales of goods, and the two trucks,
were seized on instructions of the Chief of Police. Upon investigation, a person claimed
ownership of the goods and showed to the policemen a "Statement and Receipts of Duties
Collected in Informal Entry No. 147-5501", issued by the Bureau of Customs in the name of a
certain Bienvenido Naguit.

ISSUE: WON the seizure of the goods are valid, taking into account the following irregularities:

a. the goods were seized without a valid warrant issued by a competent court;

b. that then Customs Commissioner Jacinto Gavino had illegally assigned appraisers to
examine the goods because the goods were no longer under the control and supervision of the
Commissioner of Customs; and

c. that the goods were not subject to seizure under Section 2531 of the Tariff and Customs
Code because Remedios Mago had bought them from another person without knowledge that
they were imported illegally.
HELD: The seizure was valid, notwithstanding the aforementioned irregularities.

Petitioner Martin Alagao and his companion policemen had authority to effect the seizure
without any search warrant issued by a component court. The Tariff and Customs Code does not
require said warrant in the instant case. The Code authorizes persons having police authority
under Section 2203 of the Tariff and Customs Code to enter, pass through or search any land,
inclosure, warehouse, store or building, not being a dwelling house and also to inspect, search
and examine any vessel or aircraft and any trunk, package, box or envelope or any person on
board, or stop and search and examine any vehicle, beast or person suspected of holding or
conveying any dutiable or prohibited article introduced into the Philippines contrary to law,
without mentioning the need of a search warrant in said cases. But in the search of a dwelling
house, the Code provides that said "dwelling house may be entered and searched only upon
warrant issued by a judge or justice of the peace ...." It is our considered view, therefore, that
except in the case of the search of a dwelling house, persons exercising police authority under
the customs law may effect search and seizure without a search warrant in the enforcement of
customs laws.

Pacis vs Pamaran

FACTS: Respondent Ricardo Santos is the owner of a Mercury automobile, model 1957. It was
brought into this country without the payment of customs duty and taxes, its owner Donald
James Hatch being tax-exempt. It was from him that respondent Santos acquired said car. 3 On
June 25, 1964, he paid 311.00 for customs duty and taxes.

On July 22, 1964 received from Pedro Pacis a letter to the effect that the Land Transportation
Commission reported that such automobile was a "hot car." Pacis is the Administrator of the
General Affairs Administration of the Department of National Defense. He looked in the records
and found that the customs taxes that should have been was 2,500, more or less. Based on such
discrepancy, on July 22, 1964, he instituted seizure proceedings and issued a warrant of seizure
and detention. On the strength thereof, the automobile was taken while it was parked on
Economia Street, Manila, by Department of National Defense agents who were authorized to do
so by virtue of the said warrant.

ISSUE: WON Pedro Pacis committed usurpation of authority when he issued a search warrant
and a warrant of arrest, considering that such powers were judicial functions.

HELD: There is no usurpation of authority. The search warrants issued and the search and
seizure performed in pursuance thereof is a valid exercise of police power. The Tariff and
Customs Code authorizes persons having police authority under Section 2203 of the Tariff and
Customs Code to enter, pass through or search any land, inclosure, warehouse, store or building,
not being a dwelling house and also to inspect, search and examine any vessel or aircraft and any
trunk, package, box or envelope or any person on board, or stop and search and examine any
vehicle, beast or person suspected of holding or conveing any dutiable or prohibited article
introduced into the Philippines contrary to law, without mentioning the need of a search warrant
in said cases. But in the search of a dwelling house, the Code provides that said "dwelling house
may be entered and searched only upon warrant issued by a judge or justice of the peace ...." It is
our considered view, therefore, that except in the case of the search of a dwelling house, persons
exercising police authority under the customs law may effect search and seizure without a search
warrant in the enforcement of customs laws.

From the foregoing, there was no such infringement. What was done by petitioner was strictly in
accordance with settled principles of law. No doubt need be entertained then as to the validity of
the issuance of the warrant of seizure and detention. His liability for any alleged usurpation of
judicial function is non-existent. Such imputation was definitely unfounded. Even if however the
matter were less clear, the claim that the search and seizure clause was in effect nullified is
hardly impressed with merit.

JOSE LOPEZ v. COMMISSIONER OF CUSTOMS

G.R. No. L-27968 December 3, 1975

Issue submitted "for resolution is the legality of the seizure made by the Collector of
Customs of Davao of the 1,408 sacks of copra and 86 sacks of coffee allegedly owned by the
petitioners.

FACTS:

M/V Jolo Lema had been under strict surveillance by the combined team of agents of the NBI,
PC, RASAC, and City Police of Davao prior to its apprehension at a private wharf in Batjak,
Sasa, Davao City. M/V [Jolo Lema] was skippered (sic) by Capt. Aquilino Pantinople and
chartered by Mr. Tomas Velasco. During the period from the latter part of August to September
18, 1966, the said vessel was in Indonesian waters where it loaded copra and coffee beans from
Taruna, Pitta, and Mangenito, all of Indonesia. In its trip to Indonesia it brought various
merchandise from the Philippines which were exchanged and/or bartered for copra and coffee
beans and subsequently taken to Davao City.

Search and Seizure:

Said vessel passed Marore, Indonesia on 18 September 1966 on its way to Tahuna, Indonesia
before proceeding to Davao City where it was apprehended on 19 September 1966. On the
question of the search of the hotel room, the petition alleged that at about 3:00 o'clock in the
afternoon of September 19, 1966, when the vessel was searched, a combined team of
Constabulary and Regional Anti-Smuggling Center operatives headed by NBI agent Earl
Reynolds raided the hotel room then being rented by petitioner Tomas Velasco without any
search warrant and in the absence at the time of such petitioner Tomas Velasco or the presence
of any other person, except one Teofila Ibaez, a mere manicurist of Davao City by occupation
and "forcibly opened luggages and boxes from which only several documents and papers were
found, then seized, confiscated and took away the same." or whether Mrs. Velasco volunteered
to open the suitcases and baggages of Velasco and delivered the documents and things contained
therein to Reynolds.

Collector of Customs of Davao seized 1,480 sacks of copra and 86 sacks of coffee from the
M/V motor vessel Jolo Lema.

Claims of the petitioners:

1.that the 1,408 sacks of copra and 86 sacks of coffee in question were purchased in Kiamba,
Lumatin, and Lumasal, all in the province of Cotabato, from a certain Osmea Juanday

2.that the said goods were not imported and of foreign origin, they are not legally subject to
seizure and forfeiture.

3. that the forfeiture made by the Collector of Customs of Davao was invalid because the said
forfeiture was based on documents and papers which were illegally seized by agents of the
Government through violence and intimidation.

Contention of the Respondent:

1.that the evidence is sufficient to hold that the goods in question came from Indonesia and
subsequently brought to the Philippines in violation of our laws and, therefore, subject to
forfeiture;

2. that the Indonesian documents and papers allegedly secured illegally by the combined team
NBI, PC and RASAC agents stationed in Davao, were in fact lawfully and validly secured by
them.

Respondents (represented by the then Solicitor General, now Associate Justice, Antonio P.
Barredo) : "(a) After Captain Pantinople informed the team that petitioner Tomas Velasco, the
charterer of the vessel, had other documents showing that vessel came from Indonesia carrying
smuggled copra and coffee, some members of the team proceeded to the room of petitioner
Velasco at the Skyroom Hotel in Davao City, to ask for said documents; (b) Although petitioner
Velasco was not inside the hotel room, respondent Reynolds, after identifying himself as a police
officer and after explaining his purpose, was allowed to enter the room by Mrs. Tomas Velasco
who subsequently volunteered to open the suitcases and baggages of petitioner Velasco and
delivered the documents and things contained therein to respondent Reynolds; ... (c) The said
police team did not search the room; neither did the members thereof forcibly open the luggages
and boxes nor seized and confiscated the documents and things contained therein, since that was
not necessary because ... Mrs. Tomas Velasco voluntarily opened the baggages and suitcases and
gave their contents of documents and things to respondent Reynolds. Such fact is also
established by the joint affidavit

Justice Martin as ponente: "There can be no question that without the proper search warrant, no
public official has the right to enter the premises of another without his consent for the purpose
of search and seizure."

ISSUE: Whether there was consent on the part of the person who was the occupant of the hotel
room then rented by Velasco.

HELD: There was an attempt on the part of Lopez and Velasco to counteract the force of the
recital of the written statement of Teofila Ibaez (allegedly wife of Tomas Velasco) by an
affidavit of one Corazon Y. Velasco, who stated that she is the legal wife of Velasco, and another
by Velasco himself; reiterating that the person who was present at his hotel room was one
Teofila Ibaez, "a manicurist by occupation." If such indeed were the case, then it is much more
easily understandable why that person, Teofila Ibaez, who could be aptly described as the
wrong person at the wrong place and at the wrong time, would have signified her consent readily
and immediately. Under the circumstances, that was the most prudent course of action. It would
save her and even Velasco himself from any gossip or innuendo. Nor could the officers of the
law be blamed if they would act on the appearances. There was a person inside who from all
indications was ready to accede to their request. Even common courtesy alone would have
precluded them from inquiring too closely as to why she was there. Under all the circumstances,
therefore, it can readily be concluded that there was consent sufficient in law to dispense with the
need for a search warrant.

PEOPLE v. REYNALDO CRUZ (alias RENE HAPON)

In Criminal Case No. Q-45491 of the Regional Trial Court of Quezon City, Reynaldo Cruz
alias Rene Hapon of No. 40 Sto. Cristo, Balintawak, Quezon City, was charged with the
crime of Illegal Possession of Firearm and Ammunition

The firearm and ammunition and the hand grenade, as well as the bag which contained the
same, were confiscated and ordered forfeited in favor of the government.

FACTS:

On May 9, 1986, Lt. Noel Manabat, along with the elements of CRIG stationed at Camp
Bagong Diwa, Taguig, acting on an intelligence information of a reliable informant that about
noon of that day a stolen car, coming from Quezon City, was to be sold somewhere in
MagaIlanes, Makati, nabbed Romeo Fernandez and Joey Flores at the intersection of EDSA and
Timog Street and brought them to headquarters.
After some questioning, these two (2) carnap suspects led the eight man-CRIG team to 61
Mabituan Street, Masambong, Quezon City where they alleged the other members of the carnap
gang were waiting for their shares of the proceeds from the sale of a vehicle.

At said address, a sister of appellant, who owned the apartment, opened the door to the CRIG
team. Inside the apartment, the team found appellant, sleeping on the floor, and gangmates
Herminio Rivera and Lolito Timcang. The team recognized appellant because he was pointed to
by Romeo Fernandez and Joey Flores.

These two also informed the team that appellant was armed and the team found a clutch bag
containing a caliber .38 paltik revolver, one (1) live ammunition and a hand grenade under a bar,
located one (1) meter away from the slumbering appellant.

After waking him up, sgt. Reynaldo Cachuela confronted him.

Appellant, admitted ownership of the bag, firearm, bullet and grenade. The team arrested
appellant and the rest of his group, as well as confiscated the items of the crime.

At headquarters, Lt. Noel Manabat and Sgt. Reynaldo Cachuela narrated under oath the incident
and turned over to Sgt. Jesus Ordinario, police investigator, the appellant and his party and C.
Sgt. Jesus Ordinario took down the statement of appellant (Exhibit A) wherein appellant
admitted ownership of Exhibits B, B-1, C and D, after informing- him of his constitutional
rights, such as, he had the right to remain silent the right to get his own counsel, and everything
he would state in his statement might be used for or against him.

Appellant was told that the camp had a lawyer who was ready to assist him but he replied that he
needed no lawyer and was ready to confess the truth. Exhibit A reflected appellant's answer to a
question, thus: "Bakit ka naman may baril at granada, saan mo gagamitin? Nakuha sa posesyon
ko ... at iyon ay ipinabenta . . ."After taking down the statement of appellant and the carnap
suspects Herminio Rivera and Lolito Timcang, Sgt. Jesus Ordinario prepared a written referral of
the case to the City Fiscal of Quezon City (Exhibit F), which was signed by Major Eduardo S.
Amoyo.

On June 25, 1986: Lilian Lauron of the Legal Research Branch of the Firearms and Explosives
Unit, Camp Crame, Quezon City, in answer to a police inquiry, determined that appellant
Reynaldo Cruz y Santos of No. 40 Sto. Cristo, Balintawak, Quezon City was not a licensed
holder of caliber .38 paltik revolver. As to hand grenades, she declared that only military
personnel were authorized to carry them.

Contention of the Accused:

1. The accused, denied ownership or possession of the firearm and hand grenade, as well as the
bag, which contained the same.
2. that the bag and its contents belonged to Joey Flores and was "planted" by PC operatives (was
corroborated by Romeo Fernandez and Dionisio Daracin. ) His version of the case is, as follows:

In the afternoon of 8 May 1986, between 4:30 and 5:00 o'clock, he was sleeping in the house of
his sister at No. 61 Mabituan Street, Masambong, Quezon City, together with Eutiquio Lapinig,
Jaime Rivera, and Dionisio Daracin, when PC men barged inside and rudely awakened him. He
was confronted with a gun and a hand grenade, but he denied ownership of the same. Then, he
and his companions were tied up and brought to Camp Bicutan on board a six-by-six truck.
There, he was tortured by PC officers for five days and made to sign a prepared statement
wherein he admitted ownership of the gun and hand grenade. He was not allowed to have visitors
during the period of his confinement. However, he was able to secure a medical certificate for
the injuries inflicted upon him by the PC officers, but it was left with them.

Ownership, however, is not an essential element of the offense charged. What the law requires is
merely possession, which includes not only actual physical possession but also constructive
possession or the subjection of the thing to one's control and management.

PC Lt. Noel Manabat, leader of the PC team that arrested the accused and his companions for
"carnapping," declared that after finding the bag containing the revolver and the hand grenade
under the bar in the apartment of the sister of the accused, he asked the "carnapping" suspects to
whom the bag belonged and they replied that it belonged to the herein accused Reynaldo Cruz,
and when he confronted the accused with the revolver and hand grenade, the accused readily
admitted that they belonged to him.

Appellants Contention:

1. that the trial court for giving credence to the testimony of the prosecution
witnesses which he claims to be hearsay, conflicting and biased, but the appellant does not point
to specific portions of said testimony which are allegedly conflicting and biased.

2. that the firearm and explosive in question cannot e use as evidence against him
since the PC officers had no warrant of arrest when they entered the apartment, in violation of his
constitutional rights.

The contention is devoid of merit. PC Sgt. Reynaldo Cachuela categorically declared and this
is not disputed that they were allowed by the owner of the apartment to enter.

Besides, it should be noted that the unlicensed firearm and explosive were found when they
arrested the accused and his companions for "carnapping" and not for illegal possession of
firearm ammunition. In Magoncia vs. Palacio, the Court ruled that an unlicensed firearm may be
seized without the necessity of obtaining a search warrant.

As Mr. Justice Perfecto explained it in his concurring opinion in said case:


... The illegality of the search is independent from the illegal possession of prohibited arms. The
illegality of the search did not make legal an illegal possession of firearms. When, in pursuing an
illegal action or, in the commission of a criminal offense, the offending Police officers should
happen to discover, a criminal offense being committed by any person, they are not precluded
from performing their duties as police officers for the apprehension of the guilty person all the
taking of the corpus delicti.

Finally, the accused, citing the case of Morales vs. Ponce Enrile, claims that the extrajudicial
confession obtained from him during custodial investigation, is inadmissible in evidence against
him for having been obtained in violation of his constitutional rights.

We agree. It would appear that the police officers failed to comply with the strictures laid down
by the Court in the cited case of Morales vs. Ponce Enrile, for police officers to follow in a
custodial investigation in that, while Police Sgt. Jesus Ordinario testified that he had informed
the accused of his constitutional rights to remain silent and to be represented by counsel and that
the accused waived such rights, the waiver of constitutional rights was not made with the
assistance or even in the presence of counsel.

However, the conviction of the appellant is not based upon his extra-judicial confession alone.
The evidence presented by the prosecution, even without said extrajudicial confession, is
abundant, to support a finding of guilty

WHEREFORE, the judgment appealed from is hereby AFFIRMED, with costs against the
accused-appellant.

4. Seize of Evidence in Plain View

4.1 People v. Abe Valdez

FACTS:

SPO3 Marcelo Tipay, a member of the police force of Villaverde, Nueva Vizcaya, received a tip
from an unnamed informer about the presence of a marijuana plantation. It was allegedly owned
by Abe Valdez, and it was situated close to his hut. Police Inspector Alejandro Parungao, Chief
of Police of Villaverde, Nueva Vizcaya then formed a reaction team from his operatives to verify
the report. The team was composed of five members, namely, SPO3 Marcelo Tipay, SPO2 Noel
Libunao, SPO2 Pedro Morales, SPO1 Romulo Tobias, and PO2 Alfermer Balut. The specific
instructions of Inspector Parungao were, to uproot said marijuana plants and arrest the cultivator
of the same.

Version of the Prosecution: At approximately 5:00 AM the following day, the police team,
together with the informant, went to the site where the marijuana was allegedly being grown.
After a three-hour, uphill trek, they arrived at the site and found Valdez alone in his nipa hut.
They looked for the marijuana plants around the area and spotted seven (7) five-foot high,
flowering marijuana plants in two rows, approximately twentyfive (25) meters from Valdezs
hut. PO2 Balut asked Valdez who owned the prohibited plants, and Valdez admitted that they
were his. Subsequently, the police team uprooted the seven (7) marijuana plants which weighed
2.194 kilograms. They even took photos of Valdez standing beside the cannabis plants. Valdez
was then arrested.

One of the plants weighing 1.090 kilograms was sent to the Philippine National Police Crime
Laboratory for analysis. Inspector Prevy Fabros Luwis affirmed that it was marijuana upon
finding calcium carbonate, a positive indication for marijuana. The prosecution also presented a
certification from the DENR that acknowledged Valdez as the occupant of the lot where the
marijuana was found.

Version of the Defense: At around 10:00 AM, Valdez was weeding his vegetable farm in Sitio
Bulan when he was called by a person who he did not know. This unknown person asked Valdez
to go with him to see something. The unknown person led Valdez to the place where the
marijuana plants were planted, approximately one hundred (100) meters from his nipa hut. Five
armed policemen were there, and asked Valdez if he had any knowledge of the marijuana plants
which Valdez denied. Upon Valdezs denial, SPO2 Libunao poked a fist at him and told him to
admit ownership of the plants. Valdez was so nervous and afraid that he admitted ownership of
the marijuana. The police then took a photo of him standing in front of one of the marijuana
plants. They also made him uproot five (5) of the marijuana plants and bring them to his hut,
where another photo of him standing next to the pile of cannabis plants was taken.

The police team brought Valdez to the police station. On the way, they were with a certain Kiko
Pascua, a barangay peace officer. Pascua threatened Valdez to admit ownership of the marijuana,
otherwise he would be put in a bad situation. It turns out that Pascua bore a grudge against
Valdez because Valdez refused to participate in Pascuas illegal logging activities. At the police
headquarters, Valdez reiterated that he knew nothing about the marijuana plants seized by the
police.

Contention of the Accused: There were ten other houses around the site where the marijuana
plants were planted, the nearest one being one hundred (100) meters away. The site is located
between the house of Valdez and the house of Carlito Pascua (Kiko Pascuas uncle). The
cannabis plants were not planted in the lot which Valdez cultivated. Thus, it cannot be concluded
that Valdez owned the marijuana plantation.

The Prosecutions Rebuttal: According to Tipay, the marijuana plantation was forty (40) meters
away from Valdezs hut, and two hundred fifty (250) meters away from Carlito Pascuas hut.
Tipay also admitted that no surveyor accompanied him when he made the measurements. He
further stated that his basis for claiming that Valdez was the owner of the cannabis plants was the
information given him by the police informer and the proximity of Valdezs hut to the site of the
cannabis plants.

***The Regional Trial Court of Bayombong, Nueva Vizcaya found Abe Valdez guilty beyond
reasonable doubt for violating Section 9 of the Dangerous Drugs Act of 1972. He was sentenced
to suffer the penalty of death by lethal injection.

Automatic Review of the Supreme Court

Contention of the Accused: There was an unlawful search because (1) the law enforcers had
ample time to secure a search warrant; and (2) although the marijuana plants were found in an
unfenced lot, that does not remove appellant from the mantle of protection against unreasonable
searches and seizures.

Contention of the State: The OSG argues that there was no search made by the police team in the
first place. The OSG points out that the marijuana plants in question were grown in an unfenced
lot and as each grew about five (5) feet tall, they were visible from afar, and were, in fact,
immediately spotted by the police officers when they reached the site. The seized marijuana
plants were, thus, in plain view of the police officers. The instant case must, therefore, be treated
as a warrantless lawful search under the "plain view" doctrine.

ISSUES:

1. Whether the search and seizure of the marijuana plants is lawful; and

2. Whether the seized plants are admissible in evidence against the accused.

HELD:

1. No, the search and seizure of the marijuana plants is unlawful. In the instant case, there
was no search warrant issued by a judge after personal determination of the existence of
probable cause. From the declarations of the police officers themselves, it is clear that
they had at least one (1) day to obtain a warrant to search appellant's farm. Their
informant had revealed his name to them. The place where the cannabis plants were
planted was pinpointed. From the information in their possession, they could have
convinced a judge that there was probable cause to justify the issuance of a warrant. But
they did not. Instead, they uprooted the plants and apprehended the accused on the excuse
that the trip was a good six hours and inconvenient to them. We need not underscore that
the protection against illegal search and seizure is constitutionally mandated and only
under specific instances are searches allowed without warrant. The mantle of protection
extended by the Bill of Rights covers both innocent and guilty alike against any form of
high-handedness of law enforcers, regardless of the praiseworthiness of their intentions.
We find no reason to subscribe to Solicitor General's contention that we apply the "plain view"
doctrine. For the doctrine to apply, the following elements must be present:

(a) a prior valid intrusion based on the valid warrantless arrest in which the police are legally
present in the pursuit of their official duties;

(b) the evidence was inadvertently discovered by the police who have the right to be where they
are; and

(c) the evidence must be immediately apparent; and

(d) plain view justified mere seizure of evidence without further search.

In the instant case, recall that PO2 Balut testified that they first located the marijuana plants
before appellant was arrested without a warrant. Hence, there was no valid warrantless arrest
which preceded the search of appellant's premises. Note further that the police team was
dispatched to appellant's kaingin precisely to search for and uproot the prohibited flora. The
seizure of evidence in "plain view" applies only where the police officer is not searching for
evidence against the accused, but inadvertently comes across an incriminating object. Clearly,
their discovery of the cannabis plants was not inadvertent. We also note the testimony of SPO2
Tipay that upon arriving at the area, they first had to "look around the area" before they could
spot the illegal plants. Patently, the seized marijuana plants were not "immediately apparent" and
a "further search" was needed. In sum, the marijuana plants in question were not in "plain view"
or "open to eye and hand." The "plain view" doctrine, thus, cannot be made to apply.

Nor can we sustain the trial court's conclusion that just because the marijuana plants were found
in an unfenced lot, appellant could not invoke the protection afforded by the Charter against
unreasonable searches by agents of the State. The right against unreasonable searches and
seizures is the immunity of one's person, which includes his residence, his papers, and other
possessions. The guarantee refers to "the right of personal security" of the individual. As
appellant correctly points out, what is sought to be protected against the State's unlawful
intrusion are persons, not places. To conclude otherwise would not only mean swimming against
the stream, it would also lead to the absurd logic that for a person to be immune against
unreasonable searches and seizures, he must be in his home or office, within a fenced yard or a
private place. The Bill of Rights belongs as much to the person in the street as to the individual
in the sanctuary of his bedroom.

2. No, the seized plants are not admissible in evidence against the accused. We find that said
plants cannot, as products of an unlawful search and seizure, be used as evidence against
appellant. They are fruits of the proverbial poisoned tree. It was, therefore, a reversible error on
the part of the court a quo to have admitted and relied upon the seized marijuana plants as
evidence to convict appellant.
***Abe Valdez has been acquitted for insufficiency of evidence.

4.2 Abraham Miclat, Jr. v. People

FACTS:

Abraham Miclat, Jr. (Abe) was convicted by the Regional Trial Court of Caloocan City of
violating the Comprehensive Dangerous Drugs Act of 2002.

At around 1:00 PM, Police Inspector Jose Valencia received an INFOREP Memo from Camp
Crame relative to the illicit and down-right drug-trading activities happening along Palmera
Spring II, Bagumbong, Caloocan City involving Abe Miclat and two others. Valencia formed a
surveillance team headed by SPO4 Ernesto Palting and is composed of 5 more operatives. After
a short briefing at their station, and rented a passenger jeepney, and proceeded to the target area.

When the group of SPO4 Palting arrived at the target area at around 3:50 PM the same day, they
were at once led by their informant to the house of one Alias Abe. PO3 Antonio then positioned
himself at the perimeter of the house, while the rest of the members of the group deployed
themselves nearby. Through a small opening in the curtain-covered window, PO3 Antonio
peeped inside and there at a distance of 1 meter, he saw Abe arranging several pieces of small
plastic sachets which he believed to be containing shabu. Slowly, said operative inched his way
in by gently pushing the door as well as the plywood covering the same. Upon gaining entrance,
PO3 Antonio forthwith introduced himself as a police officer while Abe, on the other hand, after
being informed of such authority, voluntarily handed over to the former the four (4) pieces of
small plastic sachets the latter was earlier sorting out. PO3 Antonio immediately placed the
suspect under arrest and brought him and the four (4) pieces of plastic sachets containing white
crystalline substance to their headquarters and turned them over to PO3 Fernando Moran for
proper disposition. The suspect was identified as Abraham Miclat, 19 years old, single, and
jobless.

ISSUE: Whether peeping through a curtain-covered window is within the meaning of the plain
view doctrine for a warrantless seizure to be lawful.

HELD:

Yes, peeping through a curtain-covered window is within the purview of the plain view doctrine.

Objects falling in plain view of an officer who has a right to be in a position to have that
view are subject to seizure even without a search warrant and may be introduced in
evidence. The plain view doctrine applies when the following requisites concur: (a) the law
enforcement officer in search of the evidence has a prior justification for an intrusion or is
in a position from which he can view a particular area; (b) the discovery of evidence in
plain view is inadvertent; (c) it is immediately apparent to the officer that the item he
observes may be evidence of a crime, contraband or otherwise subject to seizure. The law
enforcement officer must lawfully make an initial intrusion or properly be in a position from
which he can particularly view the area. In the course of such lawful intrusion, he came
inadvertently across a piece of evidence incriminating the accused. The object must be open to
eye and hand and its discovery inadvertent

4.3 ELENITA FAJARDO VS PEOPLE

FACTS:

Elenita Fajardo, and one Zaldy Valerio were charged with violation of P.D. No. 1866 before
the RTC, Branch 5, Kalibo, Aklan. In the evening of August 27, 2002, members of the Provincial
Intelligence Special Operations Group (PISOG) were instructed by P/Supt. Mendoza to respond
to the complaint of residents of Barangay Andagao that armed men drinking liquor at the
residence of Fajardo were indiscriminately firing guns.The responding team saw Valerio holding
two .45 caliber pistols. Fajardo was seen tucking a .45 caliber handgun between her waist and the
waistband of her shorts, after which, she entered the house and locked the main door.

SPO2 Nava saw Valerio emerge twice on top of the house and throw something. The discarded
objects landed near the wall of petitioners house and inside the compound of a neighboring
residence. SPO2 Nava recovered the discarded objects, which turned out to be two receivers of
.45 caliber pistol, model no. M1911A1 US, with serial number 763025, and model no.
M1911A1 US, with a defaced serial number. The recovered items were then surrendered to
SPO1 Nathaniel A. Tan (SPO1 Tan), Group Investigator, who utilized them in applying for and
obtaining a search warrant.

The warrant was served on petitioner at 9:30 a.m. Together with a barangay captain, barangay
kagawad, and members of the media, as witnesses, the police team proceeded to search
petitioners house. The team found and was able to confiscate the following:

1. Two (2) pieces of Short Magazine of M16 Armalite Rifle;


2. Thirty five (35) pieces of live M16 ammos 5.56 Caliber; and
3. Fourteen (14) pieces of live ammos of Caliber 45 pistol.

Since petitioner and Valerio failed to present any documents showing their authority to possess
the confiscated firearms and the two recovered receivers, a criminal information for violation of
P.D. No. 1866, as amended by Republic Act (R.A.) No. 8294, was filed against them.

Fajardo and Valerio argued that the issuance of the search warrant was defective because the
allegation contained in the application filed and signed by SPO1 Tan was not based on his
personal knowledge.
They further asserted that the execution of the search warrant was infirm since Fajardo who was
inside the house at the time of the search, was not asked to accompany the policemen as they
explored the place, but was instead ordered to remain in the living room.

ISSUE:

W/not the discovery of the two (2) receivers does not come within the purview of the plain view
doctrine.

HELD:

We hold that the receivers were seized in plain view, hence, admissible.

No less than our Constitution recognizes the right of the people to be secure in their persons,
houses, papers, and effects against unreasonable searches and seizures. This right is encapsulated
in Article III, Section 2, of the Constitution, which states:

Sec. 2. The right of the people to be secure in their persons, houses, papers, and effects against
unreasonable searches and seizures of whatever nature and for any purpose shall be inviolable,
and no search warrant or warrant of arrest shall issue except upon probable cause to be
determined personally by the judge after examination under oath or affirmation of the
complainant and the witnesses he may produce, and particularly describing the place to be
searched and the persons or things to be seized.

Complementing this provision is the exclusionary rule embodied in Section 3(2) of the same
article

(2) Any evidence obtained in violation of this or the preceding section shall be inadmissible for
any purpose in any proceeding.

There are, however, several well-recognized exceptions to the foregoing rule. Thus, evidence
obtained through a warrantless search and seizure may be admissible under any of the following
circumstances: (1) search incident to a lawful arrest; (2) search of a moving motor vehicle; (3)
search in violation of custom laws; (4) seizure of evidence in plain view; and (5) when the
accused himself waives his right against unreasonable searches and seizures.[18]
Under the plain view doctrine, objects falling in the plain view of an officer, who has a right to
be in the position to have that view, are subject to seizure and may be presented as evidence.[19]
It applies when the following requisites concur: (a) the law enforcement officer in search of the
evidence has a prior justification for an intrusion or is in a position from which he can view a
particular area; (b) the discovery of the evidence in plain view is inadvertent; and (c) it is
immediately apparent to the officer that the item he observes may be evidence of a crime,
contraband, or otherwise subject to seizure. The law enforcement officer must lawfully make an
initial intrusion or properly be in a position from which he can particularly view the area. In the
course of such lawful intrusion, he came inadvertently across a piece of evidence incriminating
the accused. The object must be open to eye and hand, and its discovery inadvertent.

HARRIS v. UNITED STATES, (1968)

Argued: January 18, 1968 Decided: March 5, 1968

CRIME: Robbery (2-7 years imprisonment)

FACTS:

Petitioner's automobile had been seen leaving the site of the robbery. The car was traced and
petitioner was arrested as he was entering it, near his home. The police decided to impound the
car as evidence, and a crane was called to tow it to the precinct.

A regulation of the Metropolitan Police Department requires the officer who takes an impounded
vehicle in charge to search the vehicle thoroughly, to remove all valuables from it, and to attach
to the vehicle a property tag listing certain information about the circumstances of the
impounding. Pursuant to this regulation, and without a warrant, the arresting officer proceeded to
the lot to which petitioner's car had been towed, in order to search the vehicle, to place a property
tag on it, to roll up the windows, and to lock the doors.

Proceeding to the front door on the passenger side, the officer opened the door in order to secure
the window and door. He then saw the registration card, which lay face up on the metal stripping
over which the door closes. The officer returned to the precinct, brought petitioner to the car, and
confronted petitioner with the registration card. Petitioner disclaimed all knowledge of the card.
The officer then seized the card and brought it into the precinct.

At his trial in the United States District Court for the District of Columbia, petitioner moved to
suppress an automobile registration card belonging to the robbery victim, which the Government
sought to introduce in evidence. The trial court, after a hearing, ruled that the card was
admissible.

Petitioner's conviction was affirmed by the Court of Appeals over his contention that the card
had been illegally seized following a warrantless search.

ISSUE: Whether the officer discovered the registration card by means of an illegal search.
HELD:

It does not constitute illegal search.

The admissibility of evidence found as a result of a search under the police regulation is not
presented by this case. The precise and detailed findings of the District Court, accepted by the
Court of Appeals, were to the effect that the discovery of the card was not the result of a search
of the car, but of a measure taken to protect the car while it was in police custody. Nothing in the
Fourth Amendment requires the police to obtain a warrant in these narrow circumstances.

Once the door had lawfully been opened, the registration card, with the name of the robbery
victim on it, was plainly visible. It has long been settled that objects falling in the plain view of
an officer who has a right to be in the position to have that view are subject to seizure and may
be introduced in evidence.

Though Preston v. United States, is not mentioned in the Court's opinion, I assume it has
survive because in the present case (1) the car was lawfully in police custody, and the police
were responsible for protecting the car; (2) while engaged in the performance of their duty to
protect the car, and not engaged in an inventory or other search of the car, they came across
incriminating evidence.

People vs. BASILIO DAMASO @ Bernardo/BERNIE MENDOZA @ KA DADO, accused-


appellant.

CRIME: Violation of Presidential Decree No. 1866 (unlawful possession of firearm) in


furtherance of, or incident to, or in connection with the crime of subversion

FACTS:

Lt. Candido Quijardo, a Philippine Constabulary officer and his group proceeded to Bonuan,
Dagupan City, to verify the presence of CPP/NPA and put under surveillance the rented
apartment of Rosemarie Aritumba, sister of Berlina Aritumba whom they earlier arrested. They
interviewed Luzviminda Morados, a visitor of Rosemarie Aritumba. She stated that she worked
with Bernie Mendoza (Damaso), herein appellant. She guided the group to the house rented by
appellant but found that it had already been vacated by the occupants.

Since Morados was hesitant to give the new address of Bernie Mendoza (Damaso), the
group looked for the Barangay Captain of the place and requested him to point out the new house
rented by appellant. The group again required Morados to go with them. When they reached the
house, the group saw Luz Tanciangco outside. They told her that they already knew that she was
a member of the NPA in the area. At first, she denied it, but when she saw Morados she
requested the group to go inside the house. Upon entering the house, the group, as well as the
Barangay Captain, saw radio sets, pamphlets entitled "Ang Bayan," xerox copiers and a
computer machine.

They also found persons who were companions of Luz Tanciangco. The group requested the
persons in the house to allow them to look around. When Luz Tanciangco opened one of the
rooms, they saw books used for subversive orientation, one M-14 rifle, bullets and ammunitions,
Kenwood radio, artificial beard, maps of the Philippines, Zambales, Mindoro an(d) Laguna and
other items.

They confiscated the articles and brought them to their headquarters for final inventory. They
likewise brought the persons found in the house to the headquarters for investigation. Said
persons revealed that appellant was the lessee of the house and owned the items confiscated
therefrom.

Based on this, Damaso was charged with illegal possession of firearms.

ISSUE: Whether the evidence is admissible

HELD: NO

The records of this case show that the accused-appellant was singled out as the sole violator of
P.D. No. 1866, in furtherance of, or incident to, or in connection with the crime of subversion.
Yet, there is no substantial and credible evidence to establish the fact that the appellant is
allegedly the same person as the lessee of the house where the M-14 rifle and other subversive
items were found or the owner of the said items.

The witnesses testimonies are hearsay because the witnesses testified on matters not on their
own personal knowledge. The Solicitor General, however, argues that while the testimonies may
be hearsay, the same are admissible because of the failure of counsel for appellant to object
thereto.

It is true that the lack of objection to a hearsay testimony results in its being admitted as
evidence. But, one should not be misled into thinking that since these testimonies are admitted as
evidence, they now have probative value. Hearsay evidence, whether objected to or not, cannot
be given credence.

It is unfortunate that the prosecution failed to present as witnesses the persons who knew the
appellant as the lessee and owner of the M-14 rifle. In this way, the appellant could have
exercised his constitutional right to confront the witnesses and to cross-examine them for their
truthfulness. Likewise, the records do not show any other evidence which could have identified
the appellant as the lessee of the house and the owner of the subversive items. To give probative
value to these hearsay statements and convict the appellant on this basis alone would be to render
his constitutional rights useless and without meaning.
Even assuming for the sake of argument that the appellant is the lessee of the house, the case
against him still will not prosper, the reason being that the law enforcers failed to comply with
the requirements of a valid search and seizure proceedings.

The right against unreasonable searches and seizures is enshrined in the Constitution (Article III,
Section 2). The purpose of the law is to prevent violations of private security in person and
property, and unlawful invasions of the sanctity of the home by officers of the law acting under
legislative or judicial sanction and to give remedy against such usurpations when attempted.
However, such right is not absolute. There are instances when a warrantless search and seizure
becomes valid, namely: (1) search incidental to an arrest; (2) search of a moving vehicle; and (3)
seizure of evidence in plain view. None of these exceptions is present in this case.

The constitutional immunity from unreasonable searches and seizures, being personal one,
cannot be waived by anyone except the person whose rights are invaded or one who is expressly
authorized to do so in his or her behalf.

In the case at bar, the records show that appellant was not in his house at that time Luz
Tanciangco and Luz Morados, his alleged helper, allowed the authorities to enter it. We find no
evidence that would establish the fact that Luz Morados was indeed the appellant's helper or if it
was true that she was his helper, that the appellant had given her authority to open his house in
his absence. The prosecution likewise failed to show if Luz Tanciangco has such an authority.
Without this evidence, the authorities' intrusion into the appellant's dwelling cannot be given any
color of legality.

ACCORDINGLY, the decision appealed from is hereby REVERSED and the appellant is
ACQUITTED with costs de oficio.

LEONA PASION VIUDA DE GARCIA, petitioner, vs. DIEGO LOCSIN, Judge of First
Instance of Tarlac, FELIX IMPERIAL, Provincial Fiscal of Tarlac, and the ANTI-USURY
BOARD, respondents

Facts:
Mariano G. Almeda, an agent of the Anti-Usuary Board, obtained from the justice of the peace
of Tarlac, a search warrant commanding any officer of the law to search the person, house or
store of the petitioner at Victoria, Tarlac, for certain books, lists, chits, receipts, documents and
other papers relating to her activities as usurer. The search warrant was issued upon an affidavit
given by the said Almeda.
On the same date, the said Mariano G. Almeda, accompanied by a captain of the Philippine
Constabulary, went to the office of the petitioner in Victoria, Tarlac and, after showing the
search warrant to the petitioners bookkeeper, Alfredo Salas, and, without the presence of the
petitioner who was ill and confined at the time, proceeded with the execution thereof
The papers and documents seized were kept for a considerable length of time by the Anti-Usury
Board and thereafter were turned over by it to the respondent fiscal who subsequently filed six
separate criminal cases against the herein petitioner for violation of the Anti-Usury Law.
The legality of the search warrant was challenged by counsel for the petitioner in the six criminal
cases and the devolution of the documents demanded. The respondent Judge denied the
petitioners motion for the reason that though the search warrant was illegal, there was a waiver
on the part of the petitioner.

The important question presented is whether upon the facts and under the circumstances of the
present case, there has been a waiver by the petitioner of her constitutional immunity against
unreasonable searches and seizures.

While the Solicitor-General admits that, in the light of decisions of this court, the search warrant
was illegally issued, he maintains:
(1) that the petitioner had waived her constitutional right by her acquiescence after the search
and seizure, and
(2) that the application for the return of the documents illegally seized was made after an
unreasonable length of time after the date of seizure.

HELD:
Freedom from unreasonable searches and seizures is declared a popular right and for a search
warrant to be valid:
(1) it must be issued upon probable cause;
(2) the probable cause must be determined by the judge himself and not by the applicant or any
other person;
(3) in the determination of probable cause, the judge must examine, under oath or affirmation,
the complainant and such witnesses as the latter may produce; and
(4) the warrant issued must particularly describe the place to be searched and persons or things to
be seized.

In the instant case the existence of probable cause was determined not by the judge himself but
by the applicant. All that the judge did was to accept as true the affidavit made by agent Almeda.
He did not decide for himself.

It does not appear that he examined the applicant and his witnesses, if any. Even accepting the
description of the properties to be seized to be sufficient and on the assumption that the receipt
issued is sufficiently detailed within the meaning of the law, the properties seized were not
delivered to the court which issued the warrant, as required by law.

Instead, they were turned over to the respondent provincial fiscal & used by him in building up
cases against petitioner. Considering that at the time the warrant was issued, there was no case
pending against the petitioner, the averment that the warrant was issued primarily for exploration
purposes is not without basis.

WAIVER OF RIGHT OR GIVES HIS CONSENT

The constitutional immunity against unreasonable searches and seizures is a personal right which
may be waived. The waiver may be either express or implied. No express waiver has been made
in the case.

It is well-settled that to constitute a waiver of a constitutional right, it must appear;


-first, that the right exists;
-secondly, that the persons involved had knowledge, either actual or constructive, of the
existence of such right; and,
-lastly, that said person had an actual intention to relinquish the right.

It is true that the petitioner did not object to the legality of the search when it was made. She
could not have objected because she was sick and was not present when the warrant was served
upon Alfredo Salas. Certainly, the constitutional immunity from unreasonable searches and
seizures, being a personal one, cannot be waived by anyone except the person whose rights are
invaded or one who is expressly authorized to do so in his or her behalf.

..Search warrant, is hereby declared void and of no effect..

JOSE G. LOPEZ and TOMAS VELASCO, petitioners, vs. COMMISSIONER OF


CUSTOM

Facts:
M/V Jolo Lema had been under strict surveillance by the combined team of agents of the NBI,
PC, RASAC, and City Police of Davao prior to its apprehension at a private wharf in Batjak,
Sasa, Davao City. M/V [Jolo Lema] was skippered (sic) by Capt. Aquilino Pantinople and
chartered by Mr. Tomas Velasco. During the period from the latter part of August to September
18, 1966, the said vessel was in Indonesian waters where it loaded copra and coffee beans from
Taruna, Pitta, and Mangenito, all of Indonesia. In its trip to Indonesia it brought various
merchandise from the Philippines which were exchanged and/or bartered for copra and coffee
beans and subsequently taken to Davao City.

Said vessel passed Marore, Indonesia on 18 September 1966 on its a way to Tahuna, Indonesia
before proceeding to Davao City where it was apprehended on 19 September 1966. At about
3:00 p.m. of the said day, when the vessel was searched and after Captain Pantinople informed
the team that Velasco, the charterer of the vessel, had other documents showing that vessel came
from Indonesia carrying smuggled copra and coffee, a combined team of Constabulary and
Regional Anti-Smuggling Center operatives headed by Earl Reynolds, Senior NBI Agent of
Davao, proceeded to the Velasco's room at the Skyroom Hotel in Davao City, to ask for said
document. Velasco was not inside the hotel room when they entered the room.

There are conficting claims whether the manicurist Teofila Ibaez or whether Velasco's wife,
who was allegedly inside the room at that time, voluntarily allowed the police officers to enter;
and whether the police officers "forcibly opened luggages and boxes from which only several
documents and papers were found, then seized, confiscated and took away the same," or whether
Mrs. Velasco volunteered to open the suitcases and baggages of Velasco and delivered the
documents and things contained therein to Reynolds. The Collector of Customs of Davao seized
1,480 sacks of copra and 86 sacks of coffee from the M/V motor vessel Jolo Lema.

The seizure was declared lawful by the Court of Tax Appeals, and its decision was affirmed by
the Supreme Court on 29 November 1974 in Nasiad vs. Court of Tax Appeals (GR L-29318,
November 29, 1974, 61 SCRA 238). In the present special civil action for certiorari, prohibition
and mandamus; the only question left then is whether the search conducted by a party headed by
Reynolds without the search warrant for the hotel room of Velasco, who entered into a contract
with Jose G. Lopez, the awardee of such Philippine Reparations Commission vessel, for its
operation and use ostensibly for fishing, is violative of such constitutional provision.

Issue: Whether there was consent on the part of the person who was the occupant of the hotel
room then rented by Velasco.

Held:
There was an attempt on the part of Lopez and Velasco to counteract the force of the recital of
the written statement of Teofila Ibaez (allegedly wife of Tomas Velasco) by an affidavit of one
Corazon Y. Velasco, who stated that she is the legal wife of Velasco, and another by Velasco
himself; reiterating that the person who was present at his hotel room was one Teofila Ibaez, "a
manicurist by occupation."

If such indeed were the case, then it is much more easily understandable why that person, Teofila
Ibaez, who could be aptly described as the wrong person at the wrong place and at the wrong
time, would have signified her consent readily and immediately.

Under the circumstances, that was the most prudent course of action. It would save her and even
Velasco himself from any gossip or innuendo. Nor could the officers of the law be blamed if they
would act on the appearances. There was a person inside who from all indications was ready to
accede to their request. Even common courtesy alone would have precluded them from inquiring
too closely as to why she was there. Under all the circumstances, therefore, it can readily be
concluded that there was consent sufficient in law to dispense with the need for a search warrant.

The petition cannot, therefore, prevail.

6. Stop and Frisk

G.R. No. 89139 August 2, 1990

ROMEO POSADAS y ZAMORA, petitioner,

vs.

THE HONORABLE COURT OF APPEALS and THE PEOPLE OF THE PHILIPPINES,


respondents.

Facts:

1. Pat. Ursicio Ungab and Pat. Umbra Umpar, both members of the Integrated National Police
(INP) of the Davao Metrodiscom assigned with the Intelligence Task Force, were conducting a
surveillance along Magallanes Street, Davao City. While they were within the premises of the
Rizal Memorial Colleges they spotted petitioner Posadas (below 18 y.o) carrying a "buri" bag
and they noticed him to be acting suspiciously.

2. They then checked the "buri" bag of the petitioner where they found one (1) caliber .38 Smith
& Wesson revolver with Serial No. 770196 1 two (2) rounds of live ammunition for a .38 caliber
gun 2 a smoke (tear gas) grenade, 3 and two (2) live ammunitions for a .22 caliber gun.

3. He was then charged with illegal possession of firearms and ammunitions in the Regional
Trial Court of Davao City because of his failure to secure the necessary documents.

Held:

The assailed search and seizure may still be justified as akin to a "stop and frisk" situation whose
object is either to determine the identity of a suspicious individual or to maintain the status quo
momentarily while the police officer seeks to obtain more information

Clearly, the search in the case at bar, the constitutional guarantee against unreasonable searches
and seizures has not been violated.

[G.R. No. 113447. October 9, 1997]


ALAIN MANALILI y DIZON, petitioner, vs. COURT OF APPEALS and PEOPLE OF THE
PHILIPPINES, respondents.

Facts:

1. Petitioner Alain Manalili y Dizon was charged by Assistant Caloocan City Fiscal E. Juan R.
Bautista with violation of Section 8, Article II of Republic Act No. 6425, wherein crushed
marijuana residue was found in his possession.

2. At about 2:10 oclock in the afternoon of April 11, 1988, as the policemen ( Pat. Romeo
Espiritu and Pat. Anger Lumabas) from the Anti-Narcotics Unit of the Kalookan City Police
Station were conducting a surveillance along A. Mabini street, Kalookan City, in front of the
Kalookan City Cemetery. They then chanced upon a male person in front of the cemetery who
appeared high on drugs. The male person was observed to have reddish eyes and to be walking in
a swaying manner. When this male person tried to avoid the policemen, the latter approached
him and introduced themselves as police officers. The policemen then asked the male person
what he was holding in his hands. The male person tried to resist. Pat. Romeo Espiritu asked the
male person if he could see what said male person had in his hands.The latter showed the wallet
and allowed Pat. Romeo Espiritu to examine the same. Pat. Espiritu took the wallet and
examined it. He found suspected crushed marijuana residue inside. He kept the wallet and its
marijuana contents.

Issue: On the Admissibility of the Evidence Seized During a Stop-and-Frisk

Held:

In the case at hand, Patrolman Espiritu and his companions observed during their surveillance
that appellant had red eyes and was wobbling like a drunk along the Caloocan City Cemetery,
which according to police information was a popular hangout of drug addicts. From his
experience as a member of the Anti-Narcotics Unit of the Caloocan City Police, such suspicious
behavior was characteristic of drug addicts who were high. The policemen therefore had
sufficient reason to stop petitioner to investigate if he was actually high on drugs .During such
investigation, they found marijuana in petitioners possession

The search was valid, being akin to a stop-and-frisk. In the landmark case of Terry vs. Ohio a
stop-and-frisk was defined as the vernacular designation of the right of a police officer to stop a
citizen on the street, interrogate him, and pat him for weapon(s):

x x x (W)here a police officer observes an unusual conduct which leads him reasonably to
conclude in light of his experience that criminal activity may be afoot and that the persons with
whom he is dealing may be armed and presently dangerous, where in the course of investigating
this behavior he identified himself as a policeman and makes reasonable inquiries, and where
nothing in the initial stages of the encounter serves to dispel his reasonable fear for his own or
others safety, he is entitled for the protection of himself and others in the area to conduct a
carefully limited search of the outer clothing of such persons in an attempt to discover weapons
which might be used to assault him. Such a search is a reasonable search under the Fourth
Amendment, and any weapon seized may properly be introduced in evidence against the person
from whom they were taken

In Philippine jurisprudence, the general rule is that a search and seizure must be validated by a
previously secured judicial warrant; otherwise, such search and seizure is unconstitutional and
subject to challenge

In the case of People vs. Lacerna five recognized exceptions to the rule against warrantless
search and seizure are enumerated;

(1) search incidental to a lawful arrest,

(2) search of moving vehicles,

(3) seizure in plain view,

(4) customs search, and

(5) waiver by the accused themselves of their right agains

MALACAT y MANDAR vs HON. COURT OF APPEALS

Facts:

In response to bomb threats reported one week before the incident of arrest of the
appellant, units of the Western Police District of Manila patrolled the streets of Quiapo, Manila,
near the Mercury Drugstore in Plaza Miranda. They chanced upon two groups of Muslim-
looking men, with each group, comprised of three to four men, posted at opposite sides of the
corner of Quezon Boulevard near the Mercury Drug Store. These men were acting suspiciously
with "[t]their eyes . . . moving very fast."

Alarmed, the police officers approached one of them, who then fired in different directions. The
policemen chased the fleeing men, but caught up with them and apprehended them. Upon
searching the petitioners, they found a frag grenade and a .38 Caliber Smith and Wesson
revolver.
During trial, petitioner challenged the validity of the search conducted upon his person, but the
trial court upheld the search to be valid. According to the court, the search is valid under stop and
frisk.

Issue:

Was the search conducted by elements of the Western Police District of Manila valid under stop
and frisk?

Held:

No. The Supreme Court said that they are not justified under stop and frisk. "Stop-and-frisk", as
laid down in US v Terry, is a limited protective search of outer clothing for weapons. Here,
petitioners allegedly found the grenade on his waistline, which is not outer clothing. The case of
Terry also holds that mere hunch or suspicion is insufficient to warrant stop and frisk
searches. A genuine reason must exist, in light of the police officer's experience and surrounding
conditions, to warrant the belief that the person detained has weapons concealed about him.
Finally, a "stop-and-frisk" serves a two-fold interest: (1) the general interest of effective crime
prevention and detection, which underlies the recognition that a police officer may, under
appropriate circumstances and in an appropriate manner, approach a person for purposes of
investigating possible criminal behavior even without probable cause; and (2) the more pressing
interest of safety and self-preservation which permit the police officer to take steps to assure
himself that the person with whom he deals is not armed with a deadly weapon that could
unexpectedly and fatally be used against the police officer.

There was nothing in petitioner's behavior or conduct which could have reasonably elicited even
mere suspicion other than that his eyes were "moving very fast" an observation which leaves
us incredulous since Yu and his teammates were nowhere near petitioner and it was already 6:30
p.m., thus presumably dusk. Petitioner and his companions were merely standing at the corner
and were not creating any commotion or trouble. Nor was there a valid ground to suspect that
petitioner carried a weapon. The item was not visible nor was there a bulge that can be seen
because the grenade was inside the petitioners waistline.

GUAZON vs DE VILLA

Facts:

The petition was filed to stop the police from conducting saturation drives or area target zonings
along Metro Manila.
(saturation patrol is a police or military patrol tactic wherein a large number of officers are
concentrated into a small geographic area. Saturation patrols are used for hot-spot crime
reduction, DUI checkpoints, and other location-specific patrols)

The drives were conducted in critical areas pinpointed by the military and police as places
where the subversives are hiding. The police were able to arrest a number of people in
Bangkusay, Tondo and at Lower Maricaban, Pasay. The petitioners claim that the drives follow
a common pattern of human rights abuse:

"1. Having no specific target house in mind, in the dead of the night or early morning hours,
police and military units without any search warrant or warrant of arrest cordon an area of more
than one residence and sometimes whole barangay or areas of barangay in Metro Manila. Most
of them are in civilian clothes and without nameplates or identification cards.

"2. These raiders rudely rouse residents form their sleep by banging on the walls and windows of
their homes, shouting, kicking their doors open (destroying some in the process), and then
ordering the residents within to come out of their respective residences.

"3. The residents at the point of high-powered guns are herded like cows, the men are ordered to
strip down to their briefs and examined for tattoo marks and other imagined marks.

"4. While the examination of the bodies of the men are being conducted by the raiders, some of
the members of the raiding team force their way into each and every house within the cordoned
off area and then proceed to conduct search of the said houses without civilian witnesses from
the neighborhood.

"5. In many instances, many residents have complained that the raiders ransack their homes,
tossing about the residents' belongings without total regard for their value. In several instances,
walls are destroyed, ceilings are damaged in the raiders' illegal effort to 'fish' for incriminating
evidence.

"6. Some victims of these illegal operations have complained with increasing frequency that their
money and valuables have disappeared after the said operations.

"7. All men and some women who respond to these illegal and unwelcome intrusions are
arrested on the spot and hauled off to waiting vehicles that take them to detention centers where
they are any warrants of arrest duly issued by a judge, nor under the conditions that will
authorize warrantless arrest. Some hooded men are used to fingerpoint suspected subversives.

"8. In some instances, arrested persons are released after the expiration of the period wherein
they can be legally detained without any charge at all. In other instances, some arrested persons
are released without charge after a few days of arbitrary detention.
"9. The raiders almost always brandish their weapons and point them at the residents during
these illegal operations.

"10. Many have also reported incidents of 'on-the-spot beatings', maulings and maltreatment.

"11. Those who are detained for further 'verification' by the raiders are subjected to mental and
physical torture to extract confessions and tactical information."

In their defense, public respondents cite the Commander-in-Chief provision of Section 18,
Article VII, and the good faith provision of Section 17, Article VII.

Issue:

Are the searches and arrests conducted by virtue of the saturation drives valid?

Held:

No. The Supreme Court said, There is, of course, nothing in the Constitution which denies the
authority of the Chief Executive, invoked by the Solicitor General, to order police actions to stop
unabated criminality, rising lawlessness, and alarming communist activities. The Constitution
grants to Government the power to seek and cripple subversive movements which would bring
down constituted authority and substitute a regime where individual liberties are suppressed as a
matter of policy in the name of security of the State. However, all police actions are governed by
the limitations of the Bill of Rights. The Government cannot adopt the same reprehensible
methods of authoritarian systems both of the right and of the left, the enlargement of whose
spheres of influence it is trying hard to suppress. Our democratic institutions may still be fragile
but they are not in the least bit strengthened through violations of the constitutional protections
which are their distinguishing features.

Furthermore, they cited Roan vs Gonzales, and 20th Century Fox Film Corporation vs CA,
among others, which reiterated and stressed the importance of the right of a person against
wanton and unreasonable searches upon their persons, papers, and effects.

The cases were remanded to the trial court for the determination for the prosecution of
the violations to the peoples constitutional right.

SAMULDE v SALVANI 165 SCRA 734 GRIO-AQUINO:

September 26, 1988

Facts:
Municipal Judge Samulde conducted a preliminary investigation upon a complaint for robbery
allegedly stemming from a boundary dispute between the parties.. After making a preliminary
investigation based on the affidavits of the complainant and her witnesses and counter-affidavits
of the respondent and his witnesses, Judge Samulde transmitted the records of the case to
Provincial Fiscal Salvani with his finding that "there is prima facie evidence of robbery as charge
in the complaint". The fiscal returned the records on the ground that Judge Samulde failed to
include the warrant of arrest against the accused as provided in Sec 5, Rule 112 of the 1985
Rules on Criminal Procedure. Judge Samulde sent back the records to the fiscal, pointing out that
under Sec 6, Rule 112, he may issue a warrant of arrest if he is satisfied "that a probable cause
exists and that there is a necessity of placing the respondent under immediate custody in order
not to frustrate the ends of justice, " implying that, although he found that a probable cause
existed, he did not believe that the accused should be immediately placed under custody.

Provincial Fiscal Salvani filed an action for mandamus to compel the Judge to issue a warrant of
arrest. The RTC ordered Judge Samulde to issue a warrant of arrest, and to transmit the warrant
to the Provincial Fiscal for appropriate action. He further advised the Municipal Judge "that
henceforth he adheres to the same rule in similar cases where he conducts a preliminary
investigation with a finding of a prima facie or probable cause."

Still unconvinced, Judge Samulde appealed the decision to this Court.

Issue:

Whether a judge may be compelled to issue a warrant of arrest upon a finding of probable cause
by the fiscal.

Held:

No. In order that a warrant of arrest can be issued, 3 conditions must first concur. The
investigating judge must: (a) have examined in writing and under oath the complainant and his
witnesses by searching questions and answers; (b) be satisfied that a probable cause exists; and
(c) that there is a need to place the respondent under immediate custody in order not to frustrate
the ends of justice.

It is not obligatory, but merely discretionary, upon the investigating judge to issue a warrant for
the arrest of the accused, for the determination of whether a probable cause exists and whether it
is necessary to arrest the accused in order not to frustrate the ends of justice, is left to his sound
judgment or discretion. In this particular case, since the robbery charge was the offshoot of a
boundary dispute between two property owners, the investigating judge did not believe there was
any danger of the accused absconding before the filing of the information against him by the
fiscal, hence, he found no need to place him under immediate custody.

DR. NEMESIO E. PRUDENTE, petitioner.

vs. THE HON. EXECUTIVE JUDGE ABELARDO M. DAYRIT, RTC Manila, Branch 33
and PEOPLE OF THE PHILIPPINES, respondents.

Facts:

P/Major Alladin Dimagmaliw, Chief of the Intelligence Special Action Division (ISAD) of the
Western Police District (WPD) filed with the RTC of Manila, Branch 33, presided over by Judge
Abelardo Dayrit (now Associate Justice of the Court of Appeals) an application for the issuance
of a search warrant for VIOLATION OF PD NO. 1866 (Illegal Possession of Firearms, etc.).

In support of the application for issuance of search warrant, P/Lt. Florenio C. Angeles, executed
a "Deposition of Witness." (Questions asked were: Do you know P/Major Alladin Dimagmaliw,
the applicant for a Search Warrant? Do you know the premises of Polytechnic University of the
Philippines at Anonas St., Sta. Mesa, Sampaloc, Manila? Do you have personal knowledge that
in the said premises is kept the following properties subject of the offense of violation of PD No.
1866 or intended to be used as a means of committing an offense? Do you know who is or who
are the person or persons who has or have control of the above-described premises?)

After this, the Judge issued the Search Warrant. On a Sunday and All Saints Day, the search
warrant was enforced by some 200 WPD operatives.

In Dr. Prudente's office was found a bulging brown envelope with three (3) live fragmentation
hand grenades separately wrapped with old newspapers.

Dr. Prudente moved to quash the search warrant. He claimed that

(1) the complainant's lone witness, Lt. Florenio C. Angeles, had no personal knowledge of the
facts which formed the basis for the issuance of the search warrant;

(2) the examination of the said witness was not in the form of searching questions and
answers;

(3) the search warrant was a general warrant, for the reason that it did not particularly describe
the place to be searched and that it failed to charge one specific offense; and
(4) the search warrant was issued in violation of Circular No. 19 of the Supreme Court in that the
complainant failed to allege under oath that the issuance of the search warrant on a Saturday was
urgent.

Issue: Whether the examination conducted by the judge before issuing the warrant was in the
form of searching questions and answers?

Held: No. The search warrants are annulled.

(1-2) For a valid search warrant to issue, there must be probable cause, which is to be determined
personally by the judge, after examination under oath or affirmation of the complainant and the
witnesses he may produce, and particularly describing the place to be searched and the persons
or things to be seized. The probable cause must be in connection with one specific offense and
the judge must, before issuing the warrant, personally examine in the form of searching questions
and answers, in writing and under oath, the complainant and any witness he may produce, on
facts personally known to them and attach to the record their sworn statements together with any
affidavits submitted.

While it is true that in his application for search warrant, applicant P/Major Dimagmaliw stated
that he verified the information he had earlier received that petitioner had in his possession and
custody there is nothing in the record to show or indicate how and when said applicant verified
the earlier information acquired by him as to justify his conclusion that he found such
information to be a fact. This point might have been clarified if there had been searching
questions and answers, but there were none. In fact, the records yield no questions and
answers, whether searching or not, vis-a-vis the said applicant.

What the records show is the deposition of witness, P/Lt. Angeles, as the only support to P/Major
Dimagmaliw's application, and the said deposition is based on hearsay. For, it avers that they
(presumably, the police authorities) had conducted continuous surveillance for several days of
the suspected premises and, as a result, they "gathered information from verified sources" that
the holders of the subject firearms and explosives are not licensed to possess them.

An examination of the deposition of P/Lt. Florenio Angeles shows that it was too brief and
short. The Judge did not examine him "in the form of searching questions and answers."
On the contrary, the questions asked were leading as they called for a simple "yes" or "no"
answer.

In the case at bar, the evidence failed to show the existence of probable cause to justify the
issuance of the search warrant.

The requirements are stringent but the purpose is to assure that the constitutional right of the
individual against unreasonable search and seizure shall remain both meaningful and effective.
(3) The rule is that a description of a place to be searched is sufficient if the officer with the
warrant can, with reasonable effort, ascertain and Identify the place intended. In the case at bar,
the application for search warrant and the search warrant itself described the place to be searched
as the premises of the Polytechnic University of the Philippines, located at Anonas St., Sta.
Mesa, Sampaloc, Manila more particularly, the offices of the Department of Military Science and
Tactics at the ground floor, and the Office of the President, Dr. Nemesio Prudente, at PUP,
Second Floor and other rooms at the second floor. The designation of the places to be searched
sufficiently complied with the constitutional injunction that a search warrant must particularly
describe the place to be searched, even if there were several rooms at the ground floor and
second floor of the PUP.

(4) As to the urgency for the issuance of the search warrant: Circular No. 19 reads:

Applications filed after office hours, during Saturdays, Sundays and holidays shall likewise be
taken cognizance of and acted upon by any judge of the court having jurisdiction of the place to
be searched, but in such cases the applicant shall certify and state the facts under oath, to the
satisfaction of the judge, that the issuance is urgent.

The circular merely provides for a guideline and departure from it would not necessarily affect
the validity of an otherwise valid search warrant.

Alvarez v CFI

Facts: On 3 June 1936, the chief of the secret service of the Anti-Usury Board, of the
Department of Justice, presented to Judge Eduardo Gutierrez David then presiding over the
Court of First Instance of Tayabas, an affidavit alleging that according to reliable information,
Narciso Alvarez kept in his house in Infanta, Tayabas, books, documents, receipts, lists, chits and
other papers used by him in connection with his activities as a moneylender, charging usurious
rates of interest in violation of the law.

In his oath at the end of the affidavit, the chief of the secret service stated that his answers to the
questions were correct to the best of his knowledge and belief. He did not swear to the truth of
his statements upon his own knowledge of the facts but upon the information received by him
from a reliable person.

Upon the affidavit the judge, on said date, issued the warrant which is the subject matter of the
petition, ordering the search of the Alvarezs house at any time of the day or night, the seizure of
the books and documents and the immediate delivery thereof to him to be disposed of in
accordance with the law. With said warrant, several agents of the Anti-Usury Board entered
Alvarezs store and residence at 7:00 p.m. of 4 June 1936, and seized and took possession of the
following articles: internal revenue licenses for the years 1933 to 1936, 1 ledger, 2 journals, 2
cashbooks, 9 order books, 4 notebooks, 4 check stubs, 2 memorandums, 3 bankbooks, 2
contracts, 4 stubs, 48 stubs of purchases of copra, 2 inventories, 2 bundles of bills of lading, 1
bundle of credit receipts, 1 bundle of stubs of purchases of copra, 2 packages of correspondence,
1 receipt book belonging to Luis Fernandez, 14 bundles of invoices and other papers, many
documents and loan contracts with security and promissory notes, 504 chits, promissory notes
and stubs of used checks of the Hongkong & Shanghai Banking Corporation (HSBC). The
search for and seizure of said articles were made with the opposition of Alvarez who stated his
protest below the inventories on the ground that the agents seized even the originals of the
documents. As the articles had not been brought immediately to the judge who issued the search
warrant, Alvarez, through his attorney, filed a motion on 8 June 1936, praying that the agent
Emilio L. Siongco, or any other agent, be ordered immediately to deposit all the seized articles in
the office of the clerk of court and that said agent be declared guilty of contempt for having
disobeyed the order of the court. On said date the court issued an order directing Siongco to
deposit all the articles seized within 24 hours from the receipt of notice thereof and giving him a
period of 5 days within which to show cause why he should not be punished for contempt of
court. On 10 June, Attorney Arsenio Rodriguez, representing the Anti-Usury Board, filed a
motion praying that the order of the 8th of said month be set aside and that the Anti-Usury Board
be authorized to retain the articles seized for a period of 30 days for the necessary investigation.
On June 25, the court issued an order requiring agent Siongco forthwith to file the search warrant
and the affidavit in the court, together with the proceedings taken by him, and to present an
inventory duly verified by oath of all the articles seized.

On July 2, the attorney for the petitioner filed a petition alleging that the search warrant issued
was illegal and that it had not yet been returned to date together with the proceedings taken in
connection therewith, and praying that said warrant be cancelled, that an order be issued
directing the return of all the articles seized to Alvarez, that the agent who seized them be
declared guilty of contempt of court, and that charges be filed against him for abuse of authority.

On September 10, the court issued an order holding: that the search warrant was obtained and
issued in accordance with the law, that it had been duly complied with and, consequently, should
not be cancelled, and that agent Siongco did not commit any contempt of court and must,
therefore, be exonerated, and ordering the chief of the Anti-Usury Board in Manila to show
cause, if any, within the unextendible period of 2 days from the date of notice of said order, why
all the articles seized appearing in the inventory should not be returned to Alvarez.

The assistant chief of the Anti-Usury Board of the Department of Justice filed a motion praying,
for the reasons stated therein, that the articles seized be ordered retained for the purpose of
conducting an investigation of the violation of the Anti-Usury Law committed by Alvarez. On
October 10, said official again filed another motion alleging that he needed 60 days to examine
the documents and papers seized, which are designated on pages 1 to 4 of the inventory by Nos.
5, 10, 16, 23, 25-27, 30-31 , 34, 36-43 and 45, and praying that he be granted said period of 60
days. In an order of October 16, the court granted him the period of 60 days to investigate said
19 documents. Alvarez, herein, asks that the search warrant as well as the order authorizing the
agents of the Anti-Usury Board to retain the articles seized, be declared illegal and set aside, and
prays that all the articles in question be returned to him.

Issue: Whether the search warrant issued by the court is illegal because it has been based upon
the affidavit of agent Almeda in whose oath he declared that he had no personal knowledge of
the facts which were to serve as a basis for the issuance of the warrant but that he had knowledge
thereof through mere information secured from a person whom he considered reliable, and that it
is illegal as it was not supported by other affidavits aside from that made by the applicant.

Held: Section 1, paragraph 3, of Article III of the Constitution and Section 97 of General Orders
58 require that there be not only probable cause before the issuance of a search warrant but that
the search warrant must be based upon an application supported by oath of the applicant and the
witnesses he may produce. In its broadest sense, an oath includes any form of attestation by
which a party signifies that he is bound in conscience to perform an act faithfully and truthfully;
and it is sometimes defined as an outward pledge given by the person taking it that his attestation
or promise is made under an immediate sense of his responsibility to God.

The oath required must refer to the truth of the facts within the personal knowledge of the
petitioner or his witnesses, because the purpose thereof is to convince the committing magistrate,
not the individual making the affidavit and seeking the issuance of the warrant, of the existence
of probable cause.

The true test of sufficiency of an affidavit to warrant issuance of a search warrant is whether it
has been drawn in such a manner that perjury could be charged thereon and affiant be held liable
for damages caused. The affidavit, which served as the exclusive basis of the search warrant, is
insufficient and fatally defective by reason of the manner in which the oath was made, and
therefore, the search warrant and the subsequent seizure of the books, documents and other
papers are illegal. Further, it is the practice in this jurisdiction to attach the affidavit of at least
the applicant or complainant to the application. It is admitted that the judge who issued the
search warrant in this case, relied exclusively upon the affidavit made by agent Almeda and that
he did not require nor take the deposition of any other witness. Neither the Constitution nor
General Orders 58 provides that it is of imperative necessity to take the depositions of the
witnesses to be presented by the applicant or complainant in addition to the affidavit of the latter.

The purpose of both in requiring the presentation of depositions is nothing more than to satisfy
the committing magistrate of the existence of probable cause. Therefore, if the affidavit of the
applicant or complainant is sufficient, the judge may dispense with that of other witnesses.
Inasmuch as the affidavit of the agent was insufficient because his knowledge of the facts was
not personal but merely hearsay, it is the duty of the judge to require the affidavit of one or more
witnesses for the purpose of determining the existence of probable cause to warrant the issuance
of the search warrant. When the affidavit of the applicant or complainant contains sufficient facts
within his personal and direct knowledge, it is sufficient if the judge is satisfied that there exists
probable cause; when the applicants knowledge of the facts is mere hearsay, the affidavit of one
or more witnesses having a personal knowledge of the facts is necessary. Thus the warrant issued
is likewise illegal because it was based only on the affidavit of the agent who had no personal
knowledge of the facts.

Luna vs Plaza

Facts:

This is an appeal from the decision of the Court of First Instance of Surigao del Sur,
dated April 20, 1967, dismissing the petition for a writ of habeas corpus, filed by herein
petitioner-appellant Simon Luna who was charged with murder in Criminal Case No. 655-New
of the same court.

The criminal action was commenced by T-Sgt. Candido Patosa, PC investigator of


Tandag, Surigao del Sur, by filing with respondent Municipal Judge Lorenzo M. Plaza, of the
Municipal Court of Tandag, criminal case No. 1138 charging the accused, herein petitioner, with
the crime of murder.

The respondent Judge examined the prosecution witnesses by reading to them "all over again the
questions and answers" in their statements in writing, and the witnesses-affiants declared before
said Judge that the questions were propounded by T-Sgt. Candido Patosa, and that the answers
were made by them. The affiants further declared before respondent Judge that their answers
were true, and were freely and voluntarily made; that they fully understood the questions and
answers, and that they were willing to sign their respective affidavits. The affiants signed their
respective affidavits in the presence of the respondent Judge, who also signed after the usual
procedure of administering the oath.

The respondent Judge opined that there was reasonable ground to believe that the crime of
murder had been committed and the accused was probably guilty thereof. Respondent Judge
issued the order and warrant of arrest, specifying therein that no bail should be accepted for the
provisional release of the accused.

On February 20, 1967, upon motion of petitioner that he be admitted to bail upon the ground that
the evidence of guilt was not strong, respondent Judge issued an order granting bail, fixing it at
P30,000.00; which order, however, respondent Judge later revoked, and petitioner was denied
bail.

Contention of the accused:

On April 5, 1967, petitioner filed a petition for a writ of habeas corpus with the Court of First
Instance of Surigao del Sur, therein docketed as Special Proceedings No. 105-New, claiming that
he was being deprived of liberty without the due process of law, on the ground that the
imprisonment and detention was the result of a warrant of arrest issued by respondent Judge in
violation of Republic Act No. 3828, and praying for the annulment of the order for his arrest and
his discharge from confinement.

Contention of the respondents:

Respondents filed their answer, alleging that Republic Act No. 3828 had been substantially
complied with; that a motion to quash, and not a petition for habeas corpus was the proper
remedy; and that petitioner's application for bail constituted a waiver of the right to question the
validity of the arrest.

Issue:

Petitioner, in his assignment of errors, claims that the trial court erred, as follows:

1. In giving absolute credence to the oral testimony of the respondent Judge to the effect that he
adopted and made his own the questions and answers taken by T-Sgt. Patosa, PC Investigator,
one of the prosecution witnesses, because the records show the contrary;

2. In denying the writ of habeas corpus and in dismissing the petition.

Before a municipal judge may issue a warrant of arrest, the following conditions must first be
fulfilled:

(1) he must examine the witnesses personally; (2) the examination must be under oath; (3) the
examination must be reduced to writing in the form of searching questions and answers.

Were these conditions fulfilled in the instant case?

Held:

The first condition was fulfilled. The trial court found as a fact that "the respondent judge
personally examined the witnesses for the prosecution ...;" that respondent judge adopted as his
own personal examination the questions asked by T-Sgt. Patosa as appearing in the written
statements, which he read over again to the witnesses together with the answers given therein,
asking the witnesses whether said answers were theirs, and whether the same answers were true,
to which the witness answered in the affirmative. Republic Act No. 3828 does not prohibit the
municipal Judge from adopting the questions asked by a previous investigator.

It appears that the sworn statements5 of the witnesses state at the beginning that the sworn
statement was "taken by T-Sgt. Candido L. Patosa", and does not state that it was taken by the
respondent municipal Judge himself. This circumstance is explained by the fact that said written
statements already taken by T-Sgt. Patosa were delivered to respondent Municipal Judge who
adopted the questions therein in his examination, because he considered them searching
questions. Respondent Judge presumably did not consider it necessary to change the introductory
remarks in each of the written statements. But that he made the examination personally cannot be
doubted; it is so stated in the order dated February 18, 1967, which recites:

After examining the witness personally and under oath there is reasonable ground to believe that
an offense for murder has been committed and that the accused, Simon Luna, is probably guilty
thereof. (Exh. H)

The second condition required by Republic Act No. 3828 for the issuance of a warrant of arrest
was also fulfilled. The trial court found that the complaint was "supported by statements of the
witnesses under oath." The record also shows the documents to have been subscribed and sworn
to before respondent Judge.

The third condition required by Republic Act No. 3828 was likewise fulfilled. The examination
of the witnesses was written down, in the form of searching questions and answers.

The term "searching questions and answers" means only, taking into consideration the purpose of
the preliminary examination which is to determine "whether there is a reasonable ground to
believe that an offense has been committed and the accused is probably guilty thereof so that a
warrant of arrest may be issued and the accused held for trial", such questions as have tendency
to show the commission of a crime and the perpetrator thereof.

What would be searching questions would depend on what is sought to be inquired into, such as:
the nature of the offense, the date, time, and place of its commission, the possible motives for its
commission; the subject, his age, education, status, financial and social circumstances, his
attitude toward the investigation, social attitudes, opportunities to commit the offense; the victim,
his age, status, family responsibilities, financial and social circumstances, characteristics, etc.

The points that are the subject of inquiry may differ from case to case. The questions, therefore,
must to a great degree depend upon the Judge making the investigation. At any rate, the court a
quo found that respondent judge was "satisfied that the questions and answers contained in the
sworn statements taken by T-Sgt. Patosa partake of the nature of his searching questions and
answers as required by law," so the respondent Judge adopted them.

Petitioner's further contention that the issuance of the warrant of arrest was a violation of the
constitution and of procedural due process is likewise untenable. The Constitution, in Section
1(3), Article III, provides that no warrant shall issue but upon probable cause, to be determined
by the judge after examination under oath or affirmation of the complainant and the witnesses he
may produce. The constitutional requirement of examination of witnesses under oath was, as
shown above, fulfilled. The existence of probable cause depended to a large degree upon the
finding or opinion of the judge conducting the examination. Respondent judge found that there
was probable cause, as stated in his order of arrest, that "after examining the witnesses personally
and under oath there is a reasonable ground to believe that an offense of murder has been
committed and that the accused, Simon Luna, is probably guilty thereof."

Petitioner's last contention that the warrant of arrest issued was a violation of procedural due
process because of the alleged defective preliminary examination has no leg to stand on, in view
of what we have hereinbefore stated. Moreover, this Court has held that preliminary examination
is not an essential part of due process of law. Preliminary examination may be conducted by the
municipal judge, prior to the issuance of the warrant of arrest, either in the presence, or in the
absence, of the accused. The record shows that herein petitioner waived the preliminary
investigation before respondent Municipal Judge, and instead, he filed a petition for bail. The
petition for bail was at first granted by respondent Judge, but later the order granting bail was
revoked. This conduct of petitioner indicates that he had waived his objection to whatever defect,
if any, in the preliminary examination conducted by respondent Judge prior to the issuance of the
warrant of arrest. Indeed, petitioner has no substantial much less legal ground to complain
that he was denied the due process of law.

Soledad Avillar de Mulata vs Judge Elias C. Irizari

Facts:

Soledad Avillar filed an administrative case against Judge Irizari with grave abuse of discretion,
favoritism and bias for having conducted he preliminary examination in said petitioners criminal
case without giving the accused therein an opportunity to be heard because they were granted
the occasion to cross-examine the complainant and her witness before issuing the warrant
for their arrest. The accused was arrested and detained in jail for 15 days without knowing why
they were arrested as neither of them were shown any warrant of arrest.

Issue:

Whether or not the accused is entitled to be present during the preliminary examination or to
cross-examine the witnesses presented against him before his arrest.

Ruling:

The Supreme Court ruled that under Section 5, Rule 112, of the Revised Rules of Court, the
accused is not entitled as a matter of right to be present during the preliminary examination or to
cross-examine the witnesses presented against him before his arrest. The purpose of the
preliminary examination is to determine whether or not there is sufficient reason to issue a
warrant of arrest. Section 6 of Rule 112 of the Revised Rules of Court requires the warrant of
arrest shall be issued only when the judge conducting the preliminary examination is satisfied
that the offense has in fact been committed and that there is a reasonable ground to believe that
the accused committed the same. We are not satisfied that in the circumstances attendant to the
case, respondent, in dismissing the complaint for grave coercion, has acted arbitrarily.

WHEREFORE, respondent is exonerated from the charges.

Antonio Marinas vs Judge Andres Siochi

Facts:

Petitioner was the Sheriff in charged to execute the rented house of Victoria Lasin de Atienza
and Rosario L. Atiena. That upon the enforcement of said writ of execution, Lasins jewelry
worth 590 pesos was taken by petitioner without issuing any receipt therefor. On January 1966,
when Lasin re-entered the house after securing a Court Order, she discovered that several pieces
of her jewelry and other personal items worth 1,018 pesos were missing. On February 1966, she
entered the house to get her remaining unlevied properties. She claimed that on the same day,
Marinas forcibly compelled them to deliver the unlevied personal properties (amounting to 2,645
pesos) found therein, hauled said articles into a truck and left. Two separate charges of theft was
the filed against the petitioner. Respondent Judge Siochi issued warrants for the arrest of
petitioners. The preliminary examination conducted by him was done with the aid of Lt. Jose S.
Lotoc.

Issue:

Whether or not an arrest is valid without the presence of the accused during the preliminary
examination.

Ruling:

The Supreme Court ruled that under Section 5, Rule 112, of the Revised Rules of Court, the
accused is not entitled as a matter of right to be present during the preliminary examination or to
cross-examine the witnesses presented against him before his arrest. The purpose of the
preliminary examination is to determine whether or not there is sufficient reason to issue a
warrant of arrest. Section 6 of Rule 112 of the Revised Rules of Court requires the warrant of
arrest shall be issued only when the judge conducting the preliminary examination is satisfied
that the offense has in fact been committed and that there is a reasonable ground to believe that
the accused committed the same.

Section 87, paragraph 3 of the Judiciary Act provides that before a Municipal Judge may issue a
warrant of arrest, the following conditions must first be fulfilled: (1) he must examine the
witness or witnesses personally; (2) the examination must be under oath; and (3) the examination
must be reduced to writing in the form of searching questions and answers. These requirements
have been met in the three criminal cases involved herein. By "searching questions and answers"
is meant:
the term "searching questions and answers" means only, taking into consideration the purpose of
the preliminary examination which is to determine 'whether there is a reasonable ground to
believe that an offense has been committed and the accused is probably guilty thereof so that a
warrant of arrest may be issued and the accused held for trial', such questions as have tendency to
show the commission of a crime and the perpetrator thereof.

In a nutshell, the proceedings in these three criminal cases conformed to law and jurisprudence.
But even conceding that petitioners were entitled to a preliminary investigation, the proper forum
before which absence thereof should have been raised and ventilated was in the trial Court, not in
an appellate Court because the absence of preliminary investigation does not go to the
jurisdiction of the Court but merely to the regularity of the proceedings, and bearing in mind that
preliminary investigation can be waived, as in fact, it is frequently waived.

WHEREFORE, the Petition is hereby denied

Nolasco vs Pano, 139 SCRA 541 GR No. L-69803


Facts:
AT around 11:30 am on August 6, 1984, Aguilar-Roque and Nolaso were arrested by a
Constabulary Security Group (CSG) at the intersection of Mayon St. and P. Margall St., Q.C.
(Prior to August 6, 1984, Aguilar-Roque was one of the accused of rebellion in Criminal Case #
MC-25-113 of Military Comm. No 25 wherein she was still at large.)
At 12NN of Aug 6, 1984. Elements of the CSG searched the premises at 239-B Mayon St, Q.C.
wherein 428 documents and written materials and additionally a portable typewriter and 2
wooden boxes, making 431 items in all, were seized by CSG elements.
Prior the arrest done and the search done, at around 9am that same day, Lt. Col. Virgilio
Saldajeno of the CSJ applied for a search warrant from Hon. Ernani Cruz Pano, the Exec Judge
of the RTC in Q.C. to be served at 239-B Mayon St, QC because it was determined to be the
leased residence of Aguilar-Roque, after almost a month of round the clock surveillance of the
premises as a suspected underground house of the CPP/NPA. Aguilar-Roque has long been
wanted by the military for being a high ranking officer of the CPP, particularly connected with
the MV Karagatan/ Dona Andrea cases.
On Aug 10, 1984, Nolasco et al. were charged before the QC Fiscal Office upon complaint filed
by the CSG group for Subversion, Rebellion, and/or Conspiracy to Commit
Rebellion/Subversion. On Aug 13, 1984, an information for Violation of PD 33(Illegal
Possession of Subversive Documents) were filed against Nolasco et al. before the MTC of QC
which was presided by Judge Antonio P. Santos.
ON Dec 12, 1984, Nolasco et al. asked for the suppression of the evidence (431 items seized) on
the ground that it was illegally obtained and that the search warrant was void because it was a
general warrant since it does not sufficiently describe with particularity the things subject of the
search and seizure and that probable cause has not been established for lack of searching
questions.
Issue: W/N the search warrant was valid?
Held: NO. Sec 3, Art IV of the Constitution of the Constitution guarantees the right of the
people to be secure in their persons, houses, papers, and effects against unreasonable searches
and seizures of whatever nature and for any purpose. It also specifically provides that no Search
Warrant shall issue except upon probable cause to be determined by the Judge or such other
responsible officer as may be authorized by law, after examination under oath or affirmation of
the complainant and the witnesses he may produce, and particularly describing the place to be
searched and the things to be seized.
The disputed Search Warrant (No. 80-84) describes the personalities to be seized as follows:

Documents, papers and other records of the Communist Party of the Philippines/New Peoples
Army and/or the National Democratic Front, such as Minutes of the Party Meetings, Plans of
these groups, Programs, List of possible supporters, subversive books and instructions, manuals
not otherwise available to the public, and support money from foreign or local sources.

It is at once evident that the foregoing Search Warrant authorizes the seizure of personal
properties vaguely described and not particularized. It is an all- embracing description which
includes everything conceivable regarding the Communist Party of the Philippines and the
National Democratic Front. It does not specify what the subversive books and instructions are;
what the manuals not otherwise available to the public contain to make them subversive or to
enable them to be used for the crime of rebellion. There is absent a definite guideline to the
searching team as to what items might be lawfully seized thus giving the officers of the law
discretion regarding what articles they should seize as, in fact, taken also were a portable
typewriter and 2 wooden boxes. It is thus in the nature of a general warrant and infringes on the
constitutional mandate requiring particular description of the things to be seized. In the recent
rulings of this Court, search warrants of similar description were considered null and void for
being too general.
However, considering that Aguilar-Roque has been charged with the crime of Rebellion,
notwithstanding the irregular issuance of the search warrant and although ordinarily the articles
seized under an invalid search should be returned, they cannot be ordered returned in the case at
bar to Aguilar-Roque. The SC held that some searches may be made without warrant pursuant to
Sec 12 of the Rules of Court. It provides that a person charged with an offense maybe searched
for dangerous weapons or anything which may be used as proof of the commission of the
offense. The SC opines that with respect to Aguilar- Roque, the search at No 239-B Mayon, St.,
QC did not need a search warrant; this for possible effective results in the interest of public
order.
Such being the case, the personalities seized may be retained by the CSG for possible
introduction of evidence in the rebellion case leaving it to Aguilar-Roque to object to their
relevance.
7. Corro vs. Lising, 137 SCRA 541

Facts:

Respondent RTC Judge Esteban Lising, upon application filed by Lt. Col. Berlin Castillo of the
Philippine Constabulary Criminal Investigation Service, issued a search warrant authorizing the
search and seizure of articles allegedly used by Rommel Corro (Publisher and editor of the
Philippine Times) in committing the crime of inciting to sedition.

1. Printed copies of Philippine Times;

2. Manuscripts/drafts of articles for publication in the Philippine Times;

3. Newspaper dummies of the Philippine Times;

4. Subversive documents, articles, printed matters, handbills, leaflets, banners;

5. Typewriters, duplicating machines, mimeographing and tape recording machines, video


machines and tapes

Corro filed an urgent motion to recall warrant and to return documents/personal properties
alleging among others that (1) the properties seized are typewriters, duplicating machines,
mimeographing and tape recording machines, video machines and tapes which are not in any
way, inanimate or mute things as they are, connected with the offense of inciting to sedition, and
that (2) the documents/papers seized has been rendered moot and academic due to the findings of
the Agrava Board having exclusive jurisdiction to determine the facts and circumstances behind
the killing of Ninoy Aquino that a militaryconspiracy was responsible for Ninoy Aquinos
slaying.

Respondent Judge Lising denied the motion. Hence, this petition praying that the search warrant
issued by respondent Judge Esteban M. Lising be declared null and void ab initio that the
padlocked office premises of the Philippine Times be reopened.

Respondents would have this Court dismiss the petition stating that probable cause exists
justifying the issuance of a search warrant, the articles seized were adequately described in the
search warrant, a search was conducted I n an orderly manner and the padlocking of the searched
premises was with the consent of petitioner's wife.
Held:

Section 3, Article IV of the 1973 Constitution provides:

SEC. 3. ...no search warrant or warrant of arrest issue except upon probable cause to be
determined by the judge, or such other responsible officer as may be authorized by law, after
examination under oath or affirmation of the complainant and the witnesses he may produce, and
particularly describing the place to be searched and the persons or things to be seized.

and, Section 3, Rule 126 of the New Rules of Court, states that:

SEC. 3. Requisites for issuing search warrant. A search warrant shall not issue but upon
probable cause in connection with one specific offense to be determined by the judge or justice
of the peace after examination under oath or affirmation of the complainant and the witnesses he
may produce, and particularly describing the place to be searched and the persons or things to be
seized.

Probable cause may be defined as "such reasons, supported by facts and circumstances, as
will warrant a cautious man in the belief that his actions, and the means taken in
prosecuting it, are legally just and proper (Burton vs. St. Paul, M & M. Ry. Co., 33 Minn.
189, cited in U.S. vs. Addison, 28 Phil. 566)."

An application for search warrant must state with particularly the alleged subversive
materials published or intended to be published by the publisher and editor of the
Philippine Times, Rommel Corro. As We have stated in Burgos, Sr. vs. Chief of Staff of the
Armed Forces of the Philippines, 133 SCRA 800, "mere generalization will not suffice."

A search warrant should particularly describe the place to be searched and the things to be
seized. "The evident purpose and intent of this requirement is to limit the things to be seized to
those, and only those, particularly described in the search warrant- to leave the officers of the law
with no discretion regarding what articles they should seize, to the end that unreasonable
searches and seizures may not be committed, that abuses may not be committed.

The affidavit of Col. Castillo states that in several issues of the Philippine Times:
... we found that the said publication in fact foments distrust and hatred against the government
of the Philippines and its duly constituted authorities, defined and penalized by Article 142 of the
Revised Penal Code as amended by Presidential Decree No. 1835; (p. 22, Rollo)

and, the affidavit of Lt. Ignacio reads, among others

... the said periodical published by Rommel Corro, contains articles tending to incite distrust and
hatred for the Government of the Philippines or any of its duly constituted authorities. (p. 23,
Rollo)

The above statements are mere conclusions of law and will not satisfy the requirements of
probable cause. They cannot serve as basis for the issuance of search warrant, absent of the
existence of probable cause.

In the case at bar, the search warrant issued by respondent judge allowed seizure of printed
copies of the Philippine Times, manuscripts/drafts of articles for publication, newspaper
dummies, subversive documents, articles, etc., and even typewriters, duplicating machines,
mimeographing and tape recording machines.

Thus, the language used is so all embracing as to include all conceivable records and
equipment of petitioner regardless of whether they are legal or illegal. The search warrant
under consideration was in the nature of a general warrant which is constitutionally
objectionable.

PEOPLE OF THE PHILIPPINES, plaintiff-appellee,

vs. RUBEN BURGOS y TITO, defendant-appellant.

Facts: Cesar Masamlok surrendered himself to the PC stating that he was forcibly recruited by
accused Ruben Burgos as member of the NPA, threatening him with the use of firearm against
his life, if he refused. He was also asked to contribute one (1) chopa of rice and one peso (P1.00)
per month to the NPA TSN.

After this, a joint team of PC-INP units was dispatched at Tiguman; Davao del Sur, to arrest
Ruben Burgos. The team left the headquarter at 1:30 P.M., and arrived at Tiguman, at more or
less 2:00 o'clock PM where through the help of Pedro Burgos, brother of accused, the team was
able to locate accused, who was plowing his field.

Pat. Bioco asked accused about his firearm. At first, Burgos denied possession of said
firearm. Later, Sgt. Alejandro Buncalan questioned Burgos wife, she pointed to a place below
their house where a gun was buried in the ground. Aside from the firearm, subversive documents
which he allegedly kept in a stock pile of cogon near his house were also seized. (These included
pamphlets entitled Ang Bayan, Pahayagan ng Partido Komunista ng Pilipinas, Pinapatnubayan
ng Marxismo, Leninismo Kaisipang Mao Zedong and another titled Asdang Pamantalaang Masa
sa Habagatang Mindanao)

When confronted with the firearm Burgos readily admitted that it was issued to him by Nestor
Jimenez, Alias Pedipol, team leader of the sparrow unit of NPA. Ruben Burgos executed an extra
judicial confession with the assistance of Atty. Anyog (His office was adjacent to the fiscals
office). He was only assisted when he was about to make his confession. Atty. Anyog assisted
accused in the reading of his confession from English to Visayan language. Fiscal Lovitos,
before accused signed his statement, explained to him his constitutional rights to remain silent,
right to counsel and right to answer any question propounded or not.

The trial court justified the arrest of the accused-appelant without any warrant as falling under
one of the instances when arrests may be validly made without a warrant. The authorities
received an urgent report of accused's involvement in subversive activities from a reliable source
thus justifying the circumstances of his arrest, even without judicial warrant.

Issues: Was the arrest of Ruben Burgos lawful? Were the search of his house and the subsequent
confiscation of a firearm and documents allegedly found therein conducted in a lawful and valid
manner? Does the evidence sustaining the crime charged meet the test of proving guilt beyond
reasonable doubt?

Held: No, the TC is wrong. It was not lawful. The charge fails, he is acquitted.

Under Section 6(a) of Rule 113, the officer arresting a person who has just committed, is
committing, or is about to commit an offense must have personal knowledge of that fact. The
offense must also be committed in his presence or within his view. (Sayo v. Chief of Police, 80
Phil. 859). There is no such personal knowledge in this case. Whatever knowledge was
possessed by the arresting officers, it came in its entirety from the information furnished by
Cesar Masamlok. The location of the firearm was given by his wife. At the time of the
appellant's arrest, he was not in actual possession of any firearm or subversive document. Neither
was he committing any act which could be described as subversive. He was, in fact, plowing his
field at the time of the arrest.

In arrests without a warrant a crime must in fact or actually have been committed first. That a
crime has actually been committed is an essential precondition. It is not enough to suspect that a
crime may have been committed. The fact of the commission of the offense must be undisputed.
The test of reasonable ground applies only to the identity of the perpetrator.

In this case, Burgos was arrested on the sole basis of Masamlok's verbal report. Masamlok led
the authorities to suspect that the accused had committed a crime. They were still fishing for
evidence of a crime not yet ascertained. The subsequent recovery of the subject firearm on the
basis of information from the lips of a frightened wife cannot make the arrest lawful, If an arrest
without warrant is unlawful at the moment it is made, generally nothing that happened or is
discovered afterwards can make it lawful. The fruit of a poisoned tree is necessarily also tainted.

The arresting officers should have gone through the process of obtaining a warrant of arrest, if
indeed they had reasonable ground to believe that the accused had truly committed a crime.
There is no showing that there was a real apprehension that the accused was on the verge of
flight or escape. His whereabouts was also known.

Neither can it be presumed that there was a waiver, or that consent was given by the accused to
be searched simply because he failed to object. The fact that the accused failed to object to the
entry into his house does not amount to a permission to make a search therein (Magoncia v.
Palacio).

Considering that the questioned firearm and the alleged subversive documents were obtained in
violation of the accused's constitutional rights against unreasonable searches and seizures, it
follows that they are inadmissible as evidence.

Other aspect of the case: The accused was never informed of his constitutional rights at the time
of his arrest. So that when the accused allegedly admitted ownership of the gun and pointed to
the location of the subversive documents after questioning, the admissions were obtained in
violation of the constitutional right against self-incrimination under Sec. 20 of Art. IV of the Bill
of Rights. He was not accorded his constitutional right to be assisted by counsel during the
custodial interrogation. Although Atty. Anyog was provided the accused when he subscribed
under oath to his statement at the Fiscal's Office, it was too late. It could have no palliative
effect. It cannot cure the absence of counsel at the time of the custodial investigation when the
extrajudicial statement was being taken.

JOSE BURGOS, SR., JOSE BURGOS, JR., BAYANI SORIANO and J. BURGOS MEDIA
SERVICES, INC., petitioners,

vs. THE CHIEF OF STAFF, ARMED FORCES OF THE PHILIPPINES, THE CHIEF,
PHILIPPINE CONSTABULARY, THE CHIEF LEGAL OFFICER, PRESIDENTIAL
SECURITY COMMAND, THE JUDGE ADVOCATE GENERAL, ET AL., respondents.

(Read on history? Story of Burgos: http://cmfr-phil.org/media-ethics-


responsibility/ethics/heroes-of-press-freedomjose-burgos-jr-a-timeless-hero/)

Facts: Two search warrants were issued by Judge Ernani Cruz-Pano, Executive Judge of the
then CFI of Rizal [Quezon City]. The search was done in (1) No. 19, Road 3, Project 6, Quezon
City, and (2)784 Units C & D, RMS Building, Quezon Avenue, Quezon City, business addresses
of the "Metropolitan Mail" and "We Forum" newspapers, respectively. They were searched, and
several items were seized, including printing machines, equipment, paraphernalia, motor vehicles
and other articles used in the printing, publication and distribution of the newspapers, as well as
numerous papers, documents, books and other written literature.

Petitioners petitioned for the return of the seized articles and that the respondents be enjoined
from using the articles seized as evidence in the criminal case against Jose Burgos Jr. They assail
the validity of the warrant since only one address was indicated in the warrant.

The respondents wants the Court to dismiss the case on the ground that petitioners cae to the SC
without having previously sought the quashal of the search warrants with judge Pano.

Note: The search of the "We Forum" offices, which was televised in Channel 7 and widely
publicized in all metropolitan dailies.

Issue: Whether or not the two warrants were valid to justify seizure of the items.

Held: There was typographical error only, but the warrants are null
The defect pointed out (the single address) is obviously a typographical error. Precisely, two
search warrants were applied for and issued because the purpose and intent were to search two
distinct premises. It would be quite absurd and illogical for respondent judge to have issued two
warrants intended for one and the same place. Besides, the addresses of the places sought to be
searched were specifically set forth in the application, and since it was Col. Abadilla himself
who headed the team which executed the search warrants, the ambiguity that might have arisen
by reason of the typographical error is more apparent than real. The fact is that the place for
which the Search Warrant was applied for was 728 Units C & D, RMS Building, Quezon
Avenue, Quezon City, which address appeared in the opening paragraph of the said warrant.
Obviously this is the same place that respondent judge had in mind when he issued the warrant.

Nonetheless, there was no probable cause for the valid issuance of the warrant. Probable cause
for a search is defined as such facts and circumstances which would lead a reasonably discreet
and prudent man to believe that an offense has been committed and that the objects sought in
connection with the offense are in the place sought to be searched. And when the search warrant
applied for is directed against a newspaper publisher or editor in connection with the publication
of subversive materials, as in the case at bar, the application and/or its supporting affidavits must
contain a specification, stating with particularity the alleged subversive material he has published
or is intending to publish. Mere generalization will not suffice. Thus, the broad statement in Col.
Abadilla's application that petitioner "is in possession or has in his control printing equipment
and other paraphernalia, news publications and other documents which were used and are all
continuously being used as a means of committing the offense of subversion do not satisfy the
requirement of probable cause. (Their basis for probable cause was that they had conducted
surveillance of the premises)

Another factor which makes the search warrants under consideration constitutionally
objectionable is that they are in the nature of general warrants. It is argued that the search
warrants described the articles sought to be seized, but their subversive nature was not shown
(e.g. the cars/trucks, the prnting equipment, etc,) However, the description of the articles sought
to be seized under the search warrants in question cannot be characterized differently.

As a result of the search of the offices, the premises were padlocked and sealed, with the further
result that the printing and publication of said newspapers were discontinued. Such closure is in
the nature of previous restraint or censorship abhorrent to the freedom of the press guaranteed
under the fundamental law, and constitutes a virtual denial of petitioners' freedom to express
them in print.
Note: Pres. Marcos himself in the December 10, 1982 issue of the Daily Express, denied the
request of the military authorities to sequester the property seized from petitioners on December
7, 1982.

PEOPLE VS VELOSO

FACTS:

In May, 1923, the building located at No. 124 Calle Arzobispo, City of Manila, was used by
anorganization known as the Parliamentary Club. Jose Ma. Veloso was at that time a member of
theHouse of Representative of the Philippine Legislature. He was also the manager of the club.-
The police of Manila had reliable information that the so-called Parliamentary Club was
nothingmore than a gambling house. Indeed, on May 19, 1923, J. F. Townsend, the chief of the
gamblingsquad, had been to the club and verified this fact. As a result, on May 25, 1923,
Detective AndresGeronimo of the secret service of the City of Manila, applied for, and obtained
a search warrantfrom Judge Garduo of the municipal court. Thus provided, the police attempted
to raid theParliamentary Club a little after three in the afternoon of the date above- mentioned.
They foundthe doors to the premises closed and barred. Accordingly, one band of police
including policemanRosacker, ascended a telephone pole, so as to enter a window of the house.
Other policemen,headed by Townsend, broke in the outer door.

Once inside the Parliamentary Club, nearly fifty persons were apprehended by the police. One of
them was the defendant Veloso. Veloso asked Townsend what he wanted, and the latter
showedhim the search warrant. Veloso read it and told Townsend that he was Representative
Veloso andnot John Doe, and that the police had no right to search the house. Townsend
answered thatVeloso was considered as John Doe. As Veloso's pocket was bulging, as if it
contained gamblingutensils, Townsend required Veloso to show him the evidence of the game.
About five minuteswas consumed in conversation between the policemen and the accused the
policemen insistingon searching Veloso, and Veloso insisting in his refusal to submit to the
search.

At last the patience of the officers was exhausted. So policeman Rosacker took hold of
Velosoonly to meet with his resistance. Veloso bit Rosacker in the right forearm, and gave him a
blow inanother part of the body, which injured the policeman quite severely. Through the
combinedefforts of Townsend and Rosacker, Veloso was finally laid down on the floor, and long
sheets of paper, of reglas de monte, cards, cardboards, and chips were taken from his pockets.

All of the persons arrested were searched and then conducted to the patrol wagons. Velosoaguain
refused to obey and shouted offensive epithets against the police department. It wasnecessary for
the policemen to conduct him downstairs. At the door, Veloso resisted sotenaciously that three
policemen were needed to place him in the patrol wagon.
ISSUE

WON the search warrant and the arrest of Veloso was valid.

RULING Yes.

It is provided, among other things, in the Philippine Code on Criminal Procedure that a
searchwarrant shall not issue except for probable cause and upon application supported by
oathparticularly describing the place to be searched and the person of thing to be seized. The
name and description of the accused should be inserted in the body of the warrant andwhere the
name is unknown there must be such a description of the person accused as will enable the
officer to identify him when found.

A warrant for the apprehension of a person whose true name is unknown, by the name of "John
Doe" or "Richard Roe," "whose other or true name in unknown," is void, without other and
furtherdescriptions of the person to be apprehended, and such warrant will not justify the officer
inacting under it. Such a warrant must, in addition, contain the best descriptio personae
possibleto be obtained of the person or persons to be apprehended, and this description must
besufficient to indicate clearly the proper person or persons upon whom the warrant is to
beserved; and should state his personal appearance and peculiarities, give his occupation
andplace of residence, and any other circumstances by means of which he can be identified.In
the first place, the affidavit for the search warrant and the search warrant itself described
thebuilding to be searched as "the building No. 124 Calle Arzobispo, City of Manila,
PhilippineIslands." This, without doubt, was a sufficient designation of the premises to be
searched.As the search warrant stated that John Doe had gambling apparatus in his possession in
thebuilding occupied by him at No. 124 Calle Arzobispo, City of Manila, and as this John Doe
was Jose Ma. Veloso, the manager of the club, the police could identify John Doe as Jose Ma.
Veloso without difficulty.

5. When there is waiver of a right or gives his consent.

8. PEOPLE VS. BAULA, 344 SCRA 663

FACTS:

Jupiter Caburao, allegedly decided to follow his mother, Patrocinia Caburao, who had earlier left
their house to settle her due obligations at a store owned by Brigida Tumamang. Jupiter noticed a
commotion and he focused his flashlight towards the direction where he heard the commotion
and saw Crisanto Baula and Danilo Dacucos in the act of hacking a person who was lying on the
ground, while Robert Baula and Ruben Baula stood as lookouts. The assault allegedly lasted for
about 4minutes. Jupiter went near the lifeless body of the victim who turned out to be his own
mother. Her head and face sustained four hacking wounds, two of which damaged her brain
tissues.

Police authorities, led by SPO4 Fermin Mirande, went to the locus criminis, and repaired to the
respective houses of accused-appellants. The policemen asked Ruben Baula and Crisanto Baula
for the clothing they wore on the night of the murder. Ruben Baula gave his bloodstained pair of
short pants, and Crisanto Baula turned over his bloodstained polo shirt. The policemen next went
to the hut of Danilo Dacucos. Inside the hut, the group found hanging on the wall a bloodstained
bolo. The bloodstained pair of short pants, polo shirt and bolo, together with the victim's dried
blood samples, were sent on the same day to the National Bureau of Investigation, Dagupan City
Branch Office, for forensic examination. The results of the examination disclosed that the
bloodstains found in the bolo, the bloodstains on the polo shirt and the bloodstains on the pair of
short pants had the same type "O" blood as that of the victim.

Appeal for review on the decision of the lower court in the ground that the articles sought
(bloodstained bolo, shirt and shortpants) cannot be admitted as evidence against the accused
since it was seized without a valid search and seizure warrant.

ISSUE:

Whether or not the warrantless search conducted was valid under a consented search

HELD:

No. The articles are unlawfully searched and seized. A search incidental to a valid arrest
is one of the statutory exceptions to the constitutional mandate that no search and seizure shall be
effected without a valid warrant. In this instance, the arrest should be lawful before search and
seizure by the arresting officer would be conducted. A warrantless arrest may be effected by the
arresting officer when in his presence the person arrested have committed, committing or
attempting to commit the crime. In the case at bar, Accused-appellants were not being arrested at
the time that the subject articles were allegedly taken from them but were just being questioned
by the police officers conducting the investigation about the death of Patrocinia Caburao. The
investigating officers had no personal knowledge of facts indicating that the accused had
committed the crime. Being in no position to effect a warrantless arrest, the police officers were
thus likewise barred from effecting a warrantless search and seizure.

The Court finds it less than credible the stance of the prosecution that the polo shirt and short
pants have been voluntarily given. An alleged consent to a warrantless search and seizure cannot
be based merely on the presumption of regularity in the performance of duty. This presumption,
by itself, cannot prevail against the constitutionally protected rights of an individual, and zeal in
the pursuit of criminals cannot ennoble the use of arbitrary methods that the Constitution itself
abhors.

8. VEROY VS. LAYAGUE, 210 SCRA 97

FACTS:

Capt. Reynaldo Obrero of the Talomo Patrol Station, PC/INP raided Veroys house in
Davao City on information that the said residence was being used as a safehouse of rebel
soldiers. They were able to enter the yard with the help of the caretakers but did not enter the
house since the owner was not present and they did not have a search warrant. Permission was
requested by phone to Ma. Luisa Veroy who consented on the condition that the search be
conducted in the presence of Major Macasaet. The following day, Capt. Obrero and Maj.
Macasaet met at the Veroys house to conduct the search pursuant to the authority granted by
Ma. Luisa. Capt. Obrero recovered a .45 cal. handgun with a magazine containing 7 live bullets
in a black clutch bag inside an unlocked drawer in the childrens room. 3 half-full jute sacks
containing printed materials of RAM-SFP were also found in the children's room. A search of
the children's recreation and study area revealed a big travelling bag containing assorted clothing,
a small black bag containing a book entitled "Islamic Revolution Future Path of the Nation", a
road map of the Philippines, a telescope, a plastic bag containing assorted medicines and
religious pamphlets was found in the master's bedroom.

ISSUE:

Whether the permission granted by Ma. Luisa Veroy for ascertaining thereat the presence
of alleged rebel soldiers include the authority to conduct a room to room search once inside the
house.

HELD:

The Constitution guarantees the right of the people to be secure in their persons, houses,
papers and effects against unreasonable searches and seizures (Article III, Section 2 of the 1987
Constitution). However, the rule that searches and seizures must be supported by a valid warrant
is not an absolute one. The necessity of the permission obtained from Ma. Luisa underlines the
recognition of Capt. Obrero of the need of a search warrant to enter the house. The permission
granted by was for the purpose of ascertaining thereat the presence of the alleged "rebel"
soldiers. The permission did not include any authority to conduct a room to room search once
inside the house. The police officers had ample time to procure a search warrant but did not.
Warrantless searches were declared illegal because the officials conducting the search had every
opportunity to secure a search warrant. The items taken were, therefore, products of an illegal
search, violative of their constitutional rights. As such, they are inadmissible in evidence in the
criminal actions instituted against them.
8. PEOPLE V DAMASO, 212 SCRA 457

Facts:
The Philippine Constabulary apprehended alleged members of the NPA in Bani, Dagupan. When
interrogated, they revealed that there is an underground safehouse at Gracia Village in Urdaneta,
Pangasinan. From that safehouse, they found subversive materials.

The group proceeded to Bonuan, Dagupan City and put under surveillance the rented apartment
of Rosemarie Aritumba, the sister of one who they earlier arrested. They interviewed a visitor of
Aritumba, Luzviminda Morados, and she stated that she worked with accused Damaso alias
Bernardo/BERNIE MENDOZA/Ka Dado. Morados guided them to the house rented by Damaso
and when they found that the place was already vacated, they sought the help of the barangay
captain.

They saw Luz Tanciangco outside the new house rented by Damaso and upon informing that the
authorities already knew that they were members of NPA, Tanciangco requested the group to
enter the house. Upon entering, they saw subversive materials such as radio sets, pamphlets
entitled "Ang Bayan," xerox copiers and a computer machine. The group requested the persons
in the house to allow them to further look around. One room was opened by Tanciangco and they
saw more materials including maps and one M-14 rifle, bullets and ammunitions.They seized the
articles and brought them to their headquarters for final inventory. They likewise brought the
persons found in the house and said persons revealed that appellant was the lessee of the house
and owned the items confiscated therefrom. Damaso was charged of violation of PD No. 1866
and was found guilty by the lower court.

Issue:
Whether Damaso waived his right against unreasonable search and seizure when his alleged
helpers Tanciangco and Morados requested PC to get inside the house and to look around the
appellants house

Held:
No. The constitutional immunity from unreasonable searches and seizures, being personal one,
cannot be waived by anyone except the person whose rights are invaded or one who is expressly
authorized to do so in his or her behalf. No evidence that would establish the fact that Luz
Morados was indeed the appellant's helper or if it was true that she was his helper, that the
appellant had given her authority to open his house in his absence. Without this evidence, the
authorities' intrusion into the appellant's dwelling cannot be given any color of legality.

6. Stop and Frisk

5. ESQUILLO V PEOPLE, GR No. 182010

Facts:
PO1 Alvin Cruzin, together with PO2 Angel Aguas conducted surveillance on the activities of an
alleged notorious snatcher named Ryan operating Bayanihan St., Malibay, Pasay City. Accused
Esquillo was standing three meters away from the position of the police men and she was seen
placing be a small heat-sealed transparent plastic sachet containing white substance inside a
yellow cigarette case.

Suspicious on the actions of Esquillo, PO1 Cruz approached her, introduced himself as a police
officer and inquired about the plastic sachet. Instead of replying, however, petitioner attempted
to flee to her house nearby but was timely restrained by PO1 Cruzin

After apprising petitioner of her constitutional rights, PO1 Cruzin confiscated the plastic
sachet. With the seized item, petitioner was brought for investigation to a Pasay City Police
Station. After laboratory examination of the substance, it was found to be shabu and Esquillo
after the conduct of drug test appeared to be positive.

Esquillo contends that she was framed up by the police officers to extort money from her and her
family and that she was illegally arrested. She was found guilty of violation of par. 3 of Section
11, Article II of R. A. 9165 or the Comprehensive Dangerous Drugs Act of 2002.

Issues:
Whether stop-and-frisk concept a valid exception for law enforcers to search without warrant

Whether there was probable cause to search petitioner under the stop-and-frisk concept

Held:
Yes. A search may be conducted by law enforcers on the strength of a valid search warrant
except on: (a) consented searches; (b) as an incident to a lawful arrest; (c) searches of vessels and
aircraft for violation of immigration, customs, and drug laws; (d) searches of moving vehicles;
(e) searches of automobiles at borders or constructive borders; (f) where the prohibited articles
are in plain view; (g) searches of buildings and premises to enforce fire, sanitary, and building
regulations; and (h) stop and frisk operations.[20]

Yes. Stop and frisk operation entails the act of a police officer to stop a citizen on the street,
interrogate him, and pat him for weapon(s) or contraband. The police officer should properly
introduce himself and make initial inquiries, approach and restrain a person who manifests
unusual and suspicious conduct, in order to check the latters outer clothing for possibly
concealed weapons. The apprehending police officer must have a genuine reason, in accordance
with the police officers experience and the surrounding conditions, to warrant the belief that the
person to be held has weapons (or contraband) concealed on him. It should therefore be
emphasized that a search and seizure should precede the arrest for this principle to apply.

In the case at bar, when PO1 Cruzin saw petitioner placing a plastic sachet containing white
crystalline substance into her cigarette case, it was in his plain view. Given his training as a law
enforcement officer, it was instinctive on his part to be drawn to curiosity and to approach her.
That petitioner reacted by attempting to flee after he introduced himself as a police officer and
inquired about the contents of the plastic sachet all the more pricked his curiosity.

Roan v Gonzales

145 SCRA 687

Facts: the petitioner claims he was the victim of an illegal search and seizure conducted by the
military authorities. The articles seized from him are sought to be used as evidence in his
prosecution for illegal possession of firearms. He asks that their admission be temporarily
restrained

The challenged search warrant was issued by the respondent judge on May 10, 1984.The
petitioner's house was searched two days later but none of the articles listed in the warrant was
discovered. However, the officers conducting the search found in the premises one Colt Magnum
revolver and eighteen live bullets which they confiscated. They are now the bases of the charge
against the petitioner.

The applicant was asking for the issuance of the search warrant on the basis of mere hearsay and
not of information personally known to him, as required by settled jurisprudence. In any case,
Judge Gonzales did not ask his own searching questions. He limited himself to the contents of
the affidavit when he issued the search warrant. He did not take the applicant's deposition in
writing and attach them to the record, together with the affidavit presented to him

Issue: WON the search warrant was valid

Held: VOID. The respondent judge almost unquestioningly received the witnesses' statement
that they saw eight men deliver arms to the petitioner in his house on May 2, 1984. This was
supposedly done overtly, and one witness said he saw everything through an open window of the
house while he was near the gate. He could even positively say that six of the weapons were.45
caliber pistols and two were.38 caliber revolvers.One may well wonder why it did not occur to
the respondent judge to ask how the witness could be so certain even as to the caliber of the
guns, or how far he was from the window, or whether it was on the first floor or a second floor,
or why his presence was not noticed at all, or if the acts related were really done openly, in the
full view of the witnesses, considering that these acts were against the law. These would have
been judicious questions but they were injudiciously omitted. Instead, the declarations of the
witnesses were readily accepted and the search warrant sought was issued forthwith.

The respondent judge also declared that he "saw no need to have applicant Quillosa's deposition
taken considering that he was applying for a search warrant on the basis of the information
provided by the aforenamed witnesses whose depositions as aforementioned had already been
taken by the undersigned."

In other words, the applicant was asking for the issuance of the search warrant on the basis of
mere hearsay and not of information personally known to him, as required by settled
jurisprudence." The rationale of the requirement, of course, is to provide a ground for a
prosecution for perjury in case the applicant's declarations are found to be false. His application,
standing alone, was insufficient to justify the issuance of the warrant sought. It was therefore
necessary for the witnesses themselves, by their own personal information, to establish the
applicant's claims.

Mata v Bayona

FACTS: Soriano Mata was accused under Presidential Decree (PD) 810, as amended by PD
1306, the information against him alleging that Soriano Mata offered, took and arranged bets on
the Jai Alai game by selling illegal tickets known as Masiao tickets without any authority
from the Philippine Jai Alai & Amusement Corporation or from the government authorities
concerned. Mata claimed that during the hearing of the case, he discovered that nowhere from
the records of the said case could be found the search warrant and other pertinent papers
connected to the issuance of the same, so that he had to inquire from the City Fiscal its
whereabouts, and to which inquiry Judge Josephine K. Bayona, presiding Judge of the City
Court of Ormoc replied, it is with the court. The Judge then handed the records to the Fiscal
who attached them to the records. This led Mata to file a motion to quash and annul the search
warrant and for the return of the articles seized, citing and invoking, among others, Section 4 of
Rule 126 of the Revised Rules of Court. The motion was denied by the Judge on 1 March 1979,
stating that the court has made a thorough investigation and examination under oath of Bernardo
U. Goles and Reynaldo T. Mayote, members of the Intelligence Section of 352nd PC Co./Police
District II INP; that in fact the court made a certification to that effect; and that the fact that
documents relating to the search warrant were not attached immediately to the record of the
criminal case is of no moment, considering that the rule does not specify when these documents
are to be attached to the records. Matas motion for reconsideration of the aforesaid order having
been denied, he came to the Supreme Court, with the petition for certiorari, praying, among
others, that the Court declare the search warrant to be invalid for its alleged failure to comply
with the requisites of the Constitution and the Rules of Court, and that all the articles confiscated
under such warrant as inadmissible as evidence in the case, or in any proceedings on the matter.

Issue: Whether or not Search Warrant is valid

Held: NOT VALID. The search warrant is tainted with illegality by the failure of the Judge to
conform with the essential requisites of taking the depositions in writing and attaching them to
the record, rendering the search warrant invalid.

Deposition, sometimes used in a broad sense to describe any written statement verified by oath;
but in its more technical and appropriate sense the meaning of the word is limited to written
testimony of a witness given in the course of a judicial proceeding in advance of the trial or
hearing upon oral examination.

Mere affidavits of the complainant and his witnesses are thus not sufficient. The examining
Judge has to take depositions in writing of the complainant and the witnesses he may produce
and to attach them to the record. Such written deposition is necessary in order that the Judge may
be able to property determine the existence or non-existence of the probable cause, to hold liable
for perjury the person giving if it will be found later his declarations are false.

Search and Seizure plus Right to Privacy

People vs IDEL AMMINUDIN


FACTS: Idel Amminudin, who represented himself as a watch trader, was alighting from the
vessel (which travelled from Zamboanga to Ilo-ilo) when he was searched and apprehended by
police operatives after finding marijuana in his bag along with two counterfeit wrist watches.
The operatives were acting on an intelligence report allegedly from a reliable and regular
informer. Aminnudin was convicted by the RTC for transporting illegal drugs. On appeal,
Aminnudin argued that he was searched without a warrant and the marijuana found in his
possession was inadmissible as evidence against him. The Police claim that a search warrant was
no longer necessary because he was in the act of committing a crime.
ISSUE: Whether the search was valid.
DECSION: No. The search was a violation of the constitutional rights of Aminnudin. The police
operatives could have obtained a search warrant acting on the intelligence report as the vessel
was identified, the time of arrival was certain, and the belongings of the search was identified.
The police did not exert an effort to comply with the law.
It cannot be said that he was acting suspiciously to justify the warrantless search and
arrest. He was with other passengers who were innocently disembarking through the gangplank
of the MV Wilcon 9. There was no outward reason to search and arrest him. While this is not to
say that Aminnudin is innocent, for by his very words it can be seen that he was lying, that fact
alone does not justify that he is guilty. He is presumed innocent, and he will be so declared even
if his defense is weak as long as the evidence of the prosecution is not strong enough to convict
him. The evidence is inadmissible because it was obtained in violation of the rights of the
accused.

PONSICA VS IGNALAGA
FACTS: A rally by mostly laid-off sugar workers was violently dispersed using fire hoses and
tear gas and gunshots killing 15 persons infront of the city hall of Escalante, Negros Occidental.
The military officers who were involved in the dispersal charged Rolando Ponsica of BAYAN
Negros-Chapter and other persons for inciting to sedition. However because MTC Escalante
Judge Romulo Ignalaga is on official leave, Mayor Braulio Lumayno acted on the complaint and
recommended filing of charges, arrest and bail against Ponsica et.al. The mayor then referred the
case to Judge Ignalaga. Ponsica et.al. moved to quash the warrant of arrest issued against them
by the mayor but the prosecution claimed that in the Local Government Code, in the temporary
absence of the judge, the municipal mayor may conduct the preliminary examination in criminal
cases, when in his opinion, the investigation cannot be delayed in the interest of justice. Ponsica
rejects this saying that the provision in the Loc. Gov. Code was already repealed by the 1985
Revised Rules on Criminal Procedure. The Judge, however, confirmed the warrants of arrest
issued by the mayor.
ISSUES: Was the warrant of arrest and preliminary investigation done by the mayor, as
confirmed by the judge, valid?
FACTS: No. The new constitution (1987) was already ratified by the people. The determination
of the probable cause and the subsequent issuance of the warrants of search and arrest may only
be done by the judge. As the law now stands, the mayor may no longer conduct preliminary
investigation, the authority to do so being limited under Section 2, Rule 1 1 2 of the Rules of
Court to (1) provincial or city fiscals and their assistants; (2) judges of the Municipal Trial
Courts and Municipal Circuit Trial Courts; (3) national and regional state prosecutors; and (d)
such other officers as may be authorized by law. But only "the judge" may issue search and
arrest warrants after due determination of probable cause.
Also, the affidavits of the witnesses do not show inciting to sedition or sedition, saying that the
people were shouting seditious words and were unruly and other generalities. It cannot
substantively show that there was probable cause as determined by the mayor for the grant of the
warrants of arrest. There has also been no searching investigation conducted by the mayor to the
witnesses except one Capt. Sanson, whose testimony does not show sedition or inciting to
sedition, but rather obstruction or grave coercion or malicious mischief or illegal possession of
firearms.
ABERCA v VER L-69866 April 15, 1988
FACTS: Task Force Makabansa (TFM) was ordered by General Fabian Ver "to conduct pre-
emptive strikes against known communist-terrorist (CT) underground houses in view of
increasing reports about CT plans to sow disturbances in Metro Manila,". Plaintiffs allege,
among others, that complying with said order, elements of the TFM raided several places,
employing in most cases defectively issued judicial search warrants; that during these raids,
certain members of the raiding party confiscated a number of purely personal items belonging to
plaintiffs; that plaintiffs were arrested without proper warrants issued by the courts; that for some
period after their arrest, they were denied visits of relatives and lawyers; that plaintiffs were
interrogated in violation of their rights to silence and counsel; that military men who interrogated
them employed threats, tortures and other forms of violence on them in order to obtain
incriminatory information or confessions and in order to punish them; that all violations of
plaintiffs constitutional rights were part of a concerted and deliberate plan to forcibly extract
information and incriminatory statements from plaintiffs and to terrorize, harass and punish
them, said plans being previously known to and sanctioned by defendants.
The plaintiffs filed a petition for certiorari. A motion to dismiss was filed by defendants, stating
that plaintiffs may not cause a judicial inquiry about their detention because the writ of habeas
corpus was suspended; defendants are immune from liability for acts done in their official duties;
and there was no cause of action. On Nov 8, 1983, Judge Fortun granted the motion to dismiss,
which prompted plaintiffs to file a MR on Nov 18, 1983. He later inhibited himself and was
replaced Judge Lising, who denied the MR for being filed out of time. Another MR was filed,
and was only modified to include Maj. Aguinaldo and MSgt. Balaba for officers accountable in
the said complaint.
ISSUE: WON the writ of habeas corpus suspends their rights in the constitution?
HELD: What the writ suspends is merely the right of an individual to seek release from
detention as a speedy means of obtaining liberty. It cannot suspend their rights and cause of
action for injuries suffered due to violation of their rights.
Article 32 of the Civil Code which renders any public officer or employee or any private
individual liable in damages for violating the Constitutional rights and liberties of another, as
enumerated therein, does not exempt the respondents from responsibility. Only judges are
excluded from liability under the said article, provided their acts or omissions do not constitute a
violation of the Penal Code or other penal statute.
We do not agree. We find merit in petitioners' contention that the suspension of the privilege of
the writ of habeas corpus does not destroy petitioners' right and cause of action for damages for
illegal arrest and detention and other violations of their constitutional rights. The suspension does
not render valid an otherwise illegal arrest or detention. What is suspended is merely the right of
the individual to seek release from detention through the writ of habeas corpus as a speedy
means of obtaining his liberty.
Firstly, it is wrong to at the plaintiffs' action for damages 5 Section 1, Article 19. to 'acts of
alleged physical violence" which constituted delict or wrong. Article 32 clearly specifies as
actionable the act of violating or in any manner impeding or impairing any of the constitutional
rights and liberties enumerated therein, among others
The complaint in this litigation alleges facts showing with abundant clarity and details, how
plaintiffs' constitutional rights and liberties mentioned in Article 32 of the Civil Code were
violated and impaired by defendants. The complaint speaks of, among others, searches made
without search warrants or based on irregularly issued or substantially defective warrants;
seizures and confiscation, without proper receipts, of cash and personal effects belonging to
plaintiffs and other items of property which were not subversive and illegal nor covered by the
search warrants; arrest and detention of plaintiffs without warrant or under irregular, improper
and illegal circumstances; detention of plaintiffs at several undisclosed places of 'safehouses"
where they were kept incommunicado and subjected to physical and psychological torture and
other inhuman, degrading and brutal treatment for the purpose of extracting incriminatory
statements. The complaint contains a detailed recital of abuses perpetrated upon the plaintiffs
violative of their constitutional rights.
PANGANIBAN v CESAR 159 SCRA 599 -
cant find the case online :(

Warrantless searches and seizures--when valid or not

1. VALMONTE V DE VILLA (September 29, 1989)

FACTS:
The petitioners were Atty. Ricardo Valmonte, in his capacity as a citizen, taxpayer, member of
the IBP and resident of Valenzuela, together with the Union of Lawyers and Advocates for
People's rights (ULAP).

They sought to have the National Capital Region District Command's "checkpoints" in Metro
Manila or elsewhere declared as unconstitutional. These various checkpoints were installed
pursuant to the activation of the NCRDC on Jan. 20, 1987 (as per Letter of Instruction 02/87 of
the Philippine General Headquarters, AFP). They were given the mission of conducting security
operations within its area of responsibility and peripheral areas, for the purpose of establishing
an effective territorial defense, maintaining peace and order, and providing an atmosphere
conducive to the social, economic and political development of the National Capital Region.

The petitioners averred that the residents are worried of being harassed, and that their safety were
being placed at the whimsical disposition of the military. Their cars (including that of Valmonte
on several occasions) were subjected to regular searches, at night or dawn, without the benefit of
a search warrant or court order.

This fear came from an incident on July 9, 1988, when Benjamin Parpon, a supply officer of
Valenzuela, was gunned down, allegedly in cold blood, by NCRDC members along McArthur
Highway, for ignoring and/or refusing to submit himself to the checkpoint and continuing to
speed off, in spite of warning shots fired in the air.

Petitioners contended that these checkpoints give the respondents (Gen. Renato de Villa and the
NCR District Command) blanket authority to search and seize without warrant.

ISSUE:
WON the installation of checkpoints was constitutional.
WON the petitioners' concern for their safety and apprehension from harassment by the military
were sufficient grounds to declare the checkpoints illegal.

RULING:
13 Justices voted to dismiss. 2 dissented. (see below)

HELD:
There were no sufficient grounds to declare the checkpoints as unconstitutional. The petitioners
failed to present proof that the routine checks by the military indeed resulted to specific violation
of rights against unlawful search and seizure.
In a previous case filed by ULAP (ULAP vs. Integrated National Police), it was held that
individual petitioners who do not allege that any of their rights were violated are not qualified to
bring the action, as real parties in interest.

The constitutional right against unreasonable searches and seizures is a personal right invocable
only by those whose rights have been infringed, or threatened to be infringed. What constitutes a
reasonable or unreasonable search and seizure in any particular case is purely a judicial question,
determinable from a consideration of the circumstances involved.

Valmonte's general allegation without stating the details of the incidents is not sufficient to
enable the Court to determine whether there was a violation of rights. Not all searches and
seizures are prohibited. Those which are reasonable are not forbidden. A reasonable search is not
to be determined by any fixed formula but is to be resolved according to the facts of each case.

Where, for example, the officer merely draws aside the curtain of a vacant vehicle which is
parked on the public fair grounds, or simply looks into a vehicle, or flashes a light therein, these
do not constitute unreasonable search.

The setting up of the questioned checkpoints may be considered as a security measure to enable
the NCRDC to pursue its mission of establishing effective territorial defense and maintaining
peace and order for the benefit of the public. Checkpoints may also be regarded as measures to
thwart plots to destabilize the government, in the interest of public security. (There were
increased killings of police and military men by NPA "sparrow units," abundance of unlicensed
firearms and the alarming rise in lawlessness and violence in urban centers which all sum up
to what one can rightly consider as abnormal times.)

The inherent right of the state to protect its existence and promote public welfare should
prevail over an individual's right against a warrantless search which is reasonably
conducted.

True, the manning of checkpoints by the military is susceptible of abuse by the men in uniform.
But, at the cost of occasional inconvenience, discomfort and even irritation to the citizen, the
checkpoints during these abnormal times, when conducted within reasonable limits, are part of
the price we pay for an orderly society and a peaceful community.

Justice Isagani Cruz's Dissent:


"The sweeping statements in the majority opinion are as dangerous as the checkpoints it would
sustain, and fraught with serious threats to individual liberty. The bland declaration that
individual rights must yield to the demands of national security ignores the fact that the Bill of
Rights was intended precisely to limit the authority of the State even if asserted on the ground of
national security. What is worse is that the searches and seizures are peremptorily pronounced to
be reasonable even without proof of probable cause and much less the required warrant. The
improbable excuse is that they are aimed at 'establishing an effective territorial defense,
maintaining peace and order, and providing an atmosphere conducive to the social, economic
and political development of the NCR. For these purposes, every individual may be stopped and
searched at random and at any time simply because he excites the suspicion, caprice, hostility or
malice of the officers manning the checkpoints, on pain of arrest or worse, even being shot to
death, if he resists."

Justice Sarmiento's Dissent:


He agreed that the existence alone of checkpoints makes search done therein unreasonable and
repugnant to the Constitution.
"The Charter says that the people enjoy the right of security of person, home, and effects. It is
also the bedrock the right of the people to be left alone on which the regime of law and
constitutionalism rest. It is not, as the majority would put it, a matter of occasional
inconveniences, discomfort and even irritation. To say that it is to trivialize the plain command
of the Constitution. Checkpoints are things of martial rule, and things of the past. They are
repressive measures, the same measures against which we had fought so painstakingly in our
quest for liberty, a quest that ended at EDSA and a quest that terminated a dictatorship. How
soon we forget."
He further submitted that the "checkpoints" are somewhat akin to a roving/general search
warrant, like having "Orwell's Big Brother watching every step we take and every move we
make".

RESOLUTION ON THE MOTION FOR RECONSIDERATION (185 SCRA 665)

Valmonte and ULAP filed for a motion for reconsideration, but it was likewise denied with
finality.
The Court however clarified that the Sept 1989 Decision did not legalize ALL checkpoints. It
only declared that they are not illegal per se.
Checkpoints are security measures designed to entrap criminals and insurgents, and those
who may be transporting goods (firearms, etc) for illegal trade.
Under exceptional circumstances (e.g., survival of an organized government is on the
balance, lives of the people are in grave peril), checkpoints may be allowed and installed.
When said situations however cease to exist, then can the checkpoints be declared
unreasonable.
Events that were happening then that, according to the majority who voted to dismiss,
justified the checkpoints continued validity include:
o 6th coup detat attempt on Dec. 1, 1989
o NPA sparrow units liquidating armed forces and police personnel
o Murders, sex crimes, hold-ups and drug abuse as daily occurrences
o Trade of unlicensed firearms and ammunition
o Smuggling was at an all-time high
The Court reiterated that while the routine checkpoints do intrude a motorists right of
free passage, it is only a brief detention of travellers during which they are required to
answer a brief question or two. Vehicles were neither searched nor the occupants
subjected to a body search--the inspections were limited to visual search, thus cannot
be regarded as violative of the right against unreasonable search.
They cited US v Martinez-Fuerte--Routine checkpoint stops do not intrude motorists: (1)
interference with traffic is minimal, and (2) they involve less discretionary authority--
they are done in a regular manner, and the locations are chosen by higher officials, not
the field officers.
Checkpoints as valid warrantless searches are similar to those warrantless arrests during
commission of a crime, or immediately thereafter.
They are not unconstitutional because they are founded on (1) public interest, (2) safety,
and (3) necessity.
Nonetheless, the Court also ruled that when the military/police personnel shall abuse their
authority, they shall be liable criminally and civilly. In this case, the Court just had to
assume that the men in uniform live by their code of honor, and that since a checkpoint
is a military concoction, the police were behooved to improve the quality of men
assigned to man said checkpoints.

Justice Cruz and Justice Sarmiento stood by their original dissents:


Justice Cruz averred that even criminals, more so the innocent are entitled to the right
against unreasonable searches and seizures. Constitutional shortcuts should not be
allowed in a free regime.
o On visual searches--Justice Cruz said that in light of the cited Dec 1 coup
attempt, the checkpoints were then somehow useless, because the visual search
did not detect the high powered weapons.
o He also emphasized that probable cause should be determined by judges
personally, not by a soldier or policemen.
o The remedy is to remove the source of evil, instead of leaving it unchecked and
simply suggesting a cure, which is not even effective. It is like inoculating a
patient after exposing him to contagion.
Justice Sarmiento pondered that under the Constitution, even martial law cannot suspend
the fundamental rights--how then can we allow a mere executive act to do so?
o A states right to defend itself cannot be exercised at the expense of liberty.
o Checkpoints validity cannot be justified by the rule laid down in Malasuigi
where an authority who has probable cause may place a person under arrest or
search. In checkpoints, what the authorities do is fish for probable cause even if
there was none in the beginning. Thus, it is unconstitutional.

2. THE PEOPLE OF THE PHILIPPINES vs.MIKAEL MALMSTEDT

FACTS:
In an information dated 15 June 1989, accused-appellant Mikael Malmstedt (hereinafter
referred to as the accused) was charged before the Regional Trial Court (RTC) of La Trinidad,
Benguet, Branch 10, in Criminal Case No. 89-CR-0663, for violation of Section 4, Art. II of
Republic Act 6425, as amended, otherwise known as the Dangerous Drugs Act of 1972, as
amended.
Accused Mikael Malmstedt, a Swedish national, entered the Philippines for the third time in
December 1988 as a tourist. He had visited the country sometime in 1982 and 1985.
In the evening of 7 May 1989, accused left for Baguio City. Upon his arrival thereat in
the morning of the following day, he took a bus to Sagada and stayed in that place for two (2)
days. Then in the 7 in the morning of May 11, 1989, the accused went to Nangonogan bus stop in
Sagada.
At about 8: 00 o'clock in the morning of that same day (11 May 1989), Captain Alen
Vasco, the Commanding Officer of the First Regional Command (NARCOM) stationed at Camp
Dangwa, ordered his men to set up a temporary checkpoint at Kilometer 14, Acop, Tublay,
Mountain Province, for the purpose of checking all vehicles coming from the Cordillera Region.
The order to establish a checkpoint in the said area was prompted by persistent reports that
vehicles coming from Sagada were transporting marijuana and other prohibited drugs. Moreover,
information was received by the Commanding Officer of NARCOM, that same morning that a
Caucasian coming from Sagada had in his possession prohibited drugs. The group composed of
seven (7) NARCOM officers, in coordination with Tublay Police Station, set up a checkpoint at
the designated area at about 10:00 o'clock in the morning and inspected all vehicles coming from
the Cordillera Region.
The two (2) NARCOM officers started their inspection from the front going towards the
rear of the bus. Accused who was the sole foreigner riding the bus was seated at the rear thereof.
During the inspection, CIC Galutan noticed a bulge on accused's waist. Suspecting the
bulge on accused's waist to be a gun, the officer asked for accused's passport and other
identification papers. When accused failed to comply, the officer required him to bring out
whatever it was that was bulging on his waist. The bulging object turned out to be a pouch bag
and when accused opened the same bag, as ordered, the officer noticed four (4) suspicious-
looking objects wrapped in brown packing tape, prompting the officer to open one of the
wrapped objects. The wrapped objects turned out to contain hashish, a derivative of marijuana.
Thereafter, accused was invited outside the bus for questioning. But before he alighted
from the bus, accused stopped to get two (2) travelling bags from the luggage carrier. Upon
stepping out of the bus, the officers got the bags and opened them. A teddy bear was found in
each bag. Feeling the teddy bears, the officer noticed that there were bulges inside the same
which did not feel like foam stuffing. It was only after the officers had opened the bags that
accused finally presented his passport.
Accused was then brought to the headquarters of the NARCOM at Camp Dangwa, La
Trinidad, Benguet for further investigation. At the investigation room, the officers opened the
teddy bears and they were found to also contain hashish. Representative samples were taken
from the hashish found among the personal effects of accused and the same were brought to the
PC Crime Laboratory for chemical analysis.
In the chemistry report, it was established that the objects examined were hashish. a
prohibited drug which is a derivative of marijuana. Thus, an information was filed against
accused for violation of the Dangerous Drugs Act.
During the arraignment, accused entered a plea of "not guilty." For his defense, he raised
the issue of illegal search of his personal effects. He also claimed that the hashish was planted by
the NARCOM officers in his pouch bag and that the two (2) travelling bags were not owned by
him, but were merely entrusted to him by an Australian couple whom he met in Sagada. He
further claimed that the Australian couple intended to take the same bus with him but because
there were no more seats available in said bus, they decided to take the next ride and asked
accused to take charge of the bags, and that they would meet each other at the Dangwa Station.
ISSUE: Whether the search is illegal.
HELD: NO.
The Constitution guarantees the right of the people to be secure in their persons, houses,
papers and effects against unreasonable searches and seizures. However, where the search is
made pursuant to a lawful arrest, there is no need to obtain a search warrant. A lawful arrest
without a warrant may be made by a peace officer or a private person under the following
circumstances.
Sec. 5 Arrest without warrant; when lawful. A peace officer or a private person may, without
a warrant, arrest a person:
(a) When, in his presence, the person to be arrested has committed is actually committing, or is
attempting to commit an offense;
(b) When an offense has in fact just been committed, and he has personal knowledge of facts
indicating that the person to be arrested has committed it; and
(c) When the person to be arrested is a prisoner who has escaped from a penal establishment or
place where he is serving final judgment or temporarily confined while his case is pending, or
has escaped while being transferred from one confinement to another.
Accused was searched and arrested while transporting prohibited drugs (hashish). A
crime was actually being committed by the accused and he was caught in flagrante delicto. Thus,
the search made upon his personal effects falls squarely under paragraph (1) of the foregoing
provisions of law, which allow a warrantless search incident to a lawful arrest. While it is true
that the NARCOM officers were not armed with a search warrant when the search was made
over the personal effects of accused, however, under the circumstances of the case, there was
sufficient probable cause for said officers to believe that accused was then and there committing
a crime.
Probable cause has been defined as such facts and circumstances which could lead a
reasonable, discreet and prudent man to believe that an offense has been committed, and that the
objects sought in connection with the offense are in the place sought to be searched. Warrantless
search of the personal effects of an accused has been declared by this Court as valid, because of
existence of probable cause, where the smell of marijuana emanated from a plastic bag owned by
the accused, 10 or where the accused was acting suspiciously, 11 and attempted to flee.

ALIH VS. CASTRO

FACTS: On November 25, 1984, a contingent of more than two hundred Philippine marines and
elements of the home defense forces raided the compound occupied by the petitioners at Gov.
Alvarez street, Zamboanga City, in search of loose firearms, ammunition and other explosives.
The people inside the compound resisted the invasion and a crossfire between the Philippine
marines and the petitioner occurred, resulting in number of casualties. The petitioners
surrendered the next morning and 16 occupants were arrested, later to be finger-printed, paraffin-
tested and photographed over their objection. The military also inventoried and confiscated
several M16 rifles, M14 rifle, rifle grenades and rounds of ammunition.
Petitioner filed a petition for prohibition and mandamus with preliminary injunction and
restraining order. Their purpose was to recover the articles seized from them, to prevent these
from being used as evidence against them, and to challenge their finger-printing, photographing
and paraffin-testing being violative of their right against self-incrimination. Petitioner argued that
the arms and ammunition were taken without a search warrant as required by law under Sec. 3 of
the 1973 Constitution, and it be declared inadmissible in relation to Sec 4 (2) of the 1973
Constitution.
Respondent justified their act on the ground that they were acting under superior orders
and that the measures was necessary due to the aggravation of peace and order problem in their
place.
ISSUE: Whether the seizing of the items and the taking of the fingerprints and photographs of
the petitioners and subjecting them to paraffin testing are violative of the bill of Rights and are
inadmissible as evidence against them.
HELD: YES
The precarious state of lawlessness in Zamboanga City at the time in question certainly
did not excuse the non-observance of the constitutional guaranty against unreasonable searches
and seizures. There was no state of hostilities in the area to justify, assuming it could, the
repressions committed therein against the petitioners.
It is so easy to say that the petitioners were outlaws and deserved the arbitrary treatment
they received to take them into custody; but that is a criminal argument. It is also fallacious. Its
obvious flaw lies in the conclusion that the petitioners were unquestionably guilty on the strength
alone of unsubstantiated reports that they were stockpiling weapons.
The record does not disclose that the petitioners were wanted criminals or fugitives from
justice. At the time of the "zona," they were merely suspected of the mayor's slaying and had not
in fact even been investigated for it. As mere suspects, they were presumed innocent and not
guilty as summarily pronounced by the military.
Indeed, even if were assumed for the sake of argument that they were guilty, they would
not have been any less entitled to the protection of the Constitution, which covers both the
innocent and the guilty. This is not to say, of course, that the Constitution coddles criminals.
What it does simply signify is that, lacking the shield of innocence, the guilty need the armor of
the Constitution, to protect them, not from a deserved sentence, but from arbitrary punishment.
Every person is entitled to due process. It is no exaggeration that the basest criminal, ranged
against the rest of the people who would condemn him outright, is still, under the Bill of Rights,
a majority of one.
If the respondents did not actually disdain the Constitution when they made their illegal
raid, they certainly gave every appearance of doing so. This is truly regrettable for it was
incumbent on them, especially during those tense and tindery times, to encourage rather than
undermine respect for the law, which it was their duty to uphold.
In acting as they did, they also defied the precept that "civilian authority is at all times supreme
over the military" so clearly proclaimed in the 1973 Constitution. 11 In the instant case, the
respondents simply by-passed the civil courts, which had the authority to determine whether or
not there was probable cause to search the petitioner's premises. Instead, they proceeded to make
the raid without a search warrant on their own unauthorized determination of the petitioner's
guilt.
The respondents cannot even plead the urgency of the raid because it was in fact not
urgent. They knew where the petitioners were. They had every opportunity to get a search
warrant before making the raid. If they were worried that the weapons inside the compound
would be spirited away, they could have surrounded the premises in the meantime, as a
preventive measure. There was absolutely no reason at all why they should disregard the orderly
processes required by the Constitution and instead insist on arbitrarily forcing their way into the
petitioner's premises with all the menace of a military invasion.
Conceding that the search was truly warrantless, might not the search and seizure
be nonetheless considered valid because it was incidental to a legal arrest? Surely not. If all
the law enforcement authorities have to do is force their way into any house and then pick
up anything they see there on the ground that the occupants are resisting arrest, then we
might as well delete the Bill of Rights as a fussy redundancy.
If the arrest was made under Rule 113, Section 5, of the Rules of Court in connection
with a crime about to be committed, being committed, or just committed, what was that crime?
There is no allegation in the record of such a justification. Parenthetically, it may be observed
that under the Revised Rule 113, Section 5(b), the officer making the arrest must have personal
knowledge of the ground therefor as stressed in the recent case of People v. Burgos.
If follows that as the search of the petitioners' premises was violative of the Constitution,
all the firearms and ammunition taken from the raided compound are inadmissible in evidence in
any of the proceedings against the petitioners. These articles are "fruits of the poisonous tree. As
Judge Learned Hand observed, "Only in case the prosecution which itself controls the seizing
officials, knows that it cannot profit by their wrong, will the wrong be repressed. Pending
determination of the legality of such articles, however, they shall remain in custodia legis,
subject to such appropriate disposition as the corresponding courts may decide.
The objection to the photographing, fingerprinting and paraffin-testing of the petitioners
deserves slight comment. The prohibition against self-incrimination applies to testimonial
compulsion only. As Justice Holmes put it in Holt v. United States, "The prohibition of
compelling a man in a criminal court to be a witness against himself is a prohibition of the use of
physical or moral compulsion to extort communications from him, not an exclusion of his body
as evidence when it may be material."
The fearful days of hamleting salvaging, "zona" and other dreaded operations should
remain in the past, banished with the secret marshals and their covert license to kill without trial.
We must be done with lawlessness in the name of law enforcement. Those who are supposed to
uphold the law must not be the first to violate it. As Chief Justice Claudio Teehankee stressed in
his concurring opinion in Lacanilao v. De Leon, "It is time that the martial law regime's legacy of
the law of force be discarded and that there be a return to the force and rule of law."
All of us must exert efforts to make our country truly free and democratic, where every
individual is entitled to the full protection of the Constitution and the Bill of Rights can stand as
a stolid sentinel for all, the innocent as well as the guilty, including the basest of criminals.
2. PAPA v. MAGO, 22 SCRA 857
FACTS:
Ricardo Papa, Chief of Police of Manila; Juan Ponce Enrile, Commissioner of Customs;
Pedro Pacis, Collector of Customs of the Port of Manila; and Martin Alagao, a patrolman of the
Manila Police Department, against Remedios Mago and Hon. Hilarion Jarencio, Presiding Judge
of Branch 23 of the Court of First Instance of Manila, that the order issued by Mago which
authorized the release under bond of certain goods which were seized and held by Papa, et. al. in
connection with the enforcement of the Tariff and Customs Code, but which were claimed by
Mago.
Alagao, head of the counter-intelligence unit of the Manila Police Department, acting
upon a reliable information that a certain shipment of personal effects, allegedly misdeclared and
undervalued, would be released the following day from the customs zone of the port of Manila
and loaded on two trucks, and upon orders of Ricardo Papa, Chief of Police of Manila and a duly
deputized agent of the Bureau of Customs, conducted surveillance at gate No. 1 of the customs
zone. When the trucks left gate No. 1 at about 4:30 in the afternoon of November 4, 1966,
elements of the counter-intelligence unit went after the trucks and intercepted them at the
Agrifina Circle, Ermita, Manila. The load of the two trucks consisting of nine bales of goods, and
the two trucks, were seized on instructions of the Chief of Police. Upon investigation, a person
claimed ownership of the goods and showed to the policemen a "Statement and Receipts of
Duties Collected issued by the Bureau of Customs in the name of Bienvenido Naguit.
Claiming to have been prejudiced by the seizure and detention of the two trucks and their
cargo, Remedios Mago and Valentin B. Lanopa alleged thatMago was the owner of the goods
seized, having purchased them from the Sta. Monica Grocery in San Fernando, Pampanga; that
she hired the trucks owned by Valentin Lanopa to transport, the goods from said place to her
residence at Sampaloc, Manila; that the goods were seized by members of the Manila Police
Department without search warrant issued by a competent court; that Papa denied the request of
counsel for Remedios Mago that the bales be not opened and the goods contained therein be not
examined; that then Customs Commissioner Jacinto Gavino had illegally assigned appraisers to
examine the goods because the goods were no longer under the control and supervision of the
Commissioner of Customs; that the goods, even assuming them to have been misdeclared and,
undervalued, were not subject to seizure under Section 2531 of the Tariff and Customs Code
because Mago had bought them from another person without knowledge that they were imported
illegally; that the bales had not yet been opened, although Papa had arranged with the
Commissioner of Customs regarding the disposition of the goods, and that unless restrained their
constitutional rights would be violated and they would truly suffer irreparable injury..
When the restraining order was received by petitioners, some bales had already been
opened by the examiners of the Bureau of Customs in the presence of officials of the Manila
Police Department, an assistant city fiscal and a representative of Mago.
Respondent Judge issued an order releasing the goods to Mago upon her filing of a bond
in the amount of P40,000.00.

ISSUE: whether or not, the respondent Judge had acted with jurisdiction in issuing the order of
releasing the goods in question.

HELD: The goods in question were imported from Hongkong. As long as the importation
has not been terminated the imported goods remain under the jurisdiction of the Bureau of
customs. Importation is deemed terminated only upon the payment of the duties, taxes and other
charges upon the articles, or secured to be paid, at the port of entry and the legal permit for
withdrawal shall have been granted.
The record shows that the estimated duties, taxes and other charges on the goods subject of
this case amounted to P95,772.00 and that the duties, taxes and other charges had not been paid
in full and that the quantity of the goods was underdeclared, to avoid the payment of duties The
articles contained in the nine bales in question, were, therefore, subject to forfeiture under
Section 2530, pars. e and m, (1), (3), (4), and (5) of the Tariff and Customs Code. And this Court
has held that merchandise, the importation of which is effected contrary to law, is subject to
forfeiture, and that goods released contrary to law are subject to seizure and forfeiture.
Even if it be granted, arguendo, that after the goods in question had been brought out of the
customs area the Bureau of Customs had lost jurisdiction over the same, nevertheless, when said
goods were intercepted at the Agrifina Circle on November 4, 1966 by members of the Manila
Police Department, acting under directions and orders of their Chief, Ricardo C. Papa, who had
been formally deputized by the Commissioner of Customs, the Bureau of Customs had regained
jurisdiction and custody of the goods. Section 1206 of the Tariff and Customs Code imposes
upon the Collector of Customs the duty to hold possession of all imported articles upon which
duties, taxes, and other charges have not been paid or secured to be paid, and to dispose of the
same according to law. The goods in question, therefore, were under the custody and at the
disposal of the Bureau of Customs The Court of First Instance of Manila, therefore, could not
exercise jurisdiction over said goods even if the warrant of seizure and detention of the goods for
the purposes of the seizure and forfeiture proceedings had not yet been issued by the Collector of
Customs.
It is the settled rule that the Bureau of Customs acquires exclusive jurisdiction over
imported goods, for the purposes of enforcement of the customs laws, from the moment the
goods are actually in its possession or control, even if no warrant of seizure or detention had
previously been issued by the Collector of Customs in connection with seizure and forfeiture
proceedings. The court did not acquire jurisdiction over the goods in question when the petition
for mandamus was filed before it, and so there was no need of divesting it of jurisdiction. Not
having acquired jurisdiction over the goods, it follows that the Court of First Instance of Manila
had no jurisdiction to issue the questioned order of releasing said goods.
Respondents also aver that petitioner Martin Alagao, an officer of the Manila Police
Department, could not seize the goods in question without a search warrant. This contention
cannot be sustained. The Chief of the Manila Police Department, Ricardo G. Papa, having been
deputized in writing by the Commissioner of Customs, could, for the purposes of the
enforcement of the customs and tariff laws, effect searches, seizures, and arrests, and it was his
duty to make seizure, among others, of any cargo, articles or other movable property when the
same may be subject to forfeiture or liable for any fine imposed under customs and tariff
laws. He could lawfully open and examine any box, trunk, envelope or other container wherever
found when he had reasonable cause to suspect the presence therein of dutiable articles
introduced into the Philippines contrary to law; and likewise to stop, search and examine any
vehicle, beast or person reasonably suspected of holding or conveying such article as aforesaid.
The Tariff and Customs Code does not require said warrant in the instant case. The Code
authorizes persons having police authority under Section 2203 of the Tariff and Customs Code to
enter, pass through or search any land, inclosure, warehouse, store or building, not being a
dwelling house; and also to inspect, search and examine any vessel or aircraft and any trunk,
package, or envelope or any person on board, or to stop and search and examine any vehicle,
beast or person suspected of holding or conveying any dutiable or prohibited article introduced
into the Philippines contrary to law, without mentioning the need of a search warrant in said
cases. Except in the case of the search of a dwelling house, persons exercising police authority
under the customs law may effect search and seizure without a search warrant in the enforcement
of customs laws.

2. ROLDAN VS. ARCA 65 SCRA 336


FACTS:
ARCA filed against petitioner Fisheries Commissioner Arsenio N. Roldan, Jr., for the recovery
of fishing vessel Tony Lex VI (one of two fishing boats in question) which had been seized and
impounded by theCommissioner through the Philippine Navy.
On July 20, 1965, Fisheries Commissioner requested the Philippine Navy to apprehend
vessels Tony Lex VI and Tony Lex III, also respectively called Srta. Winnie and Srta. Agnes, for
alleged violations of some provisions of the Fisheries Act and the rules and regulations
promulgated thereunder.
On August 5 or 6, 1965, the two fishing boats were actually seized for illegal fishing with
dynamite. Fish caught with dynamite and sticks of dynamite were then found aboard the two
vessels.
On October 2 and 4, likewise, the Court of First Instance of Palawan ordered the
Philippine Navy to take the boats in custody.
Respondents alleged that at the time of the seizure of the fishing boats in issue, the same
were engaged in legitimate fishing operations off the coast of Palawan; that by virtue of the offer
of compromise by respondent company to the Secretary of Agriculture and Natural Resources,
the numerous violations of the Fishery Laws, if any, by the crew members of the vessels were
settled.

HELD:
Herein petitioners can validly direct and/or effect the seizure of the vessels of private
respondent for illegal fishing by the use of dynamite and without the requisite licenses. Section 4
of Republic Act No. 3512 approved on March 20, 1963 empowers the Fisheries Commissioner to
carry out the provisions of the Fisheries Act, to make searches and seizures personally or through
his duly authorized representatives in accordance with the Rules of Court, of "explosives such as
... dynamites and the like ...; including fishery products, fishing equipment, tackle and other
things that are subject to seizure under existing fishery laws"; and "to effectively implement the
enforcement of existing fishery laws on illegal fishing."
Paragraph 5 of Section 4 of the same Republic Act 3512 likewise transferred to and
vested in the Philippine Fisheries Commission "all the powers, functions and duties heretofore
exercised by the Bureau of Customs, Philippine Navy and Philippine Constabulary over fishing
vessels and fishery matters ..."
Section 12 of the Fisheries Act, otherwise known as Republic Act No. 4003, as amended,
prohibits fishing with dynamites or other explosives which is penalized by Section 76 thereof
"by a fine of not less than P1,500.00 nor more than P5,000.00, and by imprisonment for not less
than one (1) year and six (6) months nor more than five (5) years, aside from the confiscation
and forfeiture of all explosives, boats, tackles, apparel, furniture, and other apparatus used in
fishing in violation of said Section 12 of this Act." Section 78 of the same Fisheries Law
provides that "in case of a second offense, the vessel, together with its tackle, apparel, furniture
and stores shall be forfeited to the Government."
The second paragraph of Section 12 also provides that "the possession and/or finding, of
dynamite, blasting caps and other explosives in any fishing boat shall constitute a presumption
that the said dynamite and/or blasting caps and explosives are being used for fishing purposes in
violation of this Section, and that the possession or discovery in any fishing boat or fish caught
or killed by the use of dynamite or other explosives, under expert testimony, shall constitute a
presumption that the owner, if present in the fishing boat, or the fishing crew have been fishing
with dynamite or other explosives."
Under Section 78 of the Fisheries Act, any person, association or corporation fishing in
deep sea fishery without the corresponding license prescribed in Sections 17 to 22 Article V of
the Fisheries Act or any other order or regulation deriving force from its provisions, "shall be
punished for each offense by a fine of not more than P5,000.00, or imprisonment, for not more
than one year, or both, in the discretion of the Court; Provided, That in case of an association or
corporation, the President or manager shall be directly responsible for the acts of his employees
or laborers if it is proven that the latter acted with his knowledge; otherwise the responsibility
shall extend only as far as fine is concerned: Provided, further, That in the absence of a known
owner of the vessel, the master, patron or person in charge of such vessel shall be responsible for
any violation of this Act: and Provided, finally, That in case of a second offense, the vessel
together with its tackle, apparel, furniture and stores shall be forfeited to the Government"
(Emphasis supplied).
Under Section 13 of Executive Order No. 389 of December 23, 1950, reorganizing the
Armed Forces of the Philippines, the Philippine Navy has the function, among others, "to assist
the proper governmental agencies in the enforcement of laws and regulations pertaining to ...
fishin
Section 2210 of the Tariff and Customs Code, as amended by PD No. 34 of October 27,
1972, authorized any official or person exercising police authority under the provisions of the
Code, to search and seize any vessel or air craft as well as any trunk, package, bag or envelope
on board and to search any person on board for any breach or violation of the customs and tariff
laws.
When the Philippine Navy, upon request of the Fisheries Commissioner, apprehended on
August 5 or 6, 1965 the fishing boats Tony Lex III and Tony Lex VI, otherwise known
respectively as Srta. Agnes and Srta. Winnie, these vessels were found to be without the
necessary license in violation of Section 903 of the Tariff and Customs Code and therefore
subject to seizure under Section 2210 of the same Code, and illegally fishing with explosives and
without fishing license.
The two fishing boats were apprehended on numerous occasions for fishing with
dynamite from March 28, 1963 to March 11, 1964, which violations private respondent, as
owner-operator, sought to compromise by offering to pay a fine of P21,000.00 for all said prior
violations. Such previous violations of Sections 12, 17 and 18 of the Fisheries Act committed by
the two fishing boats, Tony Lex III and Tony Lex VI, from March 28, 1963 until August 5 or 6,
1965, rendered the said vessels subject to forfeiture under Sections 76 and 78 of the Fisheries
Act, as amended.
Search and seizure without search warrant of vessels and air crafts for violations of the
customs laws have been the traditional exception to the constitutional requirement of a search
warrant, because the vessel can be quickly moved out of the locality or jurisdiction in which the
search warrant must be sought before such warrant could be secured; hence it is not practicable
to require a search warrant before such search or seizure can be constitutionally effected. The
same exception should apply to seizures of fishing vessels breaching our fishery laws. They are
usually equipped with powerful motors that enable them to elude pursuing ships of the Philippine
Navy or Coast Guard.
Under our Rules of Court, a police officer or a private individual may, without a warrant,
arrest a person (a) who has committed, is actually committing or is about to commit an offense in
his presence; (b) who is reasonably believed to have committed an offense which has been
actually committed; or (c) who is a prisoner who has escaped from confinement while serving a
final judgment or from temporary detention during the pendency of his case or while being
transferred from one confinement to another . In the case at bar, the members of the crew of the
two vessels were caught in flagrante illegally fishing with dynamite and without the requisite
license. Thus their apprehension without a warrant of arrest while committing a crime is lawful.
Consequently, the seizure of the vessel, its equipment and dynamites therein was equally valid as
an incident to a lawful arrest.
2. PEOPLE VS CFI RIZAL
101 SCRA 86
FACTS:
On February 9, 1974, a light blue Dodge car with Plate No. 21-87-73, driven by Sgt.
Jessie Hope who was accompanied by Monina Medina approached the exit gate and after giving
the toll receipt sped away towards Manila. The RASAC agents gave a chase and overtook Sgt.
Hope's car. Agent Sabado blew his whistle and signaled Sgt. Hope to stop but the latter instead
of heeding, made a U-turn back to the North Diversion Road, but he could not go through
because of the buses in front of his car. At this point, the agents succeeded in blocking Sgt.
Hope's car and the latter stopped. Manuel and Sabado who were in civilian clothes showed their
Identification cards to respondents and introduced themselves as RASAC agents.
An inspection of Sgt. Hope's car at Camp Aguinaldo yielded 11 sealed boxes, 4 on the
rear seat and seven 7 more in the baggage compartment which was opened on orders of Col.
Abad. On the same order of the intelligence officer, the boxes were opened before the presence
of respondents Hope and Medina, representatives of the Bureau of Internal Revenue, Bureau of
Customs, P.C., COSAC and photographers of the Department of National Defense. The contents
of the boxes revealed some 4,441 more or less wrist watches of assorted brands; 1,075 more or
less watch bracelets of assorted brands supposedly untaxed.
As consequence, thereof, ASAC Chairman General Pelagio Cruz requested the Bureau of
Customs to issue a Warrant of Seizure and Detention against the articles including the Dodge
car. The Collector of Customs did issue the same on February 12, 1974. It was admitted,
however, that when the apprehending agents arrested respondents and brought them together
with the seized articles to the ASAC Office in Camp Aguinaldo, the former were not armed with
a warrant of arrest and seizure.
The lower court held that in accordance with Article IV, Sec. 4, paragraph 2 of the
present Constitution, the boxes and the watches and bracelets contained therein seized from the
car of the accused Sgt. Jessie C. Hope, are inadmissible in evidence in this case.
ISSUE:
Whether or not the seizure of the merchandise in a moving vehicle by authorized agents
commissioned to enforce customs laws without warrant of seizure breaches the constitutional
immunity against unreasonable search and seizure and therefore, such merchandise are
inadmissible in evidence.
HELD:
The State holds on the proposition that the rules governing search and seizure had been
liberalized when a moving vehicle is the object of the search and the necessity of a prior warrant
has been relaxed on the ground of practicality, considering that before a warrant could be
obtained, the place, things and persons to be searched must be described to the satisfaction of the
issuing judge a requirement which borders on impossibility in the case of smuggling effected
by the use of a moving vehicle that can transport contraband from one place to another with
impunity. Petitioner vigorously contends that contraband may be seized without necessity of a
search warrant since the Constitution does not guaranty immunity to smugglers and that a
warrantless seizure of contraband in a moving vehicle is justified by the traditional exception
attached to the Fourth Amendment of the U.S. Constitution, and such exception must be adopted
in interpreting the relevant provision in the new Philippine Constitution.
We have already seen that what they did was a faithful performance of a duty authorized
under the Tariff and Customs Code directing them as authorized agents to retrieve articles
reasonably suspected of having been possessed, issued or procured in violation of the tariff laws
for which the government has a direct interest. The official capacity of the agents has never been
questioned by respondents. Neither did respondents raise an issue on the constitutionality of the
law giving the agents the power to act as mandated. There 'is no question that the Agents have
not exceeded their authority nor have they acted so licentiously to bear upon respondents moral
embarrassment or substantial prejudice beyond what is necessary. The purpose of the search and
seizure is more than clear. Hence, the suspicion that the intention is only to elicit evidence to be
used against respondents is ruled out.

2. PACIS VS PAMARAN
56 SCRA 16

FACTS:
Respondent Ricardo Santos is the owner of a Mercury automobile, model 1957, brought into the
country without payment of customs duty and taxes because its original owner, Donald James
Hatch, was tax-exempt. Santos later on paid P311.00 for customs duty and taxes.
On July 22, 1964, Acting Collector of Customs Pedro Pacis was informed by the General Affairs
Administration of the Department of National Defense that the automobile was a hot car. By
virtue thereof, Pacis, through his subordinates, looked into the records of his office and found
that although the amount of P311.00 was already paid for customs duty, the amount collectible
on the said car should be P2,500.00, more or less.
Based on such discrepancy, he instituted seizure proceedings and issued a warrant of seizure and
detention. The automobile was also taken by the Department of National Defense agents and
brought to the General Affairs Administration for compound.
In answer, Santos filed a criminal complaint against Pacis for usurpation of judicial functions
with the City Fiscal of Manila, Manuel Pamaran, alleging that Pacis did not have authority to
issue such warrant of seizure and detention.

ISSUE:
W/N petitioner, in the discharge of his official function, lay himself open to a criminal
prosecution for usurpation of judicial functions.

HELD:
It is undeniable that petitioner, as Acting Collector of Customs for the Port of Manila, Pacis had
the requisite authority for the issuance of the contested warrant of seizure and detention for the
automobile owned by respondent Ricardo Santos. What was done by him certainly could not be
the basis of a prosecution for the usurpation of judicial functions.
2. Nolasco vs Pano: 147 SCRA 509 & 139 SCRA 152
139 SCRA 152 (1985)
On August 6, 1984, Mila Aguilar-Roque, et al. were arrested by Constabulary Security
Group. After their arrest, elements of the CSG searched the premises at 239-B Mayon St, QC
wherein 431 items were seized by said CSG elements. Prior to their arrest and the search and
seizure, Mila Aguilar-Roque was one of the accused of Rebellion in criminal case # MC-25-113
of the Military Commission No. 25 wherein she was still at large. In the case, the search and
seizure done was decided to be invalid and was set aside and annulled, however, the SC ruled
that with respect to Mila Aguilar-Roque, the personalities seized under the invalid search cannot
be returned by the CSG for they are supposedly to be used for the introduction of evidence in the
crime of rebellion against Aguilar.
147 SCRA 509 (1987)
Two years after the promulgation of the decision, Mila Aguilar-Roque et al. filed their partial
motion for reconsideration assailing the portion of the 1985 decision holding that in so far as
Aguilar-Roque was concerned, the search made in the premises was incident to her arrest and
could be made without a search warrant. They assail that a warrantless search can be justified
why it is an incident to a lawful arrest and that since Mila Aguilar was not lawfully arrested, a
search without warrant could not be made. They maintain that the arrest of Nolasco et al. and the
search of their premises thereafter are both illegal and that the personalities should be ordered
returned to their owners.
In this case, the Supreme Court used the dissenting opinion of Justice Teehankee and ruled that
the search warrant in the 1985 case should be set aside and annulled and that the personalities
seized should be returned. Justice Teehankee dissented in the 1985 decision that all articles
seized under the exclusionary rule (second paragraph of sec 3, article III of the Constitution)
totally, unqualifiedly and cannot be used against Nolasco et al.
PEOPLE VS CLAUDIO, 160 SCRA 646
Facts: Anita Claudio was found guilty beyond reasonable doubt by the RTC of Olongapo City
for violating Sec 4, RA 6425. She was transporting 1.1 kilograms of marijuana dried leaves for
the purpose of selling them from Baguio City to Olongapo City.
On July 21, 1981,Anita Claudio boarded a Victory Liner Bus. She took the seat in front of Daniel
Obina, who was with the Anti-Narcotics Unit. Anita put her bag behind the seat of Obina. Her
bag was a woven buri bag which from sight contained some vegetables. Accordingly, on the trip,
Obina recognized the smell of marijuana which he found out was emanating from Claudios bag.
When the bus reached Olongao City, Claudio alighted and Obina intercepted her. He showed his
ID identifying himself as a policeman and that he told her he needed to search her bag because of
the suspicion that she was carrying marijuana. She resisted but Obina took her to the police
station where her bag was searched in the presence of two police officers, Obina and
Claudioherself. Truly, there was a bag of marijuana.
Issue: W/N the search was valid.
Held: Yes. The applicable provisions on this issue are found in the 1985 Rules on Criminal
Procedure.
Rule 113, Sec. 5(a) of the said Rules provides:
.. A peace officer or a private person may, without a warrant, arrest a person:
(a) When, in his presence, the person to be arrested has committed, is actually committing, or
is attempting to commit an offense.
Meanwhile, its Rule 126, Sec. 12 provides:
Section 12. Search incident to lawful arrest. A person lawfully arrested may be searched for
dangerous weapons or anything which may be used as proof of the commission of an offense,
without a search warrant. (12a)
Appellant Claudio was caught transporting prohibited drugs. Pat. Daniel Obia did not need a
warrant to arrest Claudio as the latter was caught in flagrante delicto. The warrantless search
being an incident to a lawful arrest is in itself lawful.Therefore, there was no infirmity in the
seizure of the 1.1 kilos of marijuana.
People vs. Joselito del Rosario GR 127755, April 14, 1999
FACTS
The accused-appellant was convicted of the robbery with homicide and sentenced to death. The
conviction of the accused was based on the testimony of a tricycle driver who claimed that the
accused was the one who drove the tricycle, which the suspects used as their get-away vehicle.
The accused was then invited by the police for questioning and he pointed to the location where
he dropped off the suspects. When the police arrived at the supposed hide-out, a shooting
incident ensued, resulting to the death of some of the suspects.
After the incident, the accused was taken back to the precint where his statement was taken on
May 14, 1996. However, this was only subscribed on May 22, 1996 and the accused was made
to execute a waiver of detention in the presence of Ex-Judge Talavera. It was noted that the
accused was handcuffed through all this time upon orders of the fiscal and based on the
authorities' belief that the accused might attempt to escape otherwise.
ISSUES
Whether the Miranda rights of the accused-appellant were violated.
Whether the warrantless arrest of the accused-appellant was lawful.
HELD
YES. It was established that the accused was not apprised of his rights to remain silent and to
have competent and independent counsel in the course of the investigation. The Court held that
the accused should always be apprised of his Miranda rights from the moment he is arrested by
the authorities as this is deemed the start of custodial investigation. In fact, the Court included
invitations by police officers in the scope of custodial investigations.
It is evident in this case that when the police invited the accused-appellant to the station, he was
already considered as the suspect in the case. Therefore, the questions asked of him were no
longer general inquiries into an unsolved crime, but were intended to elicit information about his
participation in the crime.
However, the Miranda rights may be waived, provided that the waiver is voluntary, express, in
writing and made in the presence of counsel. Unfortunately, the prosecution failed to establish
that the accused made such a waiver.
NO. There are certain situations when authorities may conduct a lawful warrantless arrest: (a)
when the accused is caught in flagrante delicto; (b) when the arrest is made immediately after
the crime was committed; and when the one to be arrested is an escaped convict. The arrest
of the accused in this case did not fall in any of these exceptions. The arrest was not conducted
immediately after the consummation of the crime; rather, it was done a day after. The authorities
also did not have personal knowledge of the facts indicating that the person to be arrested had
committed the offense because they were not there when the crime was committed. They
merely relied on the account of one eyewitness.
Unfortunately, athough the warrantless arrest was not lawful, this did not affect the jurisdiction of
the Court in this case because the accused still submitted to arraignment despite the illegality of
his arrest. In effect, he waived his right to contest the legality of the warrantless arrest.
HARVEY VS MIRRIAM DEFENSOR SANTIAGO
Facts:
Petitioners were among the 22 suspected alien pedophiles who were apprehended after
three months close surveillance by the Commission on Immigration and Deportation (CID)
agents in Pagsanjan Laguna. Two days after apprehension 17 opted for self deportation,
one released for lack of evidenced, one was charged by another offense, working without a
valid working visa. Thus, three was left to face the deportation proceedings.
Seized during petitioners apprehension were rolls of photo negatives and photos of the
suspected child prostitute shown in salacious poses as well as boys and girls engaged in the
sexual act. There were also posters and other literatures advertising the child prostitution.
Warrant of arrest was issued by respondent against petitioners for violation of Sec. 37,
45 and 46 of the Immigration act and sec. 69 of the revised administrative Code.
Issue:
Whether or not the Philippines immigration act clothed the commissioner with any
authority to arrest and detained petitioner pending determination of the existence of a probable
cause
Ruling:
The Supreme Court held that there can be no question that the right against
unreasonable search and seizure is available to all persons, including aliens, whether accused
of a crime or not.
One of the constitutional requirement of a valid search warrant or warrant of arrest is that
it must be based upon probable cause.
The arrest of petitioners was based on probable cause determined after close
surveillance for three (3) months during which period their activities were monitored. The
existence of probable cause justified the arrest and the seizure of the photo negatives,
photographs and posters without warrant. Those articles were seized as an incident to a lawful
arrest and, are therefore, admissible in evidence.
But even assuming arguendo that the arrest of petitioners was not valid at its inception,
the records show that formal deportation charges have been filed against them, as undesirable
aliens.
That petitioners were not "caught in the act" does not make their arrest illegal. Petitioners
were found with young boys in their respective rooms, the ones with John Sherman being
naked. Under those circumstances the CID agents had reasonable grounds to believe that
petitioners had committed "pedophilia" defined as "psychosexual perversion involving children"

May a non-judicial officer issue a warrant off arrest?

2. Hortencia Salazar v. Hon Tomas Achacoso

FACTS: Rosalie Tesoro, in a sworn statement filed with the Philippine Overseas Employment
Administration (POEA for) charged petitioner Hortencia Salazar, an illegal recruiter. Atty.
Ferdinand Marquez to whom said complaint was assigned, sent to the petitioner the a telegram
ordering the petitioner to appear before him. On the same day, having ascertained that the
petitioner had no license to operate a recruitment agency, public respondent Administrator
Tomas D. Achacoso(secretary of labor)issued CLOSURE AND SEIZURE ORDER. Closure of
the recruitment agency and and the seizure of the documents and paraphernalia being used or
intended to be used as the means of committing illegal recruitment. The order was implemented.
ISSUE: May the Philippine Overseas Employment Administration (or the Secretary of Labor)
validly issue warrants of search and seizure (or arrest) under Article 38 of the Labor Code?
Section 38, paragraph (c) The Minister of Labor or his duly authorized representative shall have
the power to recommend the arrest and detention of any person engaged in illegal recruitment.
HELD: For the guidance of the bench and the bar, the court reaffirm the following principles:
1. Under Article III, Section 2, of the l987 Constitution, it is only judges, and no other, who
may issue warrants of arrest and search:
2. The exception is in cases of deportation of illegal and undesirable aliens, whom the
President or the Commissioner of Immigration may order arrested, following a final order of
deportation, for the purpose of deportation.
Article 38, paragraph (c) of the Labor Code is declared UNCONSTITUTIONAL and null and
void. The respondents are ORDERED to return all materials seized as a result of the
implementation of Search and Seizure Order.

THE PRESIDENTIAL ANTI-DOLLAR SALTING TASK FORCE vs. HONORABLE


COURT OF APPEALS
FACTS:Atty. Napoleon Gatmaytan of the Bureau of Customs who is a deputized member of the
Presidential Anti-Dollar Salting(PADS)Task Force, filed an application for issuance of search
warrant to prosecute foreign exchange violations against the petitioners Karamfil Import-Export
Co., Inc., P & B Enterprises Co., Inc. State Prosecutor Jose B. Rosales, who is assigned with the
Presidential Anti-Dollar Salting Task Force issued search warrants.
Rosales also issued search warrant to Philippine Veterans Corporation, Philippine Veterans
Development Corporation, Philippine Construction Development Corporation, Philippine Lauan
Industries Corporation, Inter-trade Development (Alvin Aquino), Amelili U. Malaquiok
Enterprises and Jaime P. Lucman Enterprises.
The lower court declared the warrant null and void and ordered to return and surrender
immediately all the personal properties and documents seized by them from the petitioners.
The Presidential Anti-Dollar Salting Task Force went to the respondent Court of Appeals to
contest that the petitioner is a special quasi-judicial body, ranks with RTCs, and with express
powers enumerated under PD 1936 to prosecute foreign exchange violations defined and
punished under P.D. No. 1883.
Also, Karamfil Import-Export Co., Inc. questioned primarily of whether or not the Presidential
Anti-Dollar Salting Task Force is "such other responsible officer' countenanced by the 1973
Constitution to issue warrants of search and seizure.
ISSUE : is PADS a judicial or quasi-judicial body authorized to issue a valid warrants and whose
decisions may not be challenged before the regular courts?
HELD: It must be observed that under the present Constitution, the powers of arrest and search
are exclusive upon judges. The Task Force was not meant to exercise quasi-judicial functions,
that is, to try and decide claims and execute its judgments because its undertaking is to determine
whether or not probable cause exists to warrant the filing of charges with the proper court,
meaning to say, to conduct an inquiry preliminary to a judicial recourse, and to recommend
action "of appropriate authorities". It is not unlike a fiscal's office that conducts a preliminary
investigation to determine whether or not prima facie evidence exists to justify haling the
respondent to court, and yet, while it makes that determination, it cannot be said to be acting as a
quasi-court. For it is the courts, ultimately, that pass judgment on the accused, not the fiscal.
A quasi-judicial body has been defined as "an organ of government other than a court and other
than a legislature, which affects the rights of private parties through either adjudication or rule
making."
THE PRESIDENTIAL ANTI-DOLLAR SALTING TASK FORCE vs. HONORABLE
COURT OF APPEALS
FACTS:Atty. Napoleon Gatmaytan of the Bureau of Customs who is a deputized member of the
Presidential Anti-Dollar Salting(PADS)Task Force, filed an application for issuance of search
warrant to prosecute foreign exchange violations against the petitioners Karamfil Import-Export
Co., Inc., P & B Enterprises Co., Inc. State Prosecutor Jose B. Rosales, who is assigned with the
Presidential Anti-Dollar Salting Task Force issued search warrants.
Rosales also issued search warrant to Philippine Veterans Corporation, Philippine Veterans
Development Corporation, Philippine Construction Development Corporation, Philippine Lauan
Industries Corporation, Inter-trade Development (Alvin Aquino), Amelili U. Malaquiok
Enterprises and Jaime P. Lucman Enterprises.
The lower court declared the warrant null and void and ordered to return and surrender
immediately all the personal properties and documents seized by them from the petitioners.
The Presidential Anti-Dollar Salting Task Force went to the respondent Court of Appeals to
contest that the petitioner is a special quasi-judicial body, ranks with RTCs, and with express
powers enumerated under PD 1936 to prosecute foreign exchange violations defined and
punished under P.D. No. 1883.
Also, Karamfil Import-Export Co., Inc. questioned primarily of whether or not the Presidential
Anti-Dollar Salting Task Force is "such other responsible officer' countenanced by the 1973
Constitution to issue warrants of search and seizure.
ISSUE : Is PADS a judicial or quasi-judicial body authorized to issue a valid warrants and
whose decisions may not be challenged before the regular courts?
HELD: It must be observed that under the present Constitution, the powers of arrest and search
are exclusive upon judges. The Task Force was not meant to exercise quasi-judicial functions,
that is, to try and decide claims and execute its judgments because its undertaking is to determine
whether or not probable cause exists to warrant the filing of charges with the proper court,
meaning to say, to conduct an inquiry preliminary to a judicial recourse, and to recommend
action "of appropriate authorities". It is not unlike a fiscal's office that conducts a preliminary
investigation to determine whether or not prima facie evidence exists to justify haling the
respondent to court, and yet, while it makes that determination, it cannot be said to be acting as a
quasi-court. For it is the courts, ultimately, that pass judgment on the accused, not the fiscal.
A quasi-judicial body has been defined as "an organ of government other than a court and other
than a legislature, which affects the rights of private parties through either adjudication or rule
making."

Properties subject to seizure


Section 2, Rule 126, 1985 Rules on Criminal Procedure

Sec. 2. Court where application for search warrant shall be filed. An application for search
warrant shall be filed with the following:
(a) Any court within whose territorial jurisdiction a crime was committed.
(b) For compelling reasons stated in the application, any court within the judicial region where
the crime was committed if the place of the commission of the crime is known, or any court
within the judicial region where the warrant shall be enforced.
However, if the criminal action has already been filed, the application shall only be made in the
court where the criminal action is pending.
Sec. 3. Personal property to be seized. A search warrant may be issued for the search and
seizure of personal property:
(a) Subject of the offense;
(b) Stolen or embezzled and other proceeds, or fruits of the offense; or
(c) Used or intended to be used as the means of committing an offense.

Espano vs CA
FACTS: On July 14, 1991, at about 12:30 a.m., Pat. Romeo Pagilagan and other police officers,
namely, Pat. Wilfredo Aquino, Simplicio Rivera, and Erlindo Lumboy of the Western Police
District (WPD), Narcotics Division went to Zamora and Pandacan Streets, Manila to confirm
reports of drug pushing in the area. They saw Rodolfo Espano selling something to another
person. After the alleged buyer left, they approached petitioner, identified themselves as
policemen, and frisked him. The search yielded two plastic cellophane tea bags of marijuana.
When asked if he had more marijuana, he replied that there was more in his house. The
policemen went to his residence where they found ten more cellophane tea bags of marijuana.
Petitioner was brought to the police headquarters where he was charged with possession of
prohibited drugs. On July 24, 1991, petitioner posted bail and the trial court issued his order of
release on July 29, 1991.

Contention of the Accused: The evidences against him are inadmissible. The evidences were
obtained through a search in his house but there was no search warrant.

ISSUE: WON the evidences were inadmissible and obtained in violation of his Constitutional
right.

HELD: The issue on the admissibility of the marijuana seized should likewise be ruled upon.
Rule 113 Section 5(a) of the Rules of Court provides:
A peace officer or a private person may, without a warrant, arrest a person:
a. when, in his presence, the person to be arrested has committed, is actually committing, or is
attempting to commit an offense;
x x x x x x x x x.
Petitioners arrest falls squarely under the aforecited rule. He was caught in flagranti as a result
of a buy-bust operation conducted by police officers on the basis of information received
regarding the illegal trade of drugs within the area of Zamora and Pandacan Streets, Manila.
The police officer saw petitioner handing over something to an alleged buyer. After the buyer
left, they searched him and discovered two cellophanes of marijuana. His arrest was, therefore,
lawful and the two cellophane bags of marijuana seized were admissible in evidence, being the
fruits of the crime.
As for the ten cellophane bags of marijuana found at petitioners residence, however, the same are
inadmissible in evidence.
The 1987 Constitution guarantees freedom against unreasonable searches and seizures
under Article III, Section 2 which provides:
The right of the people to be secure in their persons, houses, papers and effects against
unreasonable searches and seizures of whatever nature and for any purpose shall be inviolable,
and no search warrant or warrant of arrest shall issue except upon probable cause to be
determined personally by the judge after examination under oath or affirmation of the
complainant and the witnesses he may produce, and particularly describing the place to be
searched and the persons or things to be seized.
An exception to the said rule is a warrantless search incidental to a lawful arrest for
dangerous weapons or anything which may be used as proof of the commission of an offense. It
may extend beyond the person of the one arrested to include the premises or surroundings under
his immediate control. In this case, the ten cellophane bags of marijuana seized at petitioners
house after his arrest at Pandacan and Zamora Streets do not fall under the said exceptions.

12.3 Josefina Garcia-Padilla v. Minister Juan Ponce Enrile, Gen. Fabian Ver, Gen. Fidel
Ramos, and Lt. Col. Miguel Coronel

FACTS:

Fourteen detainees are involved in this case. Nine (9) of them were arrested on July 6, 1982 at
about 1:45 PM in the residence of Dra. Aurora Parong . They were then having a conference in
the dining room which had been going on since 10:00 AM of that same day. The nine included
the persons of Dra. Aurora Parong, Benjamin Pineda, Sabino Padilla, Francisco Divinagracia,
Zenaida Mallari, Letty Ballogan, Norberto Portuguese, and Mariano Soriano. Three (3) teams of
the Philippine Constabulary/INP of Bayombong, Nueva Ecija conducted the raid. The teams
were led by Lt. Col. Coronel, 1 Lt. de Guzman, and 1 Lt. Baria. They were able to secure a
st st

search warrant from Judge Sofronio Sayo of the Court of First Instance of Nueva Vizcaya.

The other four (4) detainees, which included Imelda de los Santos, Eufronio Ortiz, Jr., Juanita
Granada, and Bienvenida Garcia, were arrested on the following day. Eight days later, Tom
Vasquez was arrested, and his Volkswagen car was seized by the PC authorities.

They were all detained at the PC/INP Command Headquarters, Bayombong, Nueva Vizcaya
from July 6, 1982 until their transfer on the morning of August 10, 1982 to an undisclosed place
reportedly to Camp Crame, Quezon City, to Echague, Isabela, and to Tuguegarao, Cagayan.

The mother of Sabino Padilla, Josefina Garcia-Padilla, filed this petition for the writ of habeas
corpus. It was alleged in the petition:

1. That the arrest of the detainees was patently unlawful and illegal since it was effected
without any warrant of arrest;
2. That the raiding team which made the arrest were only armed with a search warrant ; and
nowhere in said warrant was authority given to make arrests, much less detention;
3. That the search warrant which authorized the teams to seize subversive documents,
firearms of assorted calibers, medicine, and other subversive paraphernalia in the house
and clinic of Dra. Parong was a roving and general warrant and is, therefore, illegal
because it does not state the things that are to be seized;
4. That no criminal charges as have as of yet been filed against the detainees;
5. That the fourteen (14) detainees were initially held at the PC/INP Command but were
subsequently transferred by helicopter to a place only known to respondents;
6. That there is no judgment, decree, decision, or order from a court of law which would
validate the continued detention of the detainees; and
7. That, while it is true that a purported telegram stating the issuance of a Presidential
Commitment Order (PCO) was shown to the detainees on or about July 11 and 12, 1982,
but counsel and the detainees have not yet been given a copy of such PCO nor notified of
its contents, raising a doubt whether such commitment order has in fact been issued.

It is further alleged that the respondents are denying the detainees their constitutional right to
counsel because before, they were allowed to be regularly visited by counsel and relatives. But
when control of the investigation was given to Major Cristobal and Lt. Marcos, counsels were
allowed to visit only weekends. The respondents seemed to be deliberately and with concerted
efforts concealing from counsel and relatives the detainees place of detention, raising the
apprehension that respondents are using force, violence, threat, intimidation, and other means
which vitiate free will to obtain confession and statements from the detainees in violation of their
constitutional rights.

ISSUE: Whether the petitioners detention is legal.

HELD:

Yes, their detention is legal. The SC finds that petitioners have not been illegally deprived of
their constitutional right to liberty, neither in the manner of their arrest, nor by their continued
detention, and that the circumstances attendant do not warrant their release on a writ of habeas
corpus.

At the time of the arrest of the nine (9) of the fourteen (14) detainees herein on July 6, 1982,
records reveal that they were then having conference in the dining room of Dra. Parong's
residence from 10:00 a.m. of that same day. Prior thereto, all the fourteen (14) detainees were
under surveillance as they were then Identified as members of the Communist Party of the
Philippines (CPP) engaging in subversive activities and using the house of detainee Dra. Aurora
Parong in Bayombong, Nueva Viscaya, as their headquarters. Caught in flagrante delicto, the
nine (9) detainees mentioned scampered towards different directions leaving in top of their
conference table numerous subversive documents, periodicals, pamphlets, books,
correspondence, stationaries, and other papers, including a plan on how they would infiltrate the
youth and student sector (code-named YORK). Also found were one (1) .38 cal. revolver with
eight (8) live bullets, nineteen (19) rounds of ammunition for M16 armalite, eighteen thousand
six hundred fifty pesos (P18,650.00) cash believed to be CPP/NPA funds, assorted medicine
packed and ready for distribution, as sizeable quantity of printing paraphernalia, which were then
seized. There is no doubt that circumstances attendant in the arrest of the herein detainees fall
under a situation where arrest is lawful even without a judicial warrant as specifically provided
for under Section 6(a), Rule 113 of the Rules of Court and allowed under existing jurisprudence
on the matter. As provided therein, a peace officer or a private person may, without a warrant,
arrest a person when the person to be arrested has committed or actually committing, or is about
to commit an offense in his presence.

From the facts as above narrated, the claim of the petitioners that they were initially arrested
illegally is, therefore, without basis in law and in fact. The crimes of insurrection or rebellion,
subversion, conspiracy or proposal to commit such crimes, and other crimes and offenses
committed in the furtherance on the occasion thereof, or incident thereto, or in connection
therewith under Presidential Proclamation No. 2045, are all in the nature of continuing offenses
which set them apart from the common offenses, aside from their essentially involving a massive
conspiracy of nationwide magnitude. Clearly then, the arrest of the herein detainees was well
within the bounds of the law and existing jurisprudence in our jurisdiction.

12.4 Maspil, Jr. v. People

FACTS:

On October 30, 1986, Sgt. Amador Ablang and Sgt. Florentino Baillo, members of the First
Narcotics Regional Unit of the Narcotics Command stationed in Baguio City, established a
checkpoint in front of the Municipal Hall at Sayangan, Atok, Benguet, which is along the
Halsema Highway, to check on vehicles proceeding to Baguio City. This is because their
Commanding Officer, Maj. Basilio Cablayan had been earlier tipped off by some confidential
informers that the herein accused Maspil and Basking would be transporting a large volume of
marijuana to Baguio City. The informers went along with the operatives to Sayangan.
At about 2:00 o'clock in the early morning of November 1, the operatives intercepted a Sarao
type jeep driven by Maspil with Bagking as his companion. Upon inspection, the jeep was found
loaded with two (2) plastic sacks, one (1) jute sack, and three (3) big round tin cans which, when
opened contained several bundles of suspected dried marijuana leaves.

Maspil and Basking were arrested and the suspected marijuana leaves were confiscated.

The accused admitted that the marijuana dried leaves were indeed confiscated from the jeep
being then driven by Maspil with Bagking as his helper. However, they claimed that the
prohibited drugs belonged to two of their passengers who loaded them in the jeep as paying
cargo for Baguio City without the accused knowing that they were marijuana.

Maspil and Bagking were taken to the townhall where they were allegedly maltreated to admit
ownership of the confiscated marijuana. At about 4:00 o'clock in the afternoon of November 1,
the soldiers took them away from Sayangan to be transferred to their station at Baguio City. On
their way, particularly at Km. 32 or 34, they met Mike Maspil, an elder brother of Moises
Maspil, and the soldiers called for him and then Lt. Valeroso and his men mauled him on the
road.

Mike testified that between 3:00 and 4:00 o'clock in the afternoon of November 1, he was
informed by a neighbor that his brother Moises was detained at the Atok Municipal Jail. So he
called for Jose Pos-el and James Longages, his driver and helper, respectively, to go along with
him to see Moises. They rode in his jeep. On the way, they met the group of Lt. Valeroso. For no
apparent reason, Lt. Valeroso boxed and kicked him several times. Thereafter, Lt. Valeroso
placed him under arrest together with his driver and helper. They were all brought to a shoe store
on Gen. Luna Road, Baguio City, together with Moises and Bagking. There, Lt. Valeroso got his
wallet containing P21,000 and Seiko wrist watch but the receipt was issued by a certain Miss
Pingil, a companion of Valeroso. He was released after nine days. He then went to Lt. Valeroso
to claim his wallet, money and watch but he was told that they were with Miss Pingil. However,
when he went to Miss Pingil, the latter said that the items were with Lt. Valeroso. He sought the
assistance of then Tourism Deputy Minister Honorato Aquino who assigned a lawyer to assist
him. The lawyer advised him to file a case against Lt. Valeroso but because of the intervening
congressional elections, the matter has never been pursued.

**The main defense of the appellants is their claim that the prohibited drugs belonged to their
two passengers who loaded them in the jeep as paying cargo without the appellants knowing that
the cargo was marijuana.

ISSUE: Whether the search was conducted within reasonable limits.

HELD:

The search was conducted within reasonable limits. There was information that a sizeable
volume of marijuana will be transported to take advantage of the All Saints Day holiday wherein
there will be a lot of people going to and from Baguio City. In fact, during the three day (October
30, 1986 to November 1, 1986) duration of the checkpoint, there were also other drug related
arrests made aside from that of the two appellants.

This case involves a search incident to a lawful arrest which is one of the exceptions to the
general rule requiring a search warrant. This exception is embodied in Section 12 of Rule 126 of
the 1985 Rules on Criminal Procedure which provides:

Sec. 12. Search incident to lawful arrest. A person lawfully arrested may be searched for
dangerous weapons or anything which may be used as proof of the commission of an offense,
without a search warrant.

and Rule 113, Section 5 (11) which state:

Sec. 5. Arrest without warrant; when lawful. A peace officer or a private person may,
without a warrant, arrest a person:

(a) When, in his presence, the person to be arrested has committed, is actually
committing, or is attempting to commit an offense.

This case falls squarely within the exceptions. The appellants were caught in flagrante delicto
since they were transporting the prohibited drugs at the time of their arrest. A crime was actually
being committed.
The appellants, however, cite the case of People V. Aminnudin. In said case, the PC officers
received information that the accused-appellant, on board a vessel bound for Iloilo City, was
carrying marijuana. When the accused-appellant was descending the gangplank, the PC officers
detained him and inspected the bag that he was carrying and found marijuana. The Court ruled
that since the marijuana was seized illegally, it is inadmissible in evidence.

There are certain facts of the said case which are not present in the case before us. In the
Aminnudin case, the records showed that there was sufficient time and adequate information for
the PC officers to have obtained a warrant. The officers knew the name of the accused, that the
accused was on board M/V Wilcon 9, bound to Iloilo and the exact date of the arrival of the said
vessel.

On the other hand, in this case there was no information as to the exact description of the vehicle
and no definite time of the arrival. A jeepney cannot be equated with a passenger ship on the
high seas. The ruling in the Aminnudin case, is not applicable to the case at bar.

The rule in civil as well as in criminal cases is that each party must prove his own affirmative
allegations. The prosecution avers the guilt of the accused who is presumed to be innocent until
the contrary is proved. Therefore, the prosecution must prove such guilt by establishing the
existence of all the elements of the crime charged. But facts judicially known, presumed,
admitted or confessed need not be proved.

12.5POSADAS VS CA

FACTS:
On October 16, 1986 at about 10:00 o'clock in the morning Pat. Ursicio Ungab and Pat. Umbra
Umpar, both members of the Integrated National Police (INP) of the Davao Metrodiscom
assigned with the Intelligence Task Force, were conducting a surveillance along Magallanes
Street, Davao City. While they were within the premises of the Rizal Memorial Colleges they
spotted petitioner carrying a "buri" bag and they noticed him to be acting suspiciously.
They approached the petitioner and identified themselves as members of the INP. Petitioner
attempted to flee but his attempt to get away was thwarted by the two notwithstanding his
resistance.They then checked the "buri" bag of the petitioner where they found one (1) caliber
.38 Smith & Wesson revolver with Serial No. 770196 1 two (2) rounds of live ammunition for a
.38 caliber gun 2 a smoke (tear gas) grenade, 3 and two (2) live ammunitions for a .22 caliber
gun. 4 They brought the petitioner to the police station for further investigation. In the course of
the same, the petitioner was asked to show the necessary license or authority to possess firearms
and ammunitions found in his possession but he failed to do so. Petitioner avers that there being
no lawful arrest or search and seizure, the items which were confiscated from the possession of
the petitioner are inadmissible in evidence against him.

ISSUE:W/not the warrantless arrest is valid

HELD:
An arrest without a warrant may be effected by a peace officer or private person, among others,
when in his presence the person to be arrested has committed, is actually committing, or is
attempting to commit an offense; or when an offense has in fact just been committed, and he has
personal knowledge of the facts indicating that the person arrested has committed it.

At the time the peace officers in this case identified themselves and apprehended the petitioner as
he attempted to flee they did not know that he had committed, or was actually committing the
offense of illegal possession of firearms and ammunitions. They just suspected that he was
hiding something in the buri bag. They did now know what its contents were. The said
circumstances did not justify an arrest without a warrant.
Thus, as between a warrantless search and seizure conducted at military or police checkpoints
and the search thereat in the case at bar, there is no question that, indeed, the latter is more
reasonable considering that unlike in the former, it was effected on the basis of a probable cause.
The probable cause is that when the petitioner acted suspiciously and attempted to flee with the
buri bag there was a probable cause that he was concealing something illegal in the bag and it
was the right and duty of the police officers to inspect the same.
The search in the case at bar can be sustained under the exceptions heretofore discussed, and
hence, the constitutional guarantee against unreasonable searches and seizures has not been
violated.

12.6 PEOPLE V DELACRUZ

FACTS:
A "buy-bust" operation was conducted by the 13th Narcotics Regional Unit through a team
composed of T/Sgt. Jaime Raposas as Team Leader, S/Sgt. Rodelito Oblice, Sgt. Dante Yang,
Sgt. Vicente Jimenez, P/Pfc. Adolfo Arcoy as poseur-buyer and Pat. Deogracias Gorgonia at
Maliclic St., Tondo, Manila at around 2:30 o'clock in the afternoon of May 4, 1987 to catch the
pusher/s. P/Pfc. Adolfo Arcoy acted as the poseur-buyer with Arnel as his companion to buy
marijuana worth P10.00 from the two accused, Juan de la Cruz and Reynaldo Beltran. At the
scene, it was Juan de la Cruz whom Arcoy first negotiated on the purchase and when Arcoy told
De la Cruz that he was buying P10.00 worth of marijuana, De la Cruz instructed Reynaldo
Beltran to give one aluminum foil of marijuana which Beltran got from his pants' pocket and
delivered it to Arcoy. After ascertaining that the foil of suspected marijuana was really
marijuana, Arcoy gave the prearranged signal to his teammates by scratching his head and his
teammates who were strategically positioned in the vicinity, converged at the place, identified
themselves as NARCOM agents and effected the arrest of De la Cruz and Beltran. The P10.00
marked bill used by Arcoy was found in the possession of Juan de la Cruz together with two
aluminum foils and containing marijuana.

ISSUE:
Is the Buy-Bust Operation being done to enforce Republic Act 6425 is unconstitutional and any
evidence acquired under such method should not be admissible in court.

HELD:
The Solicitor General explains that a buy-bust operation is the method employed by peace
officers to trap and catch a malefactor in flagrante delicto. It is essentially a form of entrapment
since the peace officer neither instigates nor induces the accused to commit a crime. Entrapment
is the employment of such ways and means for the purpose of trapping or capturing a lawbreaker
from whose mind the criminal intent originated. Oftentimes, it is the only effective way of
apprehending a criminal in the act of the commission of the offense.
While it is conceded that in a buy-bust operation, there is seizure of evidence from one's person
without a search warrant, needless to state a search warrant is not necessary, the search being
incident to a lawful arrest. A peace officer may, without a warrant, arrest a person when, in his
presence, the person to be arrested has committed, is actually committing or is attempting to
commit an offense. It is a matter of judicial experience that in the arrest of violators of the
Dangerous Drugs Act in a buy-bust operation, the malefactors were invariably caught red-
handed. There being no violation of the constitutional right against unreasonable search and
seizure, the confiscated articles are admissible in evidence.

PEOPLE OF THE PHILIPPINES, Plaintiff-Appellee, vs. ROMEO ORTIZ y BALLARES,


Accused-Appellant.
CRIME: Selling marijuana in violation of the Dangerous Drugs Act and sentenced to life
imprisonment plus a P20,000.00 fine.
FACTS:
The Narcotics Command Unit in Calamba, Laguna, having received information that a certain
Meo, later identified as Romeo Ortiz, was selling marijuana in Canlubang, organized a buy-bust
team for his apprehension. The team consisted of Sgt. Arnel Angsioco as leader and CIC Edgar
Groyon, Sgt. Agaton Enriquez and Pat. Heidi Zulueta as members. The operation was scheduled
on June 26, 1985, and was duly recorded in the station log book. So was the serial number of the
P20.00 to be used for the pretended purchase, which was delivered by Major Vivencio Ramilo,
the Unit Commander, to Angsioco. The latter marked it by dotting the nose on the picture of
President Quezon, then gave it to Groyon, who was to pose as the buyer. As a routinary
precaution, Ramilo made sure before they left that none of the team members was carrying any
marijuana.
The team arrived at the Labor Hall in Canlubang, Laguna, at about 9:15 in the morning and
waited for their informer. Upon his arrival, the operation began. The informer, at Groyon's
bidding, went to Ortiz and told him Groyon was looking for marijuana. The informer returned to
Groyon to tell him that Ortiz was willing to sell. Groyon then approached Ortiz and gave him the
marked bill in exchange for which Ortiz handed him two tea bags. At this juncture, Groyon gave
the pre-arranged signal by touching his hair. Angsioco and Zulueta, who were at the time about
four to eight meters away, pounced upon Ortiz and arrested him after introducing themselves as
peace officers. After initial resistance, Ortiz submitted. The team recovered the marked bill and
confiscated the tea bags. Ortiz was later taken to the police station for interrogation. The tea bags
were sealed in a letter envelope initiated by Groyon, Angsioco and Zulueta and sent to the
National Bureau of Investigation, where they were subjected to microscopic and chemical and
chromotographic examination. The contents were found to be marijuana.
Groyon and Ramilo testified on the details of the buy-bust operation, while the results of the
laboratory examination were reported under oath by Carolyo Custodio, supervisor and research
biologist of the NBI Forensic Chemistry Section. The P20.00 bill and the tea bags were among
the exhibits submitted by the prosecution.
Assessing the conflicting evidence of the parties, Judge Odilon I. Bautista of the Regional Trial
Court of Laguna opted in favor of the prosecution. The trial judge had the opportunity to directly
observe the witnesses on the stand and to assess their credibility, not only by the manner of their
testimony but also by the plausibility of the testimony itself.
CONTENTION OF THE ACCUSED:
The accused-appellant entered a flat denial and victim of frame-up and an unsuccessful extortion
attempt. He insisted that he was playing billiards at the Labor Hall that morning when Angsioco,
accompanied by Groyon and another man, suddenly approached him and clamped handcuffs on
his wrist. Angsioco then put his closed hand inside Ortiz's pocket and pretended to draw out a
P20.00 bill. He claims he was dragged to a car and told he was under arrest for pushing
marijuana ("Nagtutulak ka ng damo.") On their way to Narcom headquarters, Groyon offered to
drop the charge against him if he agreed to pay them P3,000.00. Ortiz says he demurred,
protesting he had no money and had not committed any offense. Groyon's reaction was to box
him several times in the chest. The accused-appellant said he was detained at the headquarters
for three days before he was taken to the municipal jail.
The accused-appellant submits that the prosecution version of the incident is not believable as he
would not have sold the prohibited drugs to a total stranger and at such a public place as a
billiard hall. He also faults the testimonies of the prosecution witnesses and cites certain
inconsistencies therein that he claims have rendered their veracity suspect. It is also his
contention that his arrest and search were unconstitutional, having been effected without warrant
in violation of the Bill of Rights.
HELD:
The accused-appellant was not exactly dealing with a total stranger as he was negotiating the sale
through the informer, who apparently had his confidence. The site of the sale was not that
antiseptic, so to speak, as it was a billiard hall, where shady transactions are not unheard of or
even unusual among the habitues of the place. As for the alleged contradictions of the
prosecution witnesses, we have repeatedly held that minor discrepancies do not necessarily
vitiate the essential veracity of testimony and in fact may even contribute to its credibility. A
witness cannot be expected to narrate past incidents with perfect recall. Also, there is no
evidence at all of the accused- appellant's claim of manhandling, which he did not complain
about to the fiscal who investigated him or to any physician who could have given him the
proper treatment and the corresponding medical certificate. We also find that he failed to prove
the alleged extortion attempt, which does not square with the fact that the buy-bust operation was
previously recorded in the log book.
Ortiz's submission that he was subjected to an illegal search and seizure is not acceptable. The
demonstrated facts are that he was arrested in flagrante and so came under the exception to the
rule requiring previous obtention of a warrant to justify a search or seizure. Rule 113, Section 5,
provides that a peace officer or indeed even a private person may, without warrant, arrest a
person who, "in his presence, has committed, is actually committing or is attempting to commit
an offense." In these circumstances, a search may also be made without warrant, being incidental
to a lawful arrest. Articles seized as a result of such lawful search are admissible in evidence.
The person accused of an offense is entitled to the constitutional presumption of innocence but
this right may be offset by proof of his guilt beyond reasonable doubt. That proof has been
established in the case at bar. We are convinced from the evidence before us that the accused-
appellant did sell the marijuana to the narcotics team on the day in question and so violated
Section 4 of the Dangerous Drugs Act. For contributing to the spread of drug addiction, which is
exacting a heavy toll on the lives and future of our people, he is justly punished.
WHEREFORE, the challenged decision is AFFIRMED and the appeal dismissed. It is so
ordered.

Go vs CA
G.R. No. 101837, February 11, 1992
Facts:
On 2 July 1991, Eldon Maguan, the victim, was driving his car along Wilson St. San, Juan. On
the other hand, Rolito Go, petitioner, entered Wilson St., which is a one-way street and started
travelling in the opposite or wrong direction. At the corner of Wilson and J. Abad Santos St.,
petitioner's and Maguan's cars nearly bumped each other. Petitioner then alighted from his car,
walked over and shot Maguan inside his car. Petitioner then boarded his car and left the scene. A
security at a nearby restaurant was able to take down the plate number of the car and the police
arrived shortly at the scene of the shooting to retrieve an empty shell and one round of live
ammunition (9 mm caliber pistol). The car was then verified to be registered under the name of
an Elsa Ang Go.
The police returned to the scene of the shooting the next day to find out where the petitioner
came from. The police were informed that petitioner had dined in Cravings Bakeshop shortly
before the shooting They obtained a facsimile or impression of the credit card used by petitioner
at the cashier. The security guard of the bakeshop was shown a picture of the petitioner and
positively identified the same as the person who shot Maguan. The police later on launched a
manhunt for petitioner.
On July 8, 1991, petitioner presented himself before the San Juan police, accompanied by two
lawyers, to verify news reports regarding the manhunt. The police detained him and an
eyewitness, who was at the police station that day, verified that petitioner was indeed the same
person who shot Maguan. The police filed a complaint for frustrated homicide. The prosecution
informed petitioner that he can avail of the right to preliminary investigation as long as he sign a
waiver of Article 125 of the RPC but petitioner refused to sign such waiver. Hence, no
preliminary investigation was conducted.
Petitioner would pray at the office of the Prosecutor for bail and contend that the warrantless
arrest was unlawful and no preliminary investigation was conducted. The provincial prosecutor
had granted such request and filed it with the RTC. The RTC had granted such, but the following
day had recalled the grant of bail and likewise cancelled the preliminary investigation asked for
by the petitioner.
Petitioner would file certiorari, prohibition and mandamus with the Supreme Court assailing the
July 17 Order that recalled his bail, cancelled his preliminary investigation, considered his earlier
immediate release (when his bail was granted) as a mere petition for bail, and set a hearing for
such. The motions were remanded to the Court of Appeals, but respondent judge issued an order
already setting the arraignment of petitioner. This arraignment pushed through on August 23,
1991 despite motion of petitioner to restrain the same. The criminal case commenced and
prosecution even presented witnesses.
The Court of Appeals denied the earlier motions of petitioner. It held that the warrantless arrest
was valid because the offense had been freshly committed and that his identity had been
established. It also held that he actually waived any irregularity of his arrest when he posted for
bail. This included waiving his right to a preliminary investigation. The OSG submitted in this
case that petitioner was validly arrested six days after commission of the offense and invoked the
case(s) Nazareno v Station Commander et al consolidated with In the Matter of the Petition for
Habeas Corpus of Roberto Umil etc v Ramos (together with seven other cases) where the Court
upheld the validity of the warrantless arrest even after 14 days from the time of the killing.
Petitioner argues that he was not lawfully arrested without warrant because he went to the police
station six (6) days after the shooting which he had allegedly perpetrated. Thus, petitioner
argues, the crime had not been just committed at the time that he was arrested. Moreover, none
of the police officers who arrested him had been an eyewitness to the shooting of Maguan and
accordingly none had the personal knowledge required for the lawfulness of a warrantless
arrest. Since there had been no lawful warrantless arrest, Section 7, Rule 112 of the Rules of
Court which establishes the only exception to the right to preliminary investigation, could not
apply in respect of petitioner.
Issue/s:
1. Whether or not a lawful warrantless arrest had been effected by the San Juan Police in
respect of petitioner Go;
2. Whether petitioner had effectively waived his right to preliminary investigation
Held:
1. No. The Court does not believe that the warrantless arrest or detention of petitioner in the
instant case falls within the terms of Section 5 of Rule 113 of the 1985 Rules on Criminal
Procedure which provides as follows:
Sec. 5. Arrest without warrant; when lawful. A peace officer or a private person may,
without a warrant, arrest a person;
(a) When, in his presence, the person to be arrested has committed, is actually committing, or is
attempting to commit an offense;
(b) When an offense has in fact just been committed, and he has personal knowledge of facts
indicating that the person to be arrested has committed it; and
(c) When the person to be arrested is a prisoner who has escaped from a penal establishment or
place where he is serving final judgment or temporarily confined while his case is pending, or
has escaped while being transferred from one confinement to another.
In cases falling under paragraphs (a) and (b) hereof, the person arrested without a warrant shall
be forthwith delivered to the nearest police station or jail, and he shall be proceeded against in
accordance with Rule 112, Section 7.
Petitioner was not arrested at all, as when he walked in the police station, he neither expressed
surrender nor any statement that he was or was not guilty of any crime.
Petitioners arrest took place six (6) days after the shooting of Maguan. The arresting
officers obviously were not present, within the meaning of Section 5(a), at the time petitioner
had allegedly shot Maguan. Neither could the arrest effected six (6) days after the shooting be
reasonably regarded as effected when [the shooting had] in fact just been committed within the
meaning of Section 5 (b). Moreover, none of the arresting officers had any personal
knowledge of facts indicating that petitioner was the gunman who had shot Maguan. The
information upon which the police acted had been derived from statements made by alleged
eyewitnesses to the shooting one stated that petitioner was the gunman; another was able to
take down the alleged gunmans cars plate number which turned out to be registered in
petitioners wifes name. That information did not, however, constitute personal knowledge.
It is thus clear to the Court that there was no lawful warrantless arrest of petitioner within the
meaning of Section 5 of Rule 113.
2. No. In the circumstances of this case, the Court does not believe that by posting bail, petitioner
had waived his right to preliminary investigation. In People v. Selfaison, the Court held that
appellants there had waived their right to preliminary investigation because immediately after
their arrest, they filed bail and proceeded to trial without previously claiming that they did not
have the benefit of a preliminary investigation.
In the instant case, petitioner Go asked for release on recognizance or on bail and for preliminary
investigation in one omnibus motion.
He had thus claimed his right to preliminary investigation before respondent Judge approved the
cash bond posted by petitioner and ordered his release on 12 July 1991. Accordingly, the Court
cannot reasonably imply waiver of preliminary investigation on the part of petitioner. In fact,
when the Prosecutor filed a motion in court asking for leave to conduct preliminary
investigation, he clearly if impliedly recognized that petitioners claim to preliminary
investigation was a legitimate one.
When a complaint was filed to the prosecutor, preliminary investigation should have
been scheduled to determine probable cause. Prosecutor made a substantive error, petitioner is
entitled to preliminary investigation, necessarily in a criminal charge, where the same is
required appear thereat.

Horacio Morales Jr vs Minister of Defense Juan Ponce Enrile et al


FACTS: 1. The petitions are without merit and are hereby DISMISSED.
2. Petitioners were arrested on April 21, 1982 at about 9:45 a.m. while they were riding together
in a motor vehicle on Laong-Laan Street, Quezon City, by elements of Task Force Makabansa of
the Armed Forces of the Philippines. Since their arrest, they have been under detention.
Petitioner Morales filed his petition for habeas corpus with this Court on July 9, 1982, while
petitioner Moncupa filed his on July 19, 1982. On July 20, 1982 petitioners, together with
several others, were charged with rebellion (Art. 134, Revised Penal Code) before the Court of
First Instance of Rizal in Criminal Case No. Q-21091 filed by the City Fiscal of Quezon City.
The trial of the case has yet to be terminated. The continued detention of petitioners to answer
for the offense charged is therefore legal.
3. Petitioners allege that they were arrested without any warrant of arrest; that their constitutional
rights were violated, among them the right to counsel, the right to remain silent, the right to a
speedy and public trial, and the right to bail. They also air the charge that they were subjected to
maltreatment and torture; that they did not have the opportunity to present their defense before
the inquest fiscal and therefore asked this Court to order the reinvestigation of the charges
against them. Acting on such plea, this Court in a resolution en banc dated July 22, 1982 ordered
the City Fiscal of Quezon City to conduct such reinvestigation and at the same time appointed
him to act as commissioner of this Court and receive evidence of the charges made by
petitioners before this Court of alleged torture and violation of their constitutional rights,
particularly the right to counsel. On September 28, 1982, the City Fiscal submitted his report on
the reinvestigation affirming the existence of a prima facie case for rebellion against petitioners
and several others. And on February 8, 1983 he submitted to this Court the transcript of the notes
taken at the reception of the evidence on the charges of petitioners.
4. If petitioners had been arrested in a communist country, they would have no rights to speak of.
However, the Philippines is a republican state. Sovereignty resides in the people and all
government authority emanates from them. We have a Constitution framed by a constitutional
1

convention and duly ratified by the people. We subscribe to the rule of law. We believe in human
rights and we protect and defend them. Petitioners are entitled to the full enjoyment of all the
rights granted to them by law. And this Court stands as the guarantor of those rights.
ISSUE: Whether or not Morales et al can post bail.
HELD: Normally, rebellion being a non-capital offense is bailable. But because the privilege of
the writ of habeas corpus remains suspended with respect to persons at present detained as well
as other who may hereafter be similarly detained for the crimes of insurrection or rebellion,
subversion, conspiracy or proposal to commit such crimes, and for all other crimes and offenses
committed by them in furtherance of or on the occasion thereof, or incident thereto, or in
connection therewith, the natural consequence is that the right to bail for the commission of
anyone of the said offenses is also suspended. To hold otherwise would defeat the very purpose
of the suspension. Therefore, where the offense for which the detainee was arrested is anyone of
the said offenses he has no right to bail even after the charges are filed in court. The crimes of
rebellion, subversion, conspiracy or proposal to commit such crimes, and crimes or offenses
committed in furtherance thereof or in connection therewith constitute direct attacks on the life
of the State. Just as an individual has right to self-defense when his life is endangered, so does
the State. The suspension of the privilege of the writ is to enable the State to hold in preventive
imprisonment pending investigation and trial those persons who plot against it and commit acts
that endanger the States very existence. For this measure of self-defense to be effective, the
right to bail must also be deemed suspended with respect to these offenses. However, there is a
difference between preventive and punitive imprisonment. Where the filing of charges in court or
the trial of such charges already filed becomes protracted without any justifiable reason, the
detention becomes punitive in character and the detainee regains his right to freedom. Quite
notable in this case however is that the 2 division of the SC reiterated the Lansang Doctrine as
nd

opposed to what they ruled in the Garcia-Padilla Case.

PEOPLE V. BURGOS - 144 SCRA 1


FACTS:
Cesar Masamlok personally and voluntarily surrendered to the authorities stating that he
was forcibly recruited by accused Ruben Burgos as member of the NPA, threatening him with
the use of firearm against his life, if he refused. Pursuant to this information, PC-INP members
went to the house of the Burgos and saw him plowing his field when they arrived. One of the
arresting officers called Burgos and asked him about the firearm. Burgos's wife pointed to a
place below their house where a gun was buried in the ground. After recovery of said firearm,
Burgos pointed to a stock pile of cogon where the officers recovered alleged subversive
documents. Burgos further admitted that the firearm was issued to him by Nestor Jimenez, team
leader of sparrow unit.

ISSUES: Is the warrantless arrest valid? Is the warrantless search valid?

HELD:
No. Under Section 6(a) of Rule 113, the officer arresting a person who has just
committed, is committing, or is about to commit an offense must have personal knowledge of
that fact. The offense must also be committed in his presence or within his view. There is no
such personal knowledge in this case. Whatever knowledge was possessed by the arresting
officers, it came in its entirety from the information furnished by Cesar Masamlok. The location
of the firearm was given by the wife of Burgos.
In this case, the Burgos was arrested on the sole basis of Masamlok's verbal report. Masamlok
led the authorities to suspect that the accused had committed a crime. They were still fishing for
evidence of a crime not yet ascertained. The subsequent recovery of the subject firearm on the
basis of information from the lips of a frightened wife cannot make the arrest lawful. If an arrest
without warrant is unlawful at the moment it is made, generally nothing that happened or is
discovered afterward can make it lawful. The fruit of a poisoned tree is necessarily also tainted.

People vs. dela Cruz [GR 83260, 18 April 1990]


FACTS:
After receiving a confidential report from Arnel, their informant, a "buy-bust" operation was
conducted and to catch the pusher/s. P/Pfc. Adolfo Arcoy acted as the poseur-buyer with Arnel
as his companion to buy marijuana worth P10.00 from the two accused, Juan de la Cruz and
Reynaldo Beltran. At the scene, it was Juan de la Cruz whom Arcoy first negotiated with on the
purchase and when Arcoy told De la Cruz that he was buying P10.00 worth of marijuana, De la
Cruz instructed Reynaldo Beltran to give one aluminum foil of marijuana which Beltran got from
his pants' pocket and delivered it to Arcoy. After ascertaining that the foil of suspected marijuana
was really marijuana, Arcoy gave the prearranged signal to his teammates by scratching his head
and his teammates who were strategically positioned in the vicinity, converged at the place,
identified themselves as NARCOM agents and effected the arrest of De la Cruz and Beltran. The
P10.00 marked bill used by Arcoy was found in the possession of Juan de la Cruz together with
two aluminum foils and containing marijuana.
ISSUE:
Whether the warrantless seizure incidental to the buy-bust operation violates Beltrans
constitutional rights against unreasonable search and seizure.
HELD:
A buy-bust operation is the method employed by peace officers to trap and catch a malefactor in
flagrante delicto. It is essentially a form of entrapment since the peace officer neither instigates
nor induces the accused to commit a crime. Entrapment is the employment of such ways and
means for the purpose of trapping or capturing a lawbreaker from whose mind the criminal intent
originated. Oftentimes, it is the only effective way of apprehending a criminal in the act of the
commission of the offense. While it is conceded that in a buy-bust operation, there is seizure of
evidence from one's person without a search warrant, needless to state a search warrant is not
necessary, the search being incident to a lawful arrest. A peace officer may, without a warrant,
arrest a person when, in his presence, the person to be arrested has committed, is actually
committing or is attempting to commit an offense. It is a matter of judicial experience that in the
arrest of violators of the Dangerous Drugs Act in a buy-bust operation, the malefactors were
invariably caught redhanded. There being no violation of the constitutional right against
unreasonable search and seizure, the confiscated articles are admissible in evidence.

Board of Commissioners (CID) v dela Rosa


May 31, 1991

FACTS:
Petition for certiorari and prohibition filed by the Solicitor General for the Board of
Commissioners of the Bureau of Immigration and Board of Special Inquiry to set aside two
orders issued by different judges of RTCs and to enjoin public respondent judges from acting on
the ff. civil cases:

1st case: filed by Gatchalian in the RTC of Manila. Judge dela Rosa issued an order that denied
the Motion to Dismiss and restrained petitioners from commencing or continuing with any
proceedings that will lead to the deportation of William Gatchalian.

2nd case: filed by Gatchalians wife and minor children in the RTC of Valenzuela. Judge
Capulong issued enjoining petitioners from proceeding with the deportation charges against
Gatchalian

July 12, 1960: Santiago Gatchalian, grandfather of William, was recognized by the Bureau of
Immigration as a native born Filipino Citizen. He also testified that he had 5 children with his
wife Chu Gim Tee: Jose, Gloria, Francisco (Williams father), Elena, and Benjamin.

June 27, 1961: Then 12-year old William arrived in Manila from Hongkong with Gloria,
Francisco, and Johnson Gatchalian with Certificates of Registration and Identity issued by the
Philippine Consulate in Hongkong based on a cablegram from the Secretary of Foreign Affairs.

July 12, 1961: the Board of Special Inquiry admitted William and his companions as Filipino
Citizens.

July 6, 1962: Board of Commissioners, after reviewing the decision of the Board of Special
Inquiry reversed the decision of the latter and ordered the exclusion of respondent Gatchalian

1973: Gatchalian and others covered by the warrant of exclusion filed a motion for re-hearing
with the Board of Special Inquiry. Acting Commissioner Nituda later issued an order recalling
the warrant of arrest against Gatchalian.
1990: acting director of NBI wrote to the DOJ recommending that Gatchalian and others covered
by the warrant of exclusion be charged with Violation of the Immigration Act. The SOJ
endorsed the recommendation and a mission order was issued by Commissioner Domingo of the
CID ordering the arrest of Gatchalian.

Gatchalian filed the present civil cases that are being assailed in the case at bar

Issue: Whether the arrest of respondent based on the warrant of exclusion is valid.
(NO)

Held:
Pursuant to the Immigration Act, an arrest can only be effected after a determination by the
Board of Commissioner of the existence of the ground for deportation as charged against the
alien.

(In relation to Sec. 37 (b) of the Immigration Act, the rule, therefore, is:)
Deportation or exclusion proceedings should be initiated within five (5) years after the cause of
deportation or exclusion arises...

Moreover, the mission ordered issued was only for purposes of investigation. Such mission
order/warrant of arrest made no mention that the same was issued pursuant to a final order of
deportation or warrant of exclusion.

In the case at bar, petitioners' alleged cause of action and deportation against herein respondent
arose in 1962. However, the warrant of arrest of respondent was issued by Commissioner
Domingo only on August 15, 1990 28 long years after. It is clear that petitioners' cause of
action has already prescribed and by their inaction could not now be validly enforced by
petitioners against respondent William Gatchalian. Furthermore, the warrant of exclusion dated
July 6, 1962 was already recalled and the Identification certificate of respondent, among others,
was revalidated on March 15, 1973 by the then Acting Commissioner Nituda.

The Court, therefore, holds that the period of effecting deportation of an alien after entry or a
warrant of exclusion based on a final order of the BSI or BOC are not imprescriptible. The law
itself provides for a period of prescription. Prescription of the crime is forfeiture or loss of the
rights of the State to prosecute the offender after the lapse of a certain time, while prescription of
the penalty is the loss or forfeiture by the government of the right to execute the final sentence
after the lapse of a certain time.

Such was the last official act of the government that is the basis of which respondent William
Gatchalian continually exercised the rights of a Filipino citizen to the present. Consequently, the
presumption of citizenship lies in favor of respondent William Gatchalian.
Also took note that the basis for the warrant of exclusion is that the cablegram issued by the
Secretary of Foreign Affairs was forged. Even if the applicants could have entered illegally, the
mere fact that they are citizens of the Philippines entitles them to remain in the country.

PEOPLE VS SUCRO, March 18, 1991

Facts:
Edison Sucro was charged with and convicted of violation of Section 4, Article II of the
Dangerous Drugs Act

March 21, 1989: Roy Fulgencio, a member of the INP, Kalibo, Aklan, was instructed by P/Lt.
Vicente Seraspi, Jr. (Station Commander of the INP Kalibo, Aklan) to monitor the activities of
appellant Edison Sucro, because of information gathered by Seraspi that Sucro was selling
marijuana.

As planned, Fulgencio positioned himself under the house of a certain Arlie Regalado. Adjacent
to the house of Regalado, about 2 meters away, was a chapel. Thereafter, Fulgencio saw
appellant enter the chapel, taking something which turned out later to be marijuana from the
compartment of a cart found inside the chapel, and then return to the street where he handed the
same to a buyer, Aldie Borromeo. After a while appellant went back to the chapel and again
came out with marijuana which he gave to a group of persons.

On the third buyer while transacting with appellant was reported and later identified as Ronnie
Macabante. From that moment, P/Lt.Seraspi proceeded to the area. While the police officers
were at the Youth Hostel in Maagama St.Fulgencio told Lt. Seraspi to intercept.

Macabante was intercepted at crossing in front of Aklan Medical center. Macabante saw the
police and threw a tea bag of marijuana on the ground. He admitted buying the marijuana from
Sucro in front of the chapel.

The police team intercepted and arrested SUCRO at the corner of C. Quimpo and Veterans.
Recovered were 19 sticks and 4 teabags of marijuana from a cart inside the chapel and another
teabag from Macabante.

Contention of the accused:


The accused-appellant contends that his arrest was illegal, being a violation of his rights granted
under Section 2, Article III of the 1987 Constitution. He stresses that there was sufficient time
for the police officers to apply for a search and arrest warrants considering that Fulgencio
informed his Station Commander of the activities of the accused two days before March 21,
1989, the date of his arrest.

Issue:
Whether or not arrest without warrant is lawful. (Yes)

Held:
Sucros contention is without merit.

Section 5, Rule 113 of the Rules on Criminal Procedure provides for the instances where arrest
without warrant is considered lawful. The rule states:
Arrest without warrant, when lawful. A peace officer or private person may, without warrant,
arrest a person:
a. When in his presence, the person to be arrested has committed, is actually committing, or
is attempting to commit an offense;
b. When an offense has in fact just been committed, and he has personal knowledge of facts
indicating that the person to be arrested has committed it;

(at first req.)


An offense is committed in the presence or within the view of an officer, within the meaning of
the rule authorizing an arrest without a warrant, when the officer sees the offense, although at a
distance, or hears the disturbances created thereby and proceeds at once to the scene thereof.
(at 2nd req)
Second requirement, the fact that Macabante, when intercepted by the police, was caught
throwing the marijuana stick and when confronted, readily admitted that he bought the same
from accused-appellant clearly indicates that Sucro had just sold the marijuana stick to
Macabante, and therefore, had just committed an illegal act of which the police officers had
personal knowledge, being members of the team which monitored Sucro's nefarious activity.

There are several instances when a warrantless search and seizure can be effected without
necessarily being preceded by an arrest provided the same is effected on the basis of probable
cause. Under the circumstances (monitoring of transactions) there existed probable cause for the
arresting officers, to arrest appellant who was in fact selling marijuana and to seize the
contraband.

That searches and seizures must be supported by a valid warrant is not an absolute rule. Among
the exceptions granted by law is a search incidental to a lawful arrest under Sec. 12, Rule 126 of
the Rules on Criminal Procedure, which provides that:
a person lawfully arrested may be searched for dangerous weapons or anything which may be
used as proof of the commission of an offense, without a search warrant.

As earlier discussed, there is nothing unlawful about the arrest considering its compliance with
the requirements of a warrantless arrest. Ergo, the fruits obtained from such lawful arrest are
admissible in evidence.

Premises considered, this Court is convinced that appellant Edison Sucro had indeed committed
the offense charged.

PEOPLE v NILO SOLAYAO, 262 SCRA 255

Facts:
On July 9, 1992, SPO3 Jose Nio with CAFGU members went to Barangay Caulangohan,
Caibiran, Biliran to conduct an intelligence patrol as required of them by their intelligence
officer to verify reports on the presence of armed persons roaming around the barangays of
Caibiran.

In Barangay Onion, they met the group of Nilo Solayao numbering five. The former became
suspicious when they observed that the latter were drunk and that Solayao was wearing a
camouflage uniform or a jungle suit. When Solayaos companions saw the government agents,
they fled.

Police Officer Nio told accused-appellant not to run away and introduced himself as "PC," after
which he seized the dried coconut leaves which the latter was carrying and found wrapped in it a
49-inch long homemade firearm locally known as "latong." When he asked accused-appellant
who issued him a license to carry said firearm or whether he was connected with the military or
any intelligence group, the latter answered that he had no permission to possess the same.
Thereupon, SPO3 Nio confiscated the firearm and turned him over to the custody of the
policeman of Caibiran who subsequently investigated him and charged him and later convicted
with illegal possession of firearm.
Solayao appealed claiming that the arrest and search do not fall under any of the circumstances
enumerated in Section 5, Rule 113 of the 1985 Rules on Criminal Procedure which provides that
a peace officer or a private person may, without a warrant, arrest a person when in his presence,
the person to be arrested has committed, is actually committing, or is attempting to commit an
offense.

Issue:
Whether the search made on his person violated his constitutional right against unreasonable
searches and seizures

Held:
No. There was no violation of the constitutional guarantee against unreasonable searches and
seizures. Nor was there error on the part of the trial court when it admitted the homemade
firearm as evidence.

In the present case, after SPO3 Nino told accused-appellant not to run away, the former
identified himself as a government agent. The peace officers did not know that he had
committed, or was actually committing, the offense of illegal possession of firearm. Tasked with
verifying the report that there were armed men roaming around in the barangays surrounding
Caibiran, their attention was understandably drawn to the group that had aroused their suspicion.
They could not have known that the object wrapped in coconut leaves which accused-appellant
was carrying hid a firearm.

However, Solayao is acquitted due to insufficiency of evidence.

PEOPLE vs. ANTOLIN CUIZON, STEVE PUA and PAUL LEE, 256 SCRA 325

Facts:
The NBI Reaction Group conducted surveillance on the drug activities of Cuizon and his wife. In
the morning of February 21, 1992, they received a report that the spouses was arriving on the
same day at NAIA in Pasay from Hong Kong, carrying with them a big quantity of shabu. Jose
Yap headed the NBI team with four other members from Narcotics Division and Reaction Group
who will intercept the suspects.

The spouses Cuizon arrived at noon while the team was already positioned separately in the
arrival and at the parking area of NAIA. While at the arrival area, Cuizon handed 4 travelling
bags to Steve Pua and Paul Lee. The four separated and Pua and Lee boarded a taxicab and they
proceeded to Manila Peninsula Hotel in Makati. The team failed to apprehend them because the
radio used by them ran out of battery power.

Upon arriving at about 2:00 p.m. in Manila Peninsula, the team coordinated with the Chief
Security Officer of the hotel. Two members of NBI were permitted to enter Room 340, the room
occupied by Pua and Lee and to also search their bags in the presence of one Cot. Regino
Arellano. The permission was made in writing. Three of the bags yielded 2.571 kilos, 2.768
kilos, and 2.970 kilos of methamphetamine hydrochloride.

The team proceeded to the house of accused Cuizon in Caloocan City, taking with them accused
Pua and Lee and the bags with their contents. They reached the place at about 5:50 p.m..
Retrieved from accused Cuizon in his residence was another bag also containing 2.695 kilos of
methamphetamine hydrochloride. Pua, Lee, Cuizon and his wife were then brought by the
arresting officers to the NBI headquarters for further investigation.

In the meantime, at about 5:30 p.m., roomboy of the Manila Peninsula, while cleaning Room
340, discovered in the ceiling a laundry bag containing suspected shabu of more than five (5)
kilos. He informed of such discovery and turned it over to the NBI.

The Trial Court found them guilty of violating Dangerous Drugs Act. Cuizon appealed, and in a
separate brief with Pua and Lee challenged the legality and validity of his warrantless arrest and
the search and seizure.

Issue:
Whether the warrantless arrests and the warrantless searches conducted by the NBI legal and
constitutional

Held:
No. Section 5 of Rule 113 of the Revised Rules of Court provides that an arrest without a
warrant may be lawfully made by a peace officer or a private person:
a) When, in his presence, the person to be arrested has committed, is actually committing, or is
attempting to commit an offense;
(b) When an offense has in fact just been committed, and he has personal knowledge of facts
indicating that the person to be arrested has committed it; and
(c) When the person to be arrested is a prisoner who has escaped from a penal establishment or
place where he is serving final judgment or temporarily confined while his case is pending, or
has escaped while being transferred from one confinement to another.
On the occasion of any of the aforementioned instances of legitimate arrest without warrant, the
person arrested may be subjected to a search of his body and of his personal effects or
belongings, for dangerous weapons or anything which may be used as proof of the commission
of an offense, likewise without need of a search warrant.

However, where a person is searched without a warrant, and under circumstances other than
those justifying a warrantless arrest, as discussed above, upon a mere suspicion that he has
embarked on some criminal activity, and/or for the purpose of discovering if indeed a crime has
been committed by him, then the search made of such person as well as his arrest are deemed
illegal.

In application, the search of the house of Cuizon, having been conducted without any warrant,
and not on the occasion or as an incident of a valid warrantless arrest, was indubitably illegal,
and the shabu seized thereat could not be admissible in evidence.

However, what has been said for Cuizon cannot be said for Pua and Lee. While the search and
arrest carried out on him and Lee may have been illegal for not being incident to a lawful
warrantless arrest, the unfortunate fact is that they failed to challenge the validity of his arrest
and search as well as the admission of the evidence obtained thereby; he did not raise the issue or
assign the same as an error before this Court. Accordingly, any possible challenge thereto based
on constitutional grounds is deemed waived.
Cuizon is acquitted while Pua and Lee are guilty of the crime of Illegal Transport of Regulated
Drugs.

G.R. No. 93516 August 12, 1992


THE PEOPLE OF THE PHILLIPPINES, plaintiff-appellee,
vs.
BASILIO DAMASO
Facts:
1.The accused-appellant, Basilio Damaso/Bernie Mendoza, was charged in an information filed
before the Regional Trial Court of Dagupan City with violation of Presidential Decree No. 1866
in furtherance of, or incident to, or in connection with the crime of subversion, having in his
possession, custody and control, one (1) M14 Rifle bearing Serial No. 1249935 with magazine
and Fifty-Seven (57) live ammunition.
2. On June 18, 1988, Lt. Candido Quijardo, a Philippine Constabulary officer connected with the
152nd PC Company at Lingayen, Pangasinan, and some companions were sent to verify the
presence of CPP/NPA members in Barangay Catacdang, Arellano-Bani, Dagupan City.
In said place, the group apprehended Gregorio Flameniano, Berlina Aritumba, Revelina Gamboa
and Deogracias Mayaoa. When interrogated, the persons apprehended revealed that there was an
underground safehouse at Gracia Village in Urdaneta, Pangasinan.
After coordinating with the Station Commander of Urdaneta, the group proceeded to the house in
Gracia Village. They found subversive documents, a radio, a 1 x 7 caliber .45 firearm and other
items
After the raid, the group proceeded to Bonuan, Dagupan City, and put under surveillance the
rented apartment of Rosemarie Aritumba, sister of Berlina Aritumba whom they earlier arrested.
They interviewed Luzviminda Morados, a visitor of Rosemarie Aritumba. She stated that she
worked with Bernie Mendoza, herein appellant. She guided the group to the house rented by
appellant. When they reached the house, the group found that it had already been vacated by the
occupants. Since Morados was hesitant to give the new address of Bernie Mendoza, the group
looked for the Barangay Captain of the place and requested him to point out the new house
rented by appellant. The group again required Morados to go with them. When they reached the
house, the group saw Luz Tanciangco outside. They told her that they already knew that she was
a member of the NPA in the area. At first, she denied it, but when she saw Morados she
requested the group to go inside the house. Upon entering the house, the group, as well as the
Barangay Captain, saw radio sets, pamphlets entitled "Ang Bayan," xerox copiers and a
computer machine. They also found persons who were companions of Luz Tanciangco (namely,
Teresita Calosa, Ricardo Calosa, Maries Calosa, Eric Tanciangco and Luzviminda Morados).
The group requested the persons in the house to allow them to look around. When Luz
Tanciangco opened one of the rooms, they saw books used for subversive orientation, one M-14
rifle, bullets and ammunitions, Kenwood radio, artificial beard, maps of the Philippines,
Zambales, Mindoro an(d) Laguna and other items. They confiscated the articles and brought
them to their headquarters for final inventory.
3. The counsel for accused-appellant interposed then his objections to the admissibility of the
prosecution's evidence on grounds of its being hearsay, immaterial or irrelevant and illegal for
lack of a search warrant.
Held:
The law enforcers failed to comply with the requirements of a valid search and seizure
proceedings.
The constitutional immunity from unreasonable searches and seizures, being personal one,
cannot be waived by anyone except the person whose rights are invaded or one who is expressly
authorized to do so in his or her behalf (De Garcia v. Locsin, 65 Phil. 689, 695). In the case at
bar, the records show that appellant was not in his house at that time Luz Tanciangco and Luz
Morados, his alleged helper, allowed the authorities to enter it (TSN, October 31, 1989, p. 10).
The fact that they came to the house of the appellant at nighttime does not grant them the license
to go inside his house.
G.R. No. 89139 August 2, 1990
ROMEO POSADAS y ZAMORA, petitioner,
vs.
THE HONORABLE COURT OF APPEALS and THE PEOPLE OF THE PHILIPPINES,
respondents.
Facts:
1. Pat. Ursicio Ungab and Pat. Umbra Umpar, both members of the Integrated National Police
(INP) of the Davao Metrodiscom assigned with the Intelligence Task Force, were conducting a
surveillance along Magallanes Street, Davao City. While they were within the premises of the
Rizal Memorial Colleges they spotted petitioner Posadas (below 18 y.o) carrying a "buri" bag
and they noticed him to be acting suspiciously.
2. They then checked the "buri" bag of the petitioner where they found one (1) caliber .38 Smit h
& Wesson revolver with Serial No. 770196 two (2) rounds of live ammunition for a .38 caliber
1

gun a smoke (tear gas) grenade, and two (2) live ammunitions for a .22 caliber gun.
2 3
3. He was then charged with illegal possession of firearms and ammunitions in the Regional
Trial Court of Davao City because of his failure to secure the necessary documents.
Held:
The assailed search and seizure may still be justified as akin to a "stop and frisk" situation whose
object is either to determine the identity of a suspicious individual or to maintain the status quo
momentarily while the police officer seeks to obtain more information
Clearly, the search in the case at bar, the constitutional guarantee against unreasonable searches
and seizures has not been violated..

Effect posting bail or entering a plea or failing to fila a motion to quash, if the arrest was illegal

6. People vs. Nazareno

[G.R. No. 103964. August 1, 1996]


PEOPLE OF THE PHILIPPINES, plaintiff-appellee, vs. NARCISO NAZARENO, RAMIL
REGALA, ORLANDO HULAR and MANUEL LAUREAGA, accused,NARCISO NAZARENO
and RAMIL REGALA, accused-appellants.

Facts: On December 28, 1988, Ramil Regala, Narciso Nazareno, Orlando Hular, and Manuel
Laurega were arrested for being pointed as assailants for the killing of one Romulo Molet Bunye
II. Bunye was then riding a tricycle when two men followed him and upon alighting to a corner
was shot which caused his death. All the accused were identified by the tricyle drivers, Fernando
Hernandez and Rogelio de Limos. Regala executed affidavits admitting his participation in the
killing and pointing to Nazareno and one Rey Taling as his co-conspirators. He claimed that
they've been hired by Hular to kill the victim and that they will be paid with P 300,000 by
Laurega. His testimony were then corroborated by Hular. However, Regala and Hular recanted.
Regala then stated that he was tortured but Hular was not. Accused-appellant Nazareno also
claimed to have been tortured. Hular and Laurega were acquitted while Nazareno and Regala
petitioned for an appeal. They claimed that their arrest warrants were illegal.

Issue: Whether or not their arrest warrants were illegal?

Held: No. The contention of the accused-appelants were untenable. It appears that Nazareno was
upheld by the court in a petition for habeas corpus after his motion to bail was denied. It was
alleged that his arrest was illegal because it was made without warrant 14 days after the killing of
Bunye. The court denied this on the ground that the arrest was made in compliance with Rule
113, 5 (b) of the Revised Ruled on Criminal Procedure. This question has long been settled by
this Court. Furthermore, Nazareno and Regala waived objections when they pleaded not guilty
and participated in the trial. Any defect in their arrest must be deemed cured when they
voluntarily submitted in the jurisdiction of this Court. Thus, the previous judgement was
affirmed

5. Filoteo vs Sandigabayan

[G.R. No. 79543. October 16, 1996]


JOSE D. FILOTEO, JR., petitioner, vs. SANDIGANBAYAN and THE PEOPLE OF THE
PHILIPPINES, respondents.

Facts: Petitioner Jose D. Filoteo Jr. was a police investigator of the Western Police District in
Metro Manila, an old hand in dealing with suspected criminals. Being a recipient of various
awards, he never imagined himself sitting on the other side of the investigation table for being a
suspected mastermind of armed hijacking of a post delivery van. He was the tried with his co-
accused Martin Mateo, Jr. y Mijares, PC/ Sgt. Bernardo Relator Jr., CID Ed Saguindel, Ex-
PC/Sgt. Danilo Miravalles and some 4 other civilians (Ricardo Perez, Reynaldo Frias, Raul
Mendoza, Angel Liwanag, Severino Castro, and Gerardo Escalada). The post delivery van is of
Bureau of Post and is picking-up and delivering mail matters from San Fernando, Pampanga to
Manila when the alleged act took place at Meycauyan, Bulacan. The van was then recovered at
La Loma, Quezon City. Petitioner claimed that he was illegally arrested since the officers invited
him without a warrant of arrest and brought him to Camp Crame where he was allegedly tortured
for almost a month.

Issue: Whether or not the petitioner was illegally arrested?

Held: Yes, but the petitioners claim is belatedly made. He should have questioned the validity of
his arrest before entered his plea in the trial court. The court explained People vs. Lopez Jr.
which stated that the legality of the arrest could no longer be questioned when the accused
already pleaded not guilty upon arraignment. In the present case, the only move made before the
plea with regards to the arrest was to file grave coercion, grave threat and maltreatment.

7. PEOPLE OF THE PHILIPPINES, plaintiff-appellee,


vs. DIONISIO LAPURA y CAJAN, accused-appellant
Crime: Murder of Petronilo Lim

FACTS: At 7:30 in the morning of February 19, 1988, Petronilo Lim (special agent of the
Criminal Investigation Service) was on board his car, with his sister, driving along Honorio
Lopez Blvd., Balut, Tondo, Manila. Just as he started slowing down the car before turning left to
Infanta Street, two persons suddenly came forward and fired at him. Lim, all bloodied tried to
fire back using his baby armalite. He died from 3 gunshot wounds.

The witness to the incident, Edgardo Samson identified Dionisio Lapura in a police line-up to be
the person who positioned himself at the left side of the victims car and who fired a .45 caliber
pistol at the victim.

Lapura was arrested by police officers at the house of a fellow musician, Danilo Cabrera (Lapura
is a combo drummer). He and Cabrera, along with another musician friend, Reynaldo Eliezer,
were brought to Station 1 at North Bay, Tondo, Manila, where statements were taken. Later, Col.
Maganto informed Lapura of his being a suspect in the killing of Lim because he resembled the
cartograph of the killer. Cabrera added that they were watching television when the arresting
policemen suddenly entered their house shouting, You are NPAs, no one must move.
Lapura contends that People vs. Opida where the Court exonerated the appellant for
nonobservance of certain of his constitutional rights as an accused should also be applied in
acquitting him. He alleges that

Like Opida and Marcelo in that case, the constitutional rights of herein
appellant have been grossly violated. From the very time that he was arrested
without warrant on February 25, 1988 and detained at WPD station, Tondo,
Manila in connection with the murder of Petronilo Lim, five days after its
perpetration on the flimsy ground that his face resembles that of man drawn in
a cartograph prepared by the police, perhaps based on the descriptions of
people who may have witnessed the crime, to the filing of the fatally flawed
information on March 2, 1988 or 7 days after his unlawful arrest, he was
deprived of his constitutional rights against unreasonable search and seizure, of
his right against arbitrary or unlawful arrest, of his right to remain silent and to
counsel and to be informed of said rights, of his right to a preliminary
investigation in a crime cognizable by the Regional Trial Court and finally of
his right to have a valid information against him filed within the period allowed
by law implied in Article 125 of the Revised Penal Code the provisions of
which he had never waived.
Issues: (1) Whether the supposed failure of the investigating prosecutor to obtain the prior
written authority of the city prosecutor in the manner required under Section 4, Rule, 112, of the
Rules of Court, before the filling of the case is consequential.
(2) WON the Opida ruling should also be applied to the appellant

(1) Held: No. Such discrepancy is immaterial.


The investigating fiscal certificated that: I hereby certify that an ex-parte investigation in this
case has been conducted by me in accordance with law; that there is reasonable ground to believe
that the offense charged has been committed; that the accused is probably guilty thereof and that
the filing of this information is with the prior authority and approval of the City Fiscal.
Absent convincing evidence to the contrary, the presumption of regularity in the performance of
official functions has to be upheld. Moreover, this matter should have been raised below in a
proper motion to quash that appellant could have done but did not. Settled is the rule that such
[

certification is not an indispensable part of, let alone invalidate even by its absence, an
information.

Sec. 4. Duty of investigating fiscal. - If the investigating fiscal finds cause to hold the respondent
for trial, he shall prepare the resolution and corresponding information. xxx
In passing, the question of whether or not a preliminary investigation has been properly
conducted is itself one that should be interposed prior to an arraignment. Here, appellant did not
do so before entering his plea of not guilty to the charge.

(2) Held: No, The Opida ruling does not apply to the accused.
Opida is predicated on two vital premises: (a) the trial judges palpable partiality, as well as the
irregular manner in which he conducted his interrogation of the accused and their witness, and
(b) the admission of an extrajudicial confession despite strong evidence of manhandling by the
police. These circumstances are not present in this case.
During trial, while the judge did propound questions to the witnesses, they clearly appear to
be clarificatory and certainly not adversarial in character. Relative to his alleged warrantless
arrest, he has waived, by filing a petition for bail any irregularity attendant thereto. Indeed, by his
application for bail, and by entering a plea of not guilty and then submitting to the proceedings
below, appellant must be deemed to have foregone his right to preliminary investigation and to
question any irregularity that might have attended such investigation.
The conviction of Lapura is affirmed.

8. People vs Silan
Facts:
Accused-appellants were prosecuted for the special complex crime of robbery with homicide.
Silan stated that she went to the house of her auntie Vangie in the late afternoon of 1 June 1992.
Her companions in going to Vangie's house were Virgilio "Billy" Garcia and a certain Tol. They
went to Vangie's house to get her things so they can sell them as Billy needed money. Upon
reaching the house of Vangie they found it closed. They went to the back portion where Billy
used a screw driver in destroying the padlock of the kitchen door. Once the kitchen door was
opened, they entered the house, Silan going to her room while Billy and Tol went upstair(s).
While Silan was arranging her things she heard a loud sound of objects falling. She went up the
house and found that an electric fan fell on the floor. She saw Billy and Tol searching something.
She also notice (sic) that the room was in disarray. It was at this juncture that Billy told her to get
things that she wants. She then got two jackets, coleman, colored blue dress of Remia, perfume,
make-up kit, t-shirt, girdle of Remia and plate with saucer. When they went down from the
second floor of the house Billy talked with her and forced her to go to her former room where he
forced her to have sex with him. It was at that instance when her auntie Vangie arrived and
knocked at the door. (p. 6, ibid.) Billy then ordered her to stand up in one corner and told her he
will take care of things. He then put off the light and opened the door and allowed her auntie to
come in. After her auntie entered the house there was a commotion. She heard her auntie
shouting "Who are you, why did you enter the house" followed by shouts for help "Aling Lina,
Aling Lina, tulungan ninyo ako, may gustong pumatay sa akin". Soon thereafter silence
followed. Then Billy returned for her and pulled her out the house.

Issue:
Appellant (Virgillio Garcia) contends that the court erred
(1) in relying on the testimony and extrajudicial confession of appellant Silan in convicting him;
(2) in failing to consider that appellant Silan had sufficient motive to implicate him in the crime
charged;
(3) in quickly dismissing his defense of alibi; and
(4) in not considering the illegality of his arrest by the police.
Held:
Appellant Garcia bewails and assigns as reversible error the failure of the trial court to take into
account his supposed illegal arrest which later culminated in the filing of the charges against
him. We note, however, that he never objected thereto nor placed that matter in issue when,
instead, he entered his plea on arraignment and went to trial. As the Solicitor General points out,
even assuming that he was illegally arrested, this will not affect his culpability since an
allegation of a warrantless arrest cannot deprive the State of its right to convict the guilty when
all the facts on record point to his culpability
We also agree with the advertence to our pronouncement in the aforecited case of Briones which
could very well be said of this incident in the case at bar:
Immediately after their arrest, appellants . . . could have objected to the legality thereof due to
the failure of the police officer to secure first a warrant for their arrest. Not only that, without
having questioned the legality of their arrest, they even pleaded, on arraignment, to the
information filed against them. Appellants' acts constitute a clear waiver of their right against
unlawful restraint of liberty. Besides, it would be impractical, if not ridiculous, to order the court
a quo to set the appellants free then issue a warrant for their arrest, and try them all over again
when appellants themselves have waived their right to object to such irregularity and when their
objection is truly based on overwhelming evidence.

Penalty for illegal arrest

Palon v NAPOLCOM (di ko mahanap sa internet. Subukan kong hanapin sa library)

P. Judicial Pronouncements on Illegally seized evidence (106 SCRA 336)

Historical Development of Rules on Illegally Seized Evidence


a. Doctrine of Non-exclusion of Illegally Seized Evidence
1920s: In People v Carlos (47 Phil 626), the Court held that illegally seized evidence is
admissible in evidence.
In 1946, through Alvero v Dizon, and later on in Moncado v Peoples Court, the SC
established the non-exclusionary doctrine, that admissibility of evidence is not affected
by the illegality of the means to obtain them.
4 Justices dissented in MoncadoBriones, Perfecto, Bengzon, Parasbut the doctrine
was further reiterated in subsequent cases.
In Weeks v US (1958), the court pronounced that even those illegally obtained papers, if
competent and relevant to the case, are admissible. Justice Roberto Concepcion dissented
in this case, averring that if such is the case, the peoples rights to be secure against
unreasonable searches would be of no value and it might as well be stricken from the
Constitution. His view would later on be the majority opinion in the 1967 case Stonehill
v Diokno which revoked the non-exclusionary rule. (Concepcion would also later on
become CJ.)

b. Doctrine of Exclusion of Illegally Seized Evidence


In Stonehill, the SC declared upon mature deliberation that the non-exclusionary
approach must be abandoned, as most common law jurisdictions have already given up
said doctrine. They realized that adopting the exclusionary rule is the only practical
means of enforcing the constitutional right against unreasonable searches and seizures,
and it is more in line with the spirit of the law.
If the applicant for a search warrant has no competent evidence, then it is not possible for
a judge to determine probable cause, and hence, there could be no justification for its
issuance. The only possible explanation for its issuance (without competent evidence) is
the necessity of a fishing expedition for evidenceindicative of the absence of proof
to establish probable cause.
It is to be noted that as early as the 1920s this principle has been introduced. In Uy
Kheytin v Villareal, the SC ruled that seizure of a mans private papers to be used as
evidence against him is equivalent to compelling him to be a witness against himself. In
People v Sy Juco, they also held that a search warrant cannot be used to fish for evidence.

Exceptions to the Rule of Exclusion of Illegally Seized Evidence


a. Articles prohibited by law where the purpose of confiscation is to prevent its use as a
means of further violation of the law (Castro v Pabalan [1976])
b. Owner of illegally seized evidence admitted guilt where the owner confessed guilt in
committing a crime (People v Agbot [1981])

Q. The Exclusionary Rule (155 SCRA 494)

This rule states that evidence obtained in violation of the accuseds constitutional rights against
unreasonable searches and seizures (in other words, evidence illegally obtained) is inadmissible.

Constitutional provision:
Section 2. The right of the people to be secure in their persons, houses, papers, and effects
against unreasonable searches and seizures of whatever nature and for any purpose shall be
inviolable, and no search warrant or warrant of arrest shall issue except upon probable cause to
be determined personally by the judge after examination under oath or affirmation of the
complainant and the witnesses he may produce, and particularly describing the place to be
searched and the persons or things to be seized.
Section 3. (1) The privacy of communication and correspondence shall be inviolable except upon
lawful order of the court, or when public safety or order requires otherwise, as prescribed by law.
(2) Any evidence obtained in violation of this or the preceding section shall be inadmissible
for any purpose in any proceeding.

Statutory provision:
Rule 126, Sec 4 of the Revised Rules of Court
Section 4. Requisites for issuing search warrant. A search warrant shall not issue except
upon probable cause in connection with one specific offense to be determined personally by the
judge after examination under oath or affirmation of the complainant and the witnesses he may
produce, and particularly describing the place to be searched and the things to be seized which
may be anywhere in the Philippines.

R. Search warrant for pirated video tapes

1. 20TH CENTURY FOX FILM CORPORATION vs.COURT OF APPEALS, EDUARDO


M. BARRETO, RAUL SAGULLO and FORTUNE LEDESMA
Facts: The petitioner questions the application of the constitutional provision against illegal
searches and seizures to raids conducted in connection with the government's anti-film piracy
campaign.
In a letter-complaint dated August 26, 1985, 20th Century Fox Film Corporation through counsel
sought the National Bureau of Investigation's (NBI) assistance in the conduct of searches and
seizures in connection with the latter's anti-film piracy campaign. Specifically, the letter-
complaint alleged that certain videotape outlets all over Metro Manila are engaged in the
unauthorized sale and renting out of copyrighted films in videotape form which constitute a
flagrant violation of Presidential Decree No. 49 (otherwise known as the Decree on the
Protection of Intellectual Property).
The petitioner maintains that the lower court issued the questioned search warrants after finding
the existence of a probable cause justifying their issuance. According to the petitioner, the lower
court arrived at this conclusion on the basis of the depositions of applicant NBI's two witnesses
which were taken through searching questions and answers by the lower court.
The lower court, therefore, lifted the three (3) questioned search warrants in the absence of
probable cause that the private respondents violated P.D. 49. As found out by the court, the NBI
agents who acted as witnesses did not have personal knowledge of the subject matter of their
testimony which was the alleged commission of the offense by the private respondents. Only the
petitioner's counsel who was also a witness during the application for the issuance of the search
warrants stated that he had personal knowledge that the confiscated tapes owned by the private
respondents were pirated tapes taken from master tapes belonging to the petitioner. However,
the lower court did not give much credence to his testimony in view of the fact that the
master tapes of the allegedly pirated tapes were not shown to the court during the
application.
Issue: whether or not the judge properly lifted the search warrants he issued earlier
Ruling: Yes. The presentation of the master tapes of the copyrighted films from which the
pirated films were allegedly copied, was necessary for the validity of search warrants
against those who have in their possession the pirated films. The petitioner's argument to the
effect that the presentation of the master tapes at the time of application may not be necessary as
these would be merely evidentiary in nature and not determinative of whether or not a probable
cause exists to justify the issuance of the search warrants is not meritorious. The court cannot
presume that duplicate or copied tapes were necessarily reproduced from master tapes that it
owns.
The application for search warrants was directed against video tape outlets which allegedly were
engaged in the unauthorized sale and renting out of copyrighted films belonging to the petitioner
pursuant to P.D. 49.
The essence of a copyright infringement is the similarity or at least substantial similarity of the
purported pirated works to the copyrighted work. Hence, the applicant must present to the court
the copyrighted films to compare them with the purchased evidence of the video tapes allegedly
pirated to determine whether the latter is an unauthorized reproduction of the former. This
linkage of the copyrighted films to the pirated films must be established to satisfy the
requirements of probable cause. Mere allegations as to the existence of the copyrighted films
cannot serve as basis for the issuance of a search warrant.

2. COLUMBIA PICTURES VS. CA, 261 SCRA 144

Facts:
The National Bureau of Investigation has engaged in an anti-film piracy drive by
investigating various video establishments in Metro Manila involving cases violating PD No. 49,
as amended. Columbia Pictures, et al. had lodged a formal complaint with the NBI, vis--vis
their anti-film piracy drive. Eventually, the NBI obtained from NBI Senior Agent Lauro C.
Reyes a search warrant against Sunshine Video, owned and operated by Danilo A. Pelindario
with address at No. 6 Mayfair Center, Magallanes, Makati, Metro Manila, seeking to seize
pirated video tapes of copyrighted films for being violative of PD No. 49.

The NBI carried out the seizure, found and seized various video tapes of duly
copyrighted motion pictures/films owned or exclusively distributed by Columbia Pictures, Inc. et
al (Columbia et al.), and filed a return with the trial court. However, the trial court eventually
granted a motion to lift the order of search warrant the contention was that the master tapes of
the copyrighted films from which the pirated films were allegedly copied were never
presented in the proceedings for the issuance of the search warrants.

The CA dismissed the appeal brought before it. Hence, Columbia Pictures, et al. brought
the case before the SC. Sunshine Video contended that Columbia Pictures, et al. (being foreign
corporations doing business in the Philippines) should have a license in order to maintain an
action in Philippine courts and without such license, it had no right to ask for the issuance of a
search warrant. Sunshine video submitted that the fact that Columbia Pictures, et al. were
copyright owners or owners of exclusive rights of distribution in the Philippines of copyrighted
motion pictures, AND the fact that Att. Domingo had been appointed as their atty.-in-fact
constituted doing business in the Philippines, under the Rules of the Board of Investments.As
foreign corporations doing business in the Philippines, Section 133 of Batas Pambansa Blg. 68,
or the Corporation Code of the Philippines, denies them the right to maintain a suit in Philippine
courts in the absence of a license to do business. Consequently, they have no right to ask for the
issuance of a search warrant.

Columbia et al denied that they are doing business in the Philippines and contend that
Sunshine have not adduced evidence to prove that petitioners are doing such business here, as
would require them to be licensed by the Securities and Exchange Commission. Moreover, an
exclusive right to distribute a product or the ownership of such exclusive right does not
conclusively prove the act of doing business nor establish the presumption of doing business.

Issue: Whether or not Columbia et al were doing business in the Philippines, thus, needs to be
licensed before having a legal standing in Philippine courts.

Ruling:

No, foreign film corporations do not transact or do business in the Philippines and,
therefore, do not need to be licensed in order to take recourse to our courts. As acts constitutive
of doing business, the fact that Columbia et al are admittedly copyright owners or owners of
exclusive distribution rights in the Philippines of motion pictures or films does not convert such
ownership into an indicium of doing business which would require them to obtain a license
before they can sue upon a cause of action in local courts. Neither is the appointment of Atty.
Rico V. Domingo as attorney-in-fact of Columbia et al., with express authority pursuant to a
special power of attorney
Held:
Based on Article 133 of the Corporation Code and gauged by such statutory standards,
Columbia et al are not barred from maintaining the present action. There is no showing that,
under our statutory or case law, Columbia et al are doing, transacting, engaging in or carrying on
business in the Philippines as would require obtention of a license before they can seek redress
from our courts. No evidence has been offered to show that petitioners have performed any acts
or any other specific act indicative of an intention to conduct or transact business in the
Philippines.

In the seizure of alleged pirated tapes, the applicant must submit to the court the master copy of
the allegedly pirated tape in order that the search warrant to be issued be valid. It should be
presented before the judge in order to convince him of the existence of probable cause.

You might also like